You are on page 1of 125

The Official SAT Online Course 页码,1/1

更多SAT考试资料,请登录http://sat.tiandaoedu.com获取
Help | Profile | My Organizer | My Bookmarks | Logout

Answers and Explanations

Test Sections Back to Score Report

Section 1 Essay
Section 2 Online - Practice Test #1

Section 3
These sample essays were originally handwritten by students but are shown typed here for ease
Section 5 of reading. The essays are displayed exactly as students wrote them, without any corrections to
Section 6 spelling, punctuation, or syntax. One handwritten sample essay is provided to illustrate the need
for legible and clear handwriting.
Section 7
Section 8
Section 9 Exemplars:
Section 10 Essay Prompt

Think carefully about the issue presented in the following excerpt and the assignment below.


We often hear that we can learn much about someone or something just by casual
observation. We are not required to look beneath the surface or to question how something
seems. In fact, we are urged to trust our impressions, often our first impressions, of how a


ed
person or a situation seems to be. Yet appearances can be misleading. What “seems” isn’
t always what is.


Is the way something seems to be not always the same as it actually is? Plan and write an
essay in which you develop your point of view on this issue. Support your position with
er
reasoning and examples taken from your reading, studies, experience, or observations.


t

is


eg

Back to Score Report

Copyright © 2006 The College Board. All rights reserved. Privacy Policy Terms of Use Contact Us


nR



U

file://E:\新建文件夹\a2.htm 2006-11-12
The Official SAT Online Course 页码,1/21

Help | Profile | My Organizer | My Bookmarks | Logout

Answers and Explanations

Test Sections Back to Score Report

Section 1 View Answers and Explanations


Section 2 Online - Practice Test #1

Section 3
1 Despite ------- on taking rare tamarins from their habitat, the illegal trade in the tiny
Section 5 monkeys remains -------.
Section 6
(A) commendations . . obligatory
Section 7
(B) consultations . . predominant
Section 8
Section 9 (C) restrictions . . local
Section 10 (D) penalties . . illicit

(E) prohibitions . . active

ANSWERS AND EXPLANATIONS




d
Explanation for Correct Answer E :
Choice (E) is correct. "Prohibition" refers to something banned by legal authorities;

re 用
"active" in this context means in place or functioning. If one were to insert these
terms into the text, the sentence would read "Despite prohibitions on taking rare
tamarins from their habitat, the illegal trade in the tiny monkeys remains active."
Since the word "despite" at the beginning of the sentence indicates that the second


te
missing term will describe an outcome of the first missing term, it makes sense to
say that although taking tamarins from their habitat is prohibited, the illegal activity
still goes on.


is


eg

Explanation for Incorrect Answer A :


Choice (A) is incorrect. "Commendations" means expressions of approval.
"Obligatory" means legally or morally binding. If one were to insert these terms into
nR

the text, the sentence would read "Despite commendations on taking rare tamarins


from their habitat, the illegal trade in the tiny monkeys remains obligatory." It is
illogical to say that the illegal traders receive the approval of authorities or that the
illegal trade is legally binding.


U

Explanation for Incorrect Answer B :


Choice (B) is incorrect. "Consultations" means deliberations or discussions.
"Predominant" means most common. If one were to insert these terms into the
text, the sentence would read "Despite consultations on taking rare tamarins from
their habitat, the illegal trade in the tiny monkeys remains predominant." Although
it is possible to have "consultations" regarding the trade in tamarins, it does not
logically follow to say the trade remains common in spite of these discussions.

Explanation for Incorrect Answer C :


Choice (C) is incorrect. "Restrictions" means limits; "local" means confined to a
certain place. If one were to insert these terms into the text, the sentence would
read "Despite restrictions on taking rare tamarins from their habitat, the illegal
trade in the tiny monkeys remains local." It is illogical to say that there are
limitations on taking tamarins from their habitat and that the illegal trade remains
confined to a particular place.

Explanation for Incorrect Answer D :


Choice (D) is incorrect. "Penalties" means punishments prescribed by law; "illicit"
means unlawful. If one were to insert these terms into the text, the sentence would
read "Despite penalties on taking rare tamarins from their habitat, the illegal trade
in the tiny monkeys remains illicit." It is not logical to say that the illegal trade in
tamarins remains unlawful despite the punishments imposed on those who engage
in it.

file://E:\新建文件夹\a3.htm 2006-11-12
The Official SAT Online Course 页码,2/21

2 Representing a round world on a flat surface is impossible without some -------: the
Mercator projection map shows Greenland as over ten times larger than Mexico, a
country in fact only slightly smaller than Greenland.
(A) oversight
(B) simplification
(C) distortion

(D) sophistication

(E) superficiality

ANSWERS AND EXPLANATIONS


Explanation for Correct Answer C :
Choice (C) is correct. "Distortion" means a lack of proportionality. If one were to
insert this term into the text, the sentence would read "Representing a round world
on a flat surface is impossible without some distortion: the Meractor projection map
shows Greenland as over ten times larger than Mexico, a country in fact only
slightly smaller than Greenland." The use of the colon in this sentence indicates that
what follows it will be an explanation or definition of the missing word, and since
the appearance of Greenland on the Mercator map clearly represents an instance of
a lack of proportionality, choice (C) makes sense.



d
Explanation for Incorrect Answer A :
Choice (A) is incorrect. "Oversight" means watchful care. If one were to insert this

re
term into the text, the sentence would read "Representing a round world on a flat


surface is impossible without some oversight: the Mercator projection map shows
Greenland as over ten times larger than Mexico, a country in fact only slightly
smaller than Greenland." The use of the colon in this sentence indicates that what


te
follows it will be an explanation or definition of the missing word; the appearance of
Greenland on the Mercator map does not represent a case of careful watchfulness.


is
Explanation for Incorrect Answer B :
Choice (B) is incorrect. "Simplification" means a lessening in scope or complexity. If
one were to insert this term into the text, the sentence would read "Representing a


eg

round world on a flat surface is impossible without some simplification: the


Mercator projection map shows Greenland as over ten times larger than Mexico, a
country in fact only slightly smaller than Greenland." The use of the colon in this


sentence indicates that what follows it will be an explanation or definition of the
missing word, and since the appearance of Greenland on the Mercator map
nR

represents not so much a lessening of complexity as a change in relative size,


choice (B) is incorrect.

Explanation for Incorrect Answer D :


Choice (D) is incorrect. "Sophistication" would in this context most likely mean an
U

increase in development or cultivation. If one were to insert this term into the text,
the sentence would read "Representing a round world on a flat surface is impossible
without some sophistication: the Mercator projection map shows Greenland as over
ten times larger than Mexico, a country in fact only slightly smaller than
Greenland." Since the appearance of Greenland on the Mercator map in no way
represents a heightened development or level of complexity, choice (D) is incorrect.

Explanation for Incorrect Answer E :


Choice (E) is incorrect. "Superficiality" means affecting only the surface. If one were
to insert this term into the text, the sentence would read "Representing a round
world on a flat surface is impossible without some superficiality: the Mercator
projection map shows Greenland as over ten times larger than Mexico, a country in
fact only slightly smaller than Greenland." Since the appearance of Greenland on
the Mercator map does not clearly represent a case of only the surface of
something being affected or observed, choice (E) does not make sense.

3 The highly publicized redesign of the car is essentially -------: the exterior has been
updated, but the engine remains unchanged.

(A) intuitive
(B) cosmetic
(C) incoherent

file://E:\新建文件夹\a3.htm 2006-11-12
The Official SAT Online Course 页码,3/21

(D) consequential

(E) retroactive

ANSWERS AND EXPLANATIONS


Explanation for Correct Answer B :
Choice (B) is correct. "Cosmetic" means superficial, or lacking in true significance. If
one were to insert this term into the text, the sentence would read "The highly
publicized redesign of the car is essentially cosmetic: the exterior has been
updated, but the engine remains unchanged." The colon in this sentence introduces
a definition or explanation of the missing term. "Cosmetic" makes sense in this
context, since the changes to the car's exterior are more superficial than
fundamental.

Explanation for Incorrect Answer A :


Choice (A) is incorrect. "Intuitive" means perceived through instinct. If this term
were inserted into the text, the sentence would read "The highly publicized redesign
of the car is essentially intuitive: the exterior has been updated, but the engine
remains unchanged." It makes little sense to say that the changes to a car's


exterior design are "intuitive," or based on instinct.

Explanation for Incorrect Answer C :


Choice (C) is incorrect. "Incoherent" means lacking orderliness or relevance. If one

d
were to insert this term into the text, the sentence would read "The highly
publicized redesign of the car is essentially incoherent: the exterior has been

re 用
updated, but the engine remains unchanged." It is illogical to say that the new
design, which changes the exterior but not the engine, is lacking relevance.


te
Explanation for Incorrect Answer D :
Choice (D) is incorrect. "Consequential" means significant. If one were to insert this
term into the text, the sentence would read "The highly publicized redesign of the


car is essentially consequential: the exterior has been updated, but the engine
is
remains unchanged." It does not make sense to say that the car's redesign is
significant because the information after the colon suggests the opposite.


eg

Explanation for Incorrect Answer E :


Choice (E) is incorrect. "Retroactive" means extending to conditions that existed or


originated in the past. If one were to insert this term into the text, the sentence
would read "The highly publicized redesign of the car is essentially retroactive: the
nR

exterior has been updated, but the engine remains unchanged." It makes little
sense to say that the car's redesign reflects conditions that originated in the past.



U

4 Many of our memories are -------, escaping our consciousness just as we strain to
recall a face or a name.

(A) elusive
(B) pervasive
(C) unvaried

(D) insensitive

(E) impractical

ANSWERS AND EXPLANATIONS


Explanation for Correct Answer A :
Choice (A) is correct. "Elusive" means hard to apprehend or find. If one were to
insert this term into the text, the sentence would read "Many of our memories are
elusive, escaping our consciousness just as we strain to recall a face or a name."
Because the use of the comma in the sentence indicates that what follows it will be
an explanation or definition of the missing word, it makes sense to say that
memories—because they often escape our consciousness at the moment when we
are trying to grasp them—are difficult to capture.

file://E:\新建文件夹\a3.htm 2006-11-12
The Official SAT Online Course 页码,4/21

Explanation for Incorrect Answer B :


Choice (B) is incorrect. "Pervasive" means tending to permeate, or to spread
throughout. If one were to insert this term into the text, the sentence would read
"Many of our memories are pervasive, escaping our consciousness just as we strain
to recall a face or a name." This does not make sense, since the use of the comma
in the sentence indicates that what follows will be an explanation or definition of the
missing word, and "pervasive" means nearly the opposite of "escaping our
consciousness."

Explanation for Incorrect Answer C :


Choice (C) is incorrect. "Unvaried" means uniform, or all of the same kind. If one
were to insert this term into the text, the sentence would read "Many of our
memories are unvaried, escaping our consciousness just as we strain to recall a face
or a name." It would not make sense here to speak of "uniform memories,"
certainly not of the tendency of such memories to escape, or elude, our
consciousness.

Explanation for Incorrect Answer D :


Choice (D) is incorrect. "Insensitive" means lacking in feeling or being tactless. If
one were to insert this term into the text, the sentence would read "Many of our
memories are insensitive, escaping our consciousness just as we strain to recall a
face or a name." It does not make sense to speak of memories as unfeeling or
tactless; in addition, the word "insensitive" is not explained or defined by the
material after the comma.


Explanation for Incorrect Answer E :
Choice (E) is incorrect. "Impractical" means incapable of being put to use. If one
were to insert this term into the text, the sentence would read "Many of our


memories are impractical, escaping our consciousness just as we strain to recall a

ed
face or a name." It is not logical to speak of memories as being useful or not useful;
moreover, the word "impractical" is not explained by the material after the comma.


er
5


Although Caroline Gordon was rigorously objective in her journalistic writing, her
t
lively and ------- private correspondence ------- a delightful capacity for biting


is
commentary on the social scene.

(A) incisive . . disguised


eg

(B) eloquent . . derided


(C) dispassionate . . demonstrated


(D) exuberant . . minimized
nR

(E) entertaining . . exhibited


ANSWERS AND EXPLANATIONS


U

Explanation for Correct Answer E :


Choice (E) is correct. "Entertaining" means lively or amusing; "exhibited" means
demonstrated. If one were to insert these terms into the text, the sentence would
read "Although Caroline Gordon was rigorously objective in her journalistic writing,
her lively and entertaining private correspondence exhibited a delightful capacity for
biting commentary on the social scene." The word "although" indicates that the part
of the sentence following the comma will contrast with the idea of "rigorous"
objectivity, and the use of "and" before the first missing term indicates that this
term will complement the word "lively." Therefore, it makes sense to say that
Gordon's amusing private letters, in contrast to her straightforward and detached
journalistic writing, demonstrated a pleasing subjective element.

Explanation for Incorrect Answer A :


Choice (A) is incorrect. "Incisive" means clear or sharp; "disguised" means hidden
or concealed. If one were to insert these terms into the text, the sentence would
read "Although Caroline Gordon was rigorously objective in her journalistic writing,
her lively and incisive private correspondence disguised a delightful capacity for
biting commentary on the social scene." Although it does make sense to say that
Gordon wrote "incisive" letters, it does not make sense to say that such a writing
style hid a capacity for satire. On the contrary, this type of writing would more
likely reveal such a capacity.

Explanation for Incorrect Answer B :

file://E:\新建文件夹\a3.htm 2006-11-12
The Official SAT Online Course 页码,5/21

Choice (B) is incorrect. "Eloquent" means vividly expressive; "derided" means


ridiculed or treated with contempt. If one were to insert these terms into the text,
the sentence would read "Although Caroline Gordon was rigorously objective in her
journalistic writing, her lively and eloquent private correspondence derided a
delightful capacity for biting commentary on the social scene." While it makes sense
to say that Gordon wrote expressive letters, it is illogical to say that such a writing
style "derided," or was contemptuous of, a talent for satirical writing.

Explanation for Incorrect Answer C :


Choice (C) is incorrect. "Dispassionate" means unemotional; "demonstrated" means
showed. If one were to insert these terms into the text, the sentence would read
"Although Caroline Gordon was rigorously objective in her journalistic writing, her
lively and dispassionate private correspondence demonstrated a delightful capacity
for biting commentary on the social scene." Gordon's letters may have showed a
capacity for satire, but letters cannot be both lively and "dispassionate."

Explanation for Incorrect Answer D :


Choice (D) is incorrect. "Exuberant" means extremely enthusiastic; "minimized"
means to reduce to the least possible degree. If one were to insert these terms into
the text, the sentence would read "Although Caroline Gordon was rigorously
objective in her journalistic writing, her lively and exuberant private correspondence
minimized a delightful capacity for biting commentary on the social scene." It is
possible that Gordon's letters were both lively and enthusiastic, but it does not
make sense to say that such a writing style reduced a capacity for satire.

6 !

An effective member of a debating team must focus clearly on the ------- issue and

d
avoid ------- arguments.
(A) equivocal. . obstreperous
(B) designated . . pertinent
(C) comprehensive . . general re 用

te
(D) principal . . peripheral

(E) subtle. . significant


is


ANSWERS AND EXPLANATIONS
eg

Explanation for Correct Answer D :


Choice (D) is correct. "Principal" means first in importance; in this context,
"peripheral," means of minor importance. If these terms were inserted into the
nR

text, the sentence would read "An effective member of a debating team must focus


clearly on the principal issue and avoid peripheral arguments." Since the structure
of the sentence indicates that the missing terms should be opposites, it makes
sense to say that debaters should focus on important issues and avoid less
important issues.


U

Explanation for Incorrect Answer A :


Choice (A) is incorrect. "Equivocal" means uncertain; "obstreperous" means unruly.
If these terms were inserted into the text, the sentence would read "An effective
member of a debating team must focus clearly on the equivocal issue and avoid
obstreperous arguments." Since the structure of the sentence indicates that the
missing terms should be opposites, it does not make sense to say that debaters
should focus on uncertain issues while avoiding unruly or noisy arguments.

Explanation for Incorrect Answer B :


Choice (B) is incorrect. "Designated" means chosen; "pertinent" means relevant. If
these terms were inserted into the text, the sentence would read "An effective
member of a debating team must focus clearly on the designated issue and avoid
pertinent arguments." Since the structure of the sentence indicates that the missing
terms should be opposites, it may make sense here to say that good debates would
focus on the "designated," or specified, issue, but it does not make sense to say
that good debates would avoid relevant arguments.

Explanation for Incorrect Answer C :


Choice (C) is incorrect. "Comprehensive" means inclusive; in this context,
"general," means applicable to the whole. If these terms were inserted into the
text, the sentence would read "An effective member of a debating team must focus
clearly on the comprehensive issue and avoid general arguments." Since the
structure of the sentence indicates that the missing terms should be opposites, it

file://E:\新建文件夹\a3.htm 2006-11-12
The Official SAT Online Course 页码,6/21

does not make sense to say that debaters should focus on the inclusive topic while
avoiding arguments that pertain to the whole. Rather than being opposites, these
terms are somewhat synonymous.

Explanation for Incorrect Answer E :


Choice (E) is incorrect. "Subtle" means refined or artful; "significant" means
meaningful. If one were to insert these terms into the text, the sentence would read
"An effective member of a debating team must focus clearly on the subtle issue and
avoid significant arguments." Since the structure of the sentence indicates that the
missing terms should be opposites, it does not make sense to say that debaters
should focus on artful, or crafty, issues while avoiding significant arguments.
Although the missing terms are somewhat opposite in meaning, they are presented
in the wrong order to work in the sentence.

7
The ------- with which merchants and landowners in early-nineteenth-century
Maryland and Virginia ------- Joshua Johnston’s professional services attests to his
artistic skill as a portrait painter.
(A) avidness . . sought
(B) diffidence . . purchased
(C) patience . . replaced


(D) elegance . . regarded

(E) zealousness . . overlooked


d
ANSWERS AND EXPLANATIONS
Explanation for Correct Answer A :
re 用
Choice (A) is correct. "Avid" means very eager; "to seek" means to go in search of.
te

If one were to insert these terms into the text, the sentence would read "The
avidness with which merchants and landowners in early-nineteenth-century
Maryland and Virginia sought Joshua Johnston's professional services attests to his


is
artistic skill as a portrait painter." Since the structure of the sentence indicates that
the first missing term modifies or describes the second missing term, it is logical to
say that people looking for a skilled portrait painter went in search of Johnston with


eg

great eagerness.


nR

Explanation for Incorrect Answer B :


Choice (B) is incorrect. "Diffidence" means the quality or state of being hesitant;
"purchased" means bought. If one were to insert these terms into the text, the
sentence would read "The diffidence with which merchants and landowners in early-


U

nineteenth-century Maryland and Virginia purchased Joshua Johnston's professional


services attests to his artistic skill as a portrait painter." This does not make sense,
since hesitance on the part of patrons to buy Johnston's work would not be proof of
his skill.

Explanation for Incorrect Answer C :


Choice (C) is incorrect. "Patience" is a willingness to wait; "to replace" means to put
something new in the place of. If one were to insert these terms into the text, the
sentence would read "The patience with which merchants and landowners in early-
nineteenth-century Maryland and Virginia replaced Joshua Johnston's professional
services attests to his artistic skill as a portrait painter." This does not make sense,
since a willingness to put off or forestall replacing Johnston's work does not affirm
his skill.

Explanation for Incorrect Answer D :


Choice (D) is incorrect. "Elegant" means refined or graceful; "regarded" means
observed. If one were to insert these terms into the text, the sentence would read
"The elegance with which merchants and landowners in early-nineteenth-century
Maryland and Virginia regarded Joshua Johnston's professional services attests to
his artistic skills as a portrait painter." Patrons regarding, or looking at, Johnston's
services with a graceful attitude would make little sense and would also not pertain
to Johnston's skill as a painter.

Explanation for Incorrect Answer E :


Choice (E) is incorrect. "Zealous" means emotionally intense; "overlooked" means
failed to notice. If one were to insert these terms into the text, the sentence would
read "The zealousness with which merchants and landowners in early-nineteenth-
century Maryland and Virginia overlooked Joshua Johnston's professional services

file://E:\新建文件夹\a3.htm 2006-11-12
The Official SAT Online Course 页码,7/21

attests to his artistic skill as a portrait painter." It makes no sense to say that
patrons exercised great intensity in failing to notice Johnston's services.

8 The man’s colleagues characterized him as ------- because he had an irritable,


quarrelsome disposition.
(A) tyrannical
(B) disingenuous
(C) sanctimonious

(D) cantankerous

(E) morose

ANSWERS AND EXPLANATIONS


Explanation for Correct Answer D :
Choice (D) is correct. "Cantankerous" means ill-tempered and quarrelsome, or
argumentative. If one were to insert this term into the text, the sentence would
read "The man's colleagues characterized him as cantankerous because he had an
irritable, quarrelsome disposition." A man with "an irritable, quarrelsome


disposition" could correctly be characterized as cantankerous.


ed
Explanation for Incorrect Answer A :


Choice (A) is incorrect. "Tyrannical" means harsh in the use of power. If one were
to insert this term into the text, the sentence would read "The man's colleagues
r
characterized him as tyrannical because he had an irritable, quarrelsome


te
disposition." While a "tyrant," or harsh authority figure, might very well have an
irritable disposition, not all people with such a disposition possess enough power to
be tyrannical.


is
Explanation for Incorrect Answer B :
Choice (B) is incorrect. "Disingenuous" means insincere or calculating. If one were


eg

to insert this term into the text, the sentence would read "The man's colleagues
characterized him as disingenuous because he had an irritable, quarrelsome
disposition." A person described as "disingenuous" by others would not likely have a


quarrelsome disposition, but would more probably hide his dishonesty behind a
superficially pleasing manner.
nR


Explanation for Incorrect Answer C :
Choice (C) is incorrect. To be "sanctimonious" means to affect, or feign, piety or
devoutness. If one were to insert this term into the text, the sentence would read


"The man's colleagues characterized him as sanctimonious because he had an
U

irritable, quarrelsome disposition." A person described as "sanctimonious" by others


would almost certainly not have a quarrelsome disposition, but would instead
maintain a superficially pious or virtuous manner.

Explanation for Incorrect Answer E :


Choice (E) is incorrect. To be "morose" means to have a gloomy disposition. If one
were to insert this term into the text, the sentence would read "The man's
colleagues characterized him as morose because he had an irritable, quarrelsome
disposition." A person described as "morose" by others would more likely have a
disposition that was more superficially sullen, or somber, than quarrelsome and
irritable.

Passage 1 is by Dorothy Sayers; Passage 2 is adapted from a work by Raymond


Chandler.

Passage 1
The detective story does not and cannot
attain the
loftiest level of literary achievement. Though it
deals

file://E:\新建文件夹\a3.htm 2006-11-12
The Official SAT Online Course 页码,8/21

with the most desperate effects of rage,


jealousy, and
revenge, it rarely touches the heights and
Line
depths of
human passion. It presents us with an
5
accomplished
fact, and looks upon death with a dispassionate
eye. It
does not show us the inner workings of the
murderer’s
mind—it must not, for the identity of the
criminal is
hidden until the end of the book. The most
successful
writers are those who contrive to keep the
10
story running
from beginning to end upon the same
emotional level,
and it is better to err in the direction of too !
little feeling

d
than too much.
Passage 2
re 用

te
I think what was really gnawing at Dorothy
Sayers in


her critique of the detective story was the
is

15
realization that


eg

her kind of detective story was an arid formula


unable to


satisfy its own implications. If the story started
nR

to be about


real people, they soon had to do unreal things
to conform


to the artificial pattern required by the plot.
U

When they did


unreal things, they ceased to be real
20
themselves. Sayers’
own stories show that she was annoyed by this
triteness.
Yet she would not give her characters their
heads and let
them make their own mystery.
9

Which best describes the relationship between the two passages?

(A) Passage 1 explains the evolution of a genre, while Passage 2 challenges


the notion of a distinct genre.
(B) Passage 1 discusses the constraints of a genre, while Passage 2 contends
that many of these constraints are self-imposed.
(C) Passage 1 celebrates a genre, while Passage 2 points out its deficiencies.

(D) Passage 1 explains the popularity of a genre, while Passage 2 questions its
commercial success.
(E) Passage 1 compares a genre unfavorably to other types of writing, while
Passage 2 argues that the genre has unique features.

file://E:\新建文件夹\a3.htm 2006-11-12
The Official SAT Online Course 页码,9/21

ANSWERS AND EXPLANATIONS


Explanation for Correct Answer B :
Choice (B) is correct. In Passage 1 Sayers identifies the constraints felt by "the
most successful writers" of detective stories to "contrive to keep the story running
from beginning to end upon the same emotional level." Passage 2 argues that
Sayers seemed unable to free her own stories from that "arid formula."

Explanation for Incorrect Answer A :


Choice (A) is incorrect. Passage 1 describes certain aspects of writing in the
detective genre but does not address how the genre changed over time, while
Passage 2 further elaborates on those same aspects of the genre.

Explanation for Incorrect Answer C :


Choice (C) is incorrect. While Passage 2 does discuss certain deficiencies in the
approach of some writers who work in the genre, Passage 1 focuses on the
constraints inherent in writing in the detective genre, and so could not be said to be
celebrating it.

Explanation for Incorrect Answer D :


Choice (D) is incorrect. While Passage 1 does mention the approach taken by "the
most successful writers," Passage 2 confines its discussion to the artistic rather
than commercial successes of those writers.


d
Explanation for Incorrect Answer E :
Choice (E) is incorrect. While Passage 1 says that the detective genre "does not and

re 用
cannot attain the loftiest level of literary achievement," Passage 2 does not make
the argument that the genre has features that distinguish it from other fiction.
te
10 业

is
The author of Passage 2 would most likely respond to the statement in lines 4-5,
Passage 1 (“it rarely . . . passion”), by

(A) arguing that this approach limits the characters’ development


eg

(B) denying that most writers of detective stories rely on formulas


(C) agreeing that strong emotions are out of place in detective stories


nR

(D) conceding that great literature is seldom commercially successful


(E) concurring that readers are primarily interested in plot


U

ANSWERS AND EXPLANATIONS


Explanation for Correct Answer A :
Choice (A) is correct. The author of Passage 2 argues that authors like Sayers fail in
that they do not give their "characters their heads and let them make their own
mystery"— in other words, that these authors do not let their characters grow and
show the full range of human emotions.

Explanation for Incorrect Answer B :


Choice (B) is incorrect. The author of Passage 2 says that Sayers— and, by
implication, similar writers—does use formulas.

Explanation for Incorrect Answer C :


Choice (C) is incorrect. The author of Passage 2, rather than accepting the view
expressed in Passage 1 that strong emotions are inappropriate in detective fiction,
holds that such an approach is formulaic and that characters in detective stories
should "be real."

Explanation for Incorrect Answer D :


Choice (D) is incorrect. The author of Passage 2 is not concerned with the
commercial success or failure of detective stories, but with their artistic integrity.

Explanation for Incorrect Answer E :


Choice (E) is incorrect. Although the author of Passage 2 does mention plot, he

file://E:\新建文件夹\a3.htm 2006-11-12
The Official SAT Online Course 页码,10/21

does not discuss the attitude of readers toward plots in detective stories.

11 Which of the following characteristics of detective stories presented in Passage 1


would be LEAST likely to be attributed to the “pattern” mentioned in line 19,
Passage 2 ?
(A) “cannot attain the loftiest level of literary achievement” (lines 1-2)

(B) “deals with the most desperate effects of rage, jealousy, and revenge”
(lines 2-4)
(C) “presents us with an accomplished fact” (lines 5-6)

(D) “looks upon death with a dispassionate eye” (line 6)

(E) “does not show us the inner workings of the murderer’s mind” (lines 7-
8)

ANSWERS AND EXPLANATIONS


Explanation for Correct Answer B :
Choice (B) is correct. The intense emotions resulting from rage, jealousy, and
revenge are very unlikely to appear in detective stories written following the
"artificial pattern" mentioned in Passage 2.



d
Explanation for Incorrect Answer A :

re
Choice (A) is incorrect. It is unlikely that a story created following the "artificial


pattern," with its "unreal" characters, would be able to achieve the level of high
literary art.
te
Explanation for Incorrect Answer C :


Choice (C) is incorrect. Detective stories created according to the "artificial pattern"
would be more likely to focus on facts than on emotional responses.


is

Explanation for Incorrect Answer D :


Choice (D) is incorrect. It is very likely that detective stories created following the
eg


"pattern" would display a "dispassionate," or unemotional, attitude toward dramatic
events.


Explanation for Incorrect Answer E :
nR

Choice (E) is incorrect. The "artificial pattern," in order to avoid emotional ups and
downs in a story, would probably recommend avoiding insights into the "inner


workings of the murderer's mind."


U

12 Passage 1 suggests that Sayers would most likely respond to lines 17-20, Passage 2
(“If the story started . . . themselves”), by pointing out that

(A) great writers seldom explore the range of human emotions


(B) detective stories do not address the consequences of people’s emotions
(C) detective stories are driven by the plot, not by the characters

(D) readers of detective stories prefer unrealistic situations

(E) real people often act in ways that are unexpected

ANSWERS AND EXPLANATIONS


Explanation for Correct Answer C :
Choice (C) is correct. Sayers states that "the most successful writers" are those
who keep their stories at an even "emotional level" until the very end, and even
that "it is better to err in the direction of too little feeling than too much." These
lines suggest that plot should always take precedence over character development.

Explanation for Incorrect Answer A :


Choice (A) is incorrect. Sayers argues that unlike great literary fiction, detective

file://E:\新建文件夹\a3.htm 2006-11-12
The Official SAT Online Course 页码,11/21

fiction "rarely touches the heights and depths of human passion."

Explanation for Incorrect Answer B :


Choice (B) is incorrect. Sayers states that detective stories deal "with the most
desperate effects of rage, jealousy, and revenge."

Explanation for Incorrect Answer D :


Choice (D) is incorrect. Although Sayers might agree with the view that readers
prefer the formalized and unrealistic situations found in many detective stories, she
does not address this issue in the passage.

Explanation for Incorrect Answer E :


Choice (E) is incorrect. Although Sayers might agree with the notion that "real
people often act in ways that are unexpected," she argues in the passage that the
successful approach to writing detective stories necessarily involves the creation of
unreal and constrained characters.

This passage is adapted from a series in which a college professor dramatizes the
lectures of famous scientists from the past. Here he speaks as Louis Pasteur (1822-
1895). In this part of the lecture, Pasteur has just described his discovery of the effect
of heating certain microbes that infect bottled beverages (the process later named
pasteurization).

But these undesirable microbes! Where and


how did !

ed
*
they arise? By spontaneous generation, as
some believe?
When I began to ask these questions of myself
r 用

and of my
te
students and colleagues, my close friends said:
Line


“Oh, no,
is

do not waste your time on such worthless


5


philosophical
eg

problems. Many a scientist has floundered and


perished in


the quagmire of spontaneous generation.” I
nR

replied: “But


the origin of life is a profound problem.” With


few excep-
U

tions, past discourses on spontaneous


generation have been
metaphysical exercises conducted with great
10
passion, but
without adding to our scientific knowledge.
I could not set aside my burning desire to
bring a little
stone, God willing, to the frail edifice of our
knowledge of
the deep mysteries of life and death, where all
our intellects
have so lamentably failed. In defense of
15
nonapplied science
I have repeatedly told my students that without
theory,
practice is but routine. Only theory is able to
cause the
spirit of invention to arise and develop. It is

file://E:\新建文件夹\a3.htm 2006-11-12
The Official SAT Online Course 页码,12/21

important that
students should not share the opinion of those
who disdain
everything in science that has no immediate
20
application. In
science, chance favors only the mind that is
prepared.
I repeat: in science, chance favors only the
mind that
is prepared.
I first confirmed the experiments of the
Italian abbé,
Lazzaro Spallanzani, known also for his studies
25
in gastric
digestion. I made a nutritious broth, put it in a
flask such
as this [Pasteur holds up a large flask
containing a brown

solution], heated it to violent boiling, and then
sealed the

d
neck of the flask in a flame. My results agreed

30
with those
re 用
of Spallanzani: the broth remained pure. But if


the neck
te
be broken to admit air, the broth soon became


putrid. My
is

critics said that the heating made the air in the


flask unfit


eg

for spontaneous generation. Only when fresh


air is admitted


can life begin anew. I argued in vain—even
nR

before our
35

Academy of Sciences—that the putrefaction was
caused by


U

admission of bacteria. More convincing


experiments were
needed.
I opened flasks of sterilized broth in the
cellar of the
Paris observatory, where the air was still. Only
one flask
out of ten became putrid, whereas eleven flasks
40
out of
eleven opened in the courtyard quickly acquired
a rich
growth of bacteria. I journeyed to Mt.
Montanvert in the
Alps, where I opened twenty flasks of sterilized
broth. Only
one became putrid. I concluded that the air in
the cellar and
the air above the glacier were freer of bacteria

file://E:\新建文件夹\a3.htm 2006-11-12
The Official SAT Online Course 页码,13/21

45 than the air


in the city streets. But my adversaries
performed similar
experiments with different results. Perhaps they
were not
careful to follow my procedures. The neck of
the flask must
be heated first to kill the bacteria on the glass;
then a heated
instrument must be used to break the tip of the
50
flask as it is
held high above the head. Immediately
thereafter the flask
must be sealed again in a flame [Pasteur
demonstrates the
procedure]. In these difficult researches, while I
sternly


object to frivolous contradictions, I feel nothing
but grat-
itude toward those who warn me if I should be

ed
55
in error.


I then devised a conclusive experiment. I
boiled a nutri-
er

tious infusion in a flask with a long curved neck
like this
t

one. The tip of the neck was not sealed but left
is

open to the
outside air. Thus, there was no hindrance to the


eg

entrance of
fresh air with its “vital force” as claimed by
60
the advocates

nR

of spontaneous generation. But bacteria in the


entering air
would be trapped by the walls of the long glass


U

tube. The
fluid remained sterile so long as the flask was
maintained
in the vertical position. If, however, I
contaminated the
broth by allowing some of it to flow into the
65
neck and then
back into the flask, putrefaction promptly
followed. So we
see that life does not arise spontaneously. Life
comes only
from life.

* The supposed origination of living matter directly from


lifeless matter

13 The focus of the lecture is on how Pasteur

file://E:\新建文件夹\a3.htm 2006-11-12
The Official SAT Online Course 页码,14/21

(A) disproved an erroneous theory


(B) documented and published his experiments
(C) developed a process for killing microbes

(D) applied his findings on spontaneous generation to new problems

(E) contributed to the improvement of laboratory research standards

ANSWERS AND EXPLANATIONS


Explanation for Correct Answer A :
Choice (A) is correct. The lecture details how Pasteur’s experiments "disproved,"
or proved wrong, the "erroneous," or incorrect, theory of spontaneous generation.

Explanation for Incorrect Answer B :


Choice (B) is incorrect. The lecture does not discuss the documentation or
publication of Pasteur’s experiments, but it does discuss what they proved.

Explanation for Incorrect Answer C :


Choice (C) is incorrect. Pasteur’s experiments were concerned not with killing
"microbes," or tiny organisms, but with discovering how and why they come to
grow in previously sterile, or clean, liquids.


ed
Explanation for Incorrect Answer D :
Choice (D) is incorrect. Although Pasteur may have later applied his findings to new


problems, this passage is concerned only with how Pasteur proved the theory of
spontaneous generation to be inaccurate.

r

te
Explanation for Incorrect Answer E :
Choice (E) is incorrect. The lecture is concerned not with the improvement of
laboratory standards but with Pasteur’s study of the growth of bacteria.


is


eg

14
In the lecture, Pasteur concludes that the answer to the question “Where and how
did they arise?” (lines 1-2) is

(A)


spontaneously
nR

(B) from airborne bacteria


(C) from impurities in the original broth

(D) from the curved neck of a flask


U

(E) from a broken flask

ANSWERS AND EXPLANATIONS


Explanation for Correct Answer B :
Choice (B) is correct. At the end of the lecture, Pasteur declares that “[l]ife comes
only from life,” or that microbes that are observed in liquid broth have migrated
there from the air.

Explanation for Incorrect Answer A :


Choice (A) is incorrect. In the lecture, Pasteur concludes just the opposite: bacteria
do not arise "spontaneously," or suddenly, without outside influence.

Explanation for Incorrect Answer C :


Choice (C) is incorrect. Pasteur does not conclude that "impurities," or unclean
elements, in the original broth caused the growth of bacteria. Rather, his
experiment shows that existing bacteria in the air were the source of bacteria in the
broth.

Explanation for Incorrect Answer D :


Choice (D) is incorrect. The curved neck of the flask helped Pasteur better conduct
his experiment but was itself not responsible for the growth of bacteria in the broth.

file://E:\新建文件夹\a3.htm 2006-11-12
The Official SAT Online Course 页码,15/21

Explanation for Incorrect Answer E :


Choice (E) is incorrect. The broken flask helped Pasteur to control the way that air
and bacteria passed into the broth, but it does not explain where and how the
bacteria grew.

15 In the first two paragraphs (lines 1-23), Pasteur is primarily concerned with
(A) summarizing the results of his experiments about spontaneous generation
(B) criticizing those who have taken the passion out of science
(C) establishing his motivation for studying the origin of microbes

(D) attacking critics of his experiments

(E) correcting the impression that he is concerned only with experiments that
have immediate application

ANSWERS AND EXPLANATIONS


Explanation for Correct Answer C :
Choice (C) is correct. Pasteur uses the first two paragraphs of his lecture to explain
his "motivation," or reason, for studying the microbes—that he wanted to


contribute to the scientific knowledge of the origins of life.


d
re
Explanation for Incorrect Answer A :


Choice (A) is incorrect. The first two paragraphs do not explain the results of
Pasteur’s experiments. Rather, they explain why he undertook the study.


te
Explanation for Incorrect Answer B :
Choice (B) is incorrect. The opposite is true: Pasteur criticizes those who have
passion but have failed to increase scientific knowledge.


is

Explanation for Incorrect Answer D :


Choice (D) is incorrect. Although Pasteur mentions that he was faced with criticism,
eg

nowhere in these paragraphs does he attack his detractors.


Explanation for Incorrect Answer E :
Choice (E) is incorrect. The purpose of these paragraphs is not to counter a false
nR

impression concerning Pasteur’s interest in applied or nonapplied science but to


explain why he chose to study the growth of bacteria.



U

16
The word "quagmire" (line 7) is used primarily to emphasize the

(A) state of scientific ignorance in the 1800’s


(B) futility of a particular line of research
(C) moral dilemma faced by scientists like Pasteur

(D) failure of some to distinguish between pure and applied science

(E) tendency of unsuccessful scientists to look for simple solutions

ANSWERS AND EXPLANATIONS


Explanation for Correct Answer B :
Choice (B) is correct. It makes sense to describe spontaneous generation, a
frustrating area of research, as a "quagmire," or complicated situation from which it
is difficult to escape.

Explanation for Incorrect Answer A :


Choice (A) is incorrect. In this context, "quagmire" refers to the problem of
spontaneous generation, not to the general state of scientific ignorance, or lack of
knowledge.

file://E:\新建文件夹\a3.htm 2006-11-12
The Official SAT Online Course 页码,16/21

Explanation for Incorrect Answer C :


Choice (C) is incorrect. "Morality," or standards of right and wrong, is unrelated to
Pasteur’s concerns respecting spontaneous generation

Explanation for Incorrect Answer D :


Choice (D) is incorrect. Whether or not some people differentiate between "pure," or
theoretical, and "applied," or practical, science is unrelated to the difficulty of the
problem of spontaneous generation itself.

Explanation for Incorrect Answer E :


Choice (E) is incorrect. Pasteur does not criticize proposed solutions to the problem
of spontaneous generation on grounds of simplicity; rather, he says they were
"metaphysical" and not truly scientific.

17 Pasteur characterizes “past discourses on spontaneous generation” (line 9) as


having
(A) demonstrated the futility of practical scientific studies
(B) failed because of incomplete knowledge about sterilization of apparatus
(C) enabled him to understand inconsistencies in his early experiments

(D) failed to increase scientific knowledge

(E)


resolved much of the controversy surrounding the issue


d
ANSWERS AND EXPLANATIONS
Explanation for Correct Answer D :

re 用
Choice (D) is correct. Pasteur acknowledges that past experiments had been
conducted with passion but argues that they were not tested in the laboratory and
te

therefore did not increase the body of scientific knowledge on the subject.


is

Explanation for Incorrect Answer A :


eg

Choice (A) is incorrect. Pasteur does not describe past research as demonstrating
the "futility," or pointlessness, of practical studies. On the contrary, he criticizes
past studies for failing to be practical or scientific.


nR

Explanation for Incorrect Answer B :


Choice (B) is incorrect. "Nowhere" in his discussion of past research on spontaneous


generation does Pasteur mention the sterilization of any laboratory apparatus, or
tool.


U

Explanation for Incorrect Answer C :


Choice (C) is incorrect. Pasteur does not discuss his own early experiments in the
context of past research on spontaneous generation.

Explanation for Incorrect Answer E :


Choice (E) is incorrect. Pasteur claims that past experiments failed to increase
scientific knowledge. Therefore, it does not make sense to say that they "resolved,"
or settled, the "controversy," or debate, around the question of spontaneous
generation.

18
The “little stone” (lines 12-13) refers to the

(A) slight addition that Pasteur hoped to make to the existing body of facts
(B) small effect that Pasteur wanted to have on one person’s learning
(C) minor disappointment Pasteur felt at being rebuffed by his colleagues

(D) narrow-mindedness of those who cling to scientific fallacies

(E) imperceptible progress that Pasteur had made in understanding


spontaneous generation

ANSWERS AND EXPLANATIONS

file://E:\新建文件夹\a3.htm 2006-11-12
The Official SAT Online Course 页码,17/21

Explanation for Correct Answer A :


Choice (A) is correct. Pasteur refers to his desire to bring a “little stone” to the
“frail edifice,” or unsteady structure, of the human knowledge of life and death.
In other words, the goal of his research was to make a contribution to the existing
body of scientific fact, thereby increasing human knowledge.

Explanation for Incorrect Answer B :


Choice (B) is incorrect. Pasteur does not want to have a small effect on one person;
he wants to have an effect on scientific knowledge in general.

Explanation for Incorrect Answer C :


Choice (C) is incorrect. The “little stone” Pasteur is speaking of is his own
potential contribution to science, not the "rebuff," or criticism, he received at the
hands of his fellow scientists.

Explanation for Incorrect Answer D :


Choice (D) is incorrect. It does not make sense to say that Pasteur desired to
contribute to the narrow-mindedness of others.

Explanation for Incorrect Answer E :


Choice (E) is incorrect. The “little stone” refers to the contribution Pasteur
desired to make when he began his research, not the results of his experiments.



ed
19 Pasteur’s pronouncement about preparation and chance in lines 20-23 implies that

(A)


only projects that have an immediate application are important
(B) practice improves a scientist’s chances of making a significant discovery
r
te

(C) few scientists are lucky enough to devise useful theories

(D) work on projects that have no immediate application prepares scientists to


exploit chance discoveries


is
(E) most scientific discoveries that have no immediate application are the
result of good luck and timing


eg

ANSWERS AND EXPLANATIONS


Explanation for Correct Answer D :
nR

Choice (D) is correct. In the lecture, Pasteur encourages his students to pursue
projects whose applications are not yet known so that they will be better able to


recognize and "exploit," or take advantage of, chance discoveries in the laboratory.


U

Explanation for Incorrect Answer A :


Choice (A) is incorrect. The opposite is true: Pasteur believes that pursuing projects
without immediate applications prepares a scientist for future discoveries and is as
important as the practical work itself.

Explanation for Incorrect Answer B :


Choice (B) is incorrect. According to the lecture, it is not practice but a solid
grounding in theory that prepares a scientist to make "significant," or important,
discoveries.

Explanation for Incorrect Answer C :


Choice (C) is incorrect. The statement is concerned not with the number of
scientists who create useful theories but with the best way to prepare oneself for
scientific discovery.

Explanation for Incorrect Answer E :


Choice (E) is incorrect. Pasteur does not claim that good luck and timing result in
discoveries of any kind. Rather, he argues that pursuing research whose
applications are unknown prepares a scientist to better understand and make use of
later discoveries.

20 In context, the reference to the Academy of Sciences (line 35) serves to suggest why

file://E:\新建文件夹\a3.htm 2006-11-12
The Official SAT Online Course 页码,18/21

(A) Pasteur was so determined to make a significant contribution to scientific


knowledge
(B) Pasteur felt compelled to replicate Spallanzani’s experiments

(C) spontaneous generation had already begun to be discredited when Pasteur


began his experimentation
(D) Pasteur believed he needed to design experiments that were more
persuasive
(E) spontaneous generation was viewed by Pasteur’s colleagues as a topic
that was unfit for scientific study

ANSWERS AND EXPLANATIONS


Explanation for Correct Answer D :
Choice (D) is correct. The passage indicates that after failing to convince the
academy of his findings, Pasteur felt it was necessary to design more "persuasive,"
or convincing, experiments.

Explanation for Incorrect Answer A :


Choice (A) is incorrect. The fact that Pasteur argued unsuccessfully for his research


before the academy may have motivated him to prove his critics wrong, but it does
not explain his initial desire to contribute to scientific knowledge.


d
Explanation for Incorrect Answer B :
Choice (B) is incorrect. Pasteur "replicated," or repeated, Spallanzani’s
experiments before he presented his findings to the academy.

Explanation for Incorrect Answer C :


re 用
Choice (C) is incorrect. The reference to the academy illustrates how difficult it was


te
for Pasteur’s experiments to be accepted by the scientific community, not that his
research joined with that of others who were also disproving spontaneous
generation.


is
Explanation for Incorrect Answer E :
Choice (E) is incorrect. The incident at the Academy of Sciences does not suggest


eg

that Pasteur’s colleagues thought spontaneous generation was an inappropriate


research project. Rather, it shows that they were not convinced by his findings.


nR

21


In line 41, "rich" most nearly means

(A) precious


(B) vital
U

(C) abundant

(D) meaningful

(E) productive

ANSWERS AND EXPLANATIONS


Explanation for Correct Answer C :
Choice (C) is correct. In this context, which speaks of "a rich growth of bacteria,"
"rich" means abundant, or existing in large quantities.

Explanation for Incorrect Answer A :


Choice (A) is incorrect. It does not make sense to describe the growth of bacteria
as "precious," or rare.

Explanation for Incorrect Answer B :


Choice (B) is incorrect. "Vital," or indispensable for survival, does not describe the
growth of bacteria in Pasteur’s flasks.

Explanation for Incorrect Answer D :


Choice (D) is incorrect. Although the presence of bacteria was "meaningful," or

file://E:\新建文件夹\a3.htm 2006-11-12
The Official SAT Online Course 页码,19/21

significant, for Pasteur’s research, it does not make sense to describe the growth
itself as such.

Explanation for Incorrect Answer E :


Choice (E) is incorrect. In this context, "rich" is used to describe the quantity of
bacteria present. It does not make sense to define that amount as "productive," or
able to create something.

22 The "conclusive experiment" (line 56) performed by Pasteur was designed to answer
critics who argued that

(A) the apparatus used in Pasteur's earlier experiments had not been
adequately sterilized
(B) Pasteur's experiments related to spontaneous generation had no
immediate application
(C) the results of Pasteur's experiments in the Alps and in the cellar could not
be replicated
(D) the broth in the flasks of Pasteur's earlier experiments was not nutritious
enough
(E) heating made the air in the flasks of the earlier experiments unfit for
spontaneous generation

ANSWERS AND EXPLANATIONS




Explanation for Correct Answer E :

d
Choice (E) is correct. Pasteur's critics claimed that by using heat to seal the neck of
the bottle in earlier experiments, Pasteur had made the air unsuitable for

re
spontaneous generation to take place. In his last experiment, he answered this


opposition by using a special flask that permitted fresh air, but not bacteria, to
come into contact with the liquid.


e
st


Explanation for Incorrect Answer A :
i

Choice (A) is incorrect. Pasteur's critics argued that sterilizing, or purifying, the


eg

flasks had made the air unfit for spontaneous generation. In other words, there had
been too much sterilization, not an inadequate amount.


Explanation for Incorrect Answer B :
Choice (B) is incorrect. The last experiment was designed to answer critics who
nR

maintained confidence in the theory of spontaneous generation, not those who


argued against nonapplied science.

Explanation for Incorrect Answer C :


Choice (C) is incorrect. The "conclusive experiment" was not designed to address
U

inconsistencies between Pasteur's and his colleagues' data, which the passage
attributes to carelessness. It was designed to satisfy those who argued that the
liquid must come into contact with fresh air in order for spontaneous generation to
occur.

Explanation for Incorrect Answer D :


Choice (D) is incorrect. The nutritional content of the various broths is unrelated to
the results or design of the experiments.

23
In the context of the passage as a whole, the “vital force” (line 60) is best
described as

(A) what Pasteur called the basic unit of life


(B) a term that was outdated in Pasteur’s time
(C) nutrients necessary for sustaining life

(D) that which has the power to destroy life

(E) what opponents of Pasteur believed to be a source of life

ANSWERS AND EXPLANATIONS

file://E:\新建文件夹\a3.htm 2006-11-12
The Official SAT Online Course 页码,20/21

Explanation for Correct Answer E :


Choice (E) is correct. In the context of the passage, it makes sense to say that the
“vital force” is the element that Pasteur’s opponents believed to be responsible
for the spontaneous generation of life.

Explanation for Incorrect Answer A :


Choice (A) is incorrect. The “vital force” is not what Pasteur called the basic unit
of life; that was his fellow scientists’ word for an energy or substance that caused
living matter to appear.

Explanation for Incorrect Answer B :


Choice (B) is incorrect. Nothing in the passage indicates that this term was no
longer in use in Pasteur’s time. On the contrary, the fact that it was used by his
colleagues indicates that it was a popular notion.

Explanation for Incorrect Answer C :


Choice (C) is incorrect. The “vital force” is what was thought to cause life, not the
nutrients, or nourishing material.

Explanation for Incorrect Answer D :


Choice (D) is incorrect. The “vital force” was not seen as something that detroys
life but as something that brings life into being.



d
24 In his conclusive experiment, Pasteur kept the flasks vertical (line 64) in order to

re
(A) prevent fresh air from entering them
(B)

retain the boiling liquid inside the flasks


te
(C) prevent the fluid from touching trapped bacteria

(D) avoid disturbing the solution inside


is
(E) replicate his previous experiments exactly


eg

ANSWERS AND EXPLANATIONS


Explanation for Correct Answer C :


Choice (C) is correct. When kept in the vertical position, the design of the flasks
nR

prevented the fluid in the bowl from touching bacteria trapped in the curved neck.



U

Explanation for Incorrect Answer A :


Choice (A) is incorrect. The opposite of this statement is true. The passage states
that in the vertical position, there was no "hindrance," or obstacle, to the entrance
of fresh air.

Explanation for Incorrect Answer B :


Choice (B) is incorrect. Pasteur did not position the flasks vertically to "retain," or
keep, the liquid inside; he did it to control the flow of air and bacteria.

Explanation for Incorrect Answer D :


Choice (D) is incorrect. Pasteur kept the flasks in the vertical position not to avoid
disturbing the liquid but to trap the bacteria and prevent it from coming into
contact with the liquid.

Explanation for Incorrect Answer E :


Choice (E) is incorrect. Pasteur revised his method and used the curved flask not to
"replicate," or repeat, his previous experiments but to improve upon them.

Back to Score Report

Copyright © 2006 The College Board. All rights reserved. Privacy Policy Terms of Use Contact Us

file://E:\新建文件夹\a3.htm 2006-11-12
The Official SAT Online Course 页码,21/21



r

ed

te

is


eg


nR



U

file://E:\新建文件夹\a3.htm 2006-11-12
The Official SAT Online Course 页码,1/15

Help | Profile | My Organizer | My Bookmarks | Logout

Answers and Explanations

Test Sections Back to Score Report

Section 1 View Answers and Explanations


Section 2 Online - Practice Test #1

Section 3
1 Which of the following represents the total cost, in dollars, of compact discs at
Section 5
each and compact disc cases at each? (Disregard sales tax.)
Section 6
Section 7 (A)

Section 8 (B)
Section 9
(C)
Section 10
(D)

(E)

ANSWERS AND EXPLANATIONS !



d
Explanation for Correct Answer A :
Choice (A) is correct. The cost of the compact discs (CDs) purchased at a
price of
a price of
each is
each is
re 用
dollars. The cost of the CD cases purchased at
dollars. Therefore, the total cost, in dollars, is


te

is

Explanation for Incorrect Answer B :


eg

Choice (B) is not correct. There were CDs purchased (not ), and CD
cases purchased (not ).


Explanation for Incorrect Answer C :
nR

Choice (C) is not correct. This equation corresponds to the situation where all
the items purchased had a price of not the situation where some had a


price of and others had a price of .


Explanation for Incorrect Answer D :
U

Choice (D) is not correct. See the explanation for the correct response (A).

Explanation for Incorrect Answer E :


Choice (E) is not correct. See the explanation for the correct response (A).

If the areas of the two rectangles in the figure above are equal, which of the following
could be the coordinates of point ?

(A)

file://E:\新建文件夹\a4.htm 2006-11-12
The Official SAT Online Course 页码,2/15

(B)

(C)

(D)

(E)

ANSWERS AND EXPLANATIONS


Explanation for Correct Answer C :
Choice (C) is correct. Since is in the second quadrant, it must have a
negative -coordinate and a positive -coordinate. If has coordinates
then the area of the rectangle in the second quadrant is For
the two rectangles to be of equal area, must equal The only
answer choice that fits both of these criteria is

Explanation for Incorrect Answer A :


Choice (A) is not correct. is in the second quadrant, and is a


point in the third quadrant.

Explanation for Incorrect Answer B :


d
Choice (B) is not correct. If has coordinates then the rectangle in
the second quadrant has only half the area of the rectangle in the first

re 用
quadrant, and the problem requires that they have the same area.

Explanation for Incorrect Answer D :


te

Choice (D) is not correct. is in the second quadrant, and is a
point in the fourth quadrant.


is
Explanation for Incorrect Answer E :
Choice (E) is not correct. is in the second quadrant, and is a point


eg

in the first quadrant.


nR

3 A box contains solid-colored marbles that are either orange, blue, or green. If


percent of the marbles are orange and percent of the marbles are blue, what
percent are green?

(A)


U

(B)

(C)

(D)

(E)

ANSWERS AND EXPLANATIONS


Explanation for Correct Answer B :
Choice (B) is correct. Since percent of the marbles are orange,
percent are blue, and the remaining marbles are green, it follows that
percent of the marbles are green.

Explanation for Incorrect Answer A :


Choice (A) is not correct. This is the percentage of the marbles that are blue
(and also the percentage of the marbles that are orange).

Explanation for Incorrect Answer C :


Choice (C) is not correct. See the explanation for the correct response (B).

Explanation for Incorrect Answer D :

file://E:\新建文件夹\a4.htm 2006-11-12
The Official SAT Online Course 页码,3/15

Choice (D) is not correct. This is the percentage of the marbles that are not
green.

Explanation for Incorrect Answer E :


Choice (E) is not correct. This is the percentage of the marbles that are not
orange (and also the percentage that are not blue).

Sets and are shown above. If is a member of set and is a member of


set , which of the following CANNOT be equal to the product

(A)

(B)

(C)

(D)

(E)



d
ANSWERS AND EXPLANATIONS

e 用
Explanation for Correct Answer A :
er
Choice (A) is correct. The only ways to write as the product of two
positive integers (ignoring the order of the factors) are or
While and


are both members of neither nor is a
t
member of Also, while is a member of is not a member of


is
Therefore, cannot be written as the product of two integers such that one
is a member of and the other is a member of
eg



Explanation for Incorrect Answer B :
nR

Choice (B) is not correct. The number can be written as where


is a member of and is a member of The question asks for a number
that cannot be written as the product of a member of with a member of


U

Explanation for Incorrect Answer C :


Choice (C) is not correct. The number can be written as where
is a member of and is a member of The question asks for a number
that cannot be written as the product of a member of with a member of

Explanation for Incorrect Answer D :


Choice (D) is not correct. The number can be written as where
is a member of and is a member of The question asks for a number
that cannot be written as the product of a member of with a member of

Explanation for Incorrect Answer E :


Choice (E) is not correct. The number can be written as where
is a member of and is a member of The question asks for a number
that cannot be written as the product of a member of with a member of

5
If then could be which of the following?

file://E:\新建文件夹\a4.htm 2006-11-12
The Official SAT Online Course 页码,4/15

(A)

(B)

(C)

(D)

(E)

ANSWERS AND EXPLANATIONS


Explanation for Correct Answer E :

Choice (E) is correct. If then Of the choices,

the only value of that satisfies this condition is

Explanation for Incorrect Answer A :

Choice (A) is not correct. If then However,



d
Explanation for Incorrect Answer B :

Choice (B) is not correct. If then However,

e 用
er
Explanation for Incorrect Answer C :

t

is
Choice (C) is not correct. If then However,


eg

Explanation for Incorrect Answer D :


Choice (D) is not correct. If then However,
nR



U

If which of the following must be true?

(A)

(B)

(C)

(D)

(E)

ANSWERS AND EXPLANATIONS


Explanation for Correct Answer B :
Choice (B) is correct. The right side of the equation can be expanded:
Then can be subtracted from both sides, leaving

Explanation for Incorrect Answer A :


Choice (A) is not correct. See the explanation for the correct response (B).

file://E:\新建文件夹\a4.htm 2006-11-12
The Official SAT Online Course 页码,5/15

Explanation for Incorrect Answer C :


Choice (C) is not correct. If were substituted for then the equation
would be which simplifies to So could be equal to
but the equation is only true when is

Explanation for Incorrect Answer D :


Choice (D) is not correct. If were substituted for then the equation
would be which simplifies to So could be equal to
but the equation is only true when is

Explanation for Incorrect Answer E :


Choice (E) is not correct. If were substituted for then the equation
would be which simplifies to So could be equal
to but the equation is only true when is

Questions 7-9 refer to the following definition.

Let be defined by for all numbers and


7

(A)


(B)

(C)


ed
(D)

(E)

ANSWERS AND EXPLANATIONS r 用


te

Explanation for Correct Answer C :
Choice (C) is correct.


is
eg


Explanation for Incorrect Answer A :


Choice (A) is not correct. but is not equal to
nR

Explanation for Incorrect Answer B :


Choice (B) is not correct. but is not equal to


U

Explanation for Incorrect Answer D :


Choice (D) is not correct. but

Explanation for Incorrect Answer E :


Choice (E) is not correct. but is not equal to

If then

(A)

(B)

(C)

(D)

(E)

ANSWERS AND EXPLANATIONS


Explanation for Correct Answer A :
Choice (A) is correct. Since this expression is equal
to it follows that Therefore, which

file://E:\新建文件夹\a4.htm 2006-11-12
The Official SAT Online Course 页码,6/15

gives so .

Explanation for Incorrect Answer B :


Choice (B) is not correct. If then but
the problem states that

Explanation for Incorrect Answer C :


Choice (C) is not correct. If then
but the problem states that

Explanation for Incorrect Answer D :


Choice (D) is not correct. If then
but the problem states that

Explanation for Incorrect Answer E :


Choice (E) is not correct. If then
but the problem states that

9 For what value of is the statement !


always true?


d
(A)

re
(B)

(C)

(D) 用
te
(E)



is

ANSWERS AND EXPLANATIONS


eg


Explanation for Correct Answer B :
Choice (B) is correct. Since
be rewritten as Subtracting
the equation
from both sides yields
can

用or Thus, either or Therefore, the


nR

only value of that always makes true is



U

Explanation for Incorrect Answer A :


Choice (A) is not correct. If then This
is only equal to or in this case when but the question asks for
a value of that will make the equation true for all values of

Explanation for Incorrect Answer C :


Choice (C) is not correct. If then This is only
equal to or in this case when but the question asks for a value
of that will make the equation true for all values of

Explanation for Incorrect Answer D :


Choice (D) is not correct. If then This is
only equal to or in this case when but the question asks for a
value of that will make the equation true for all values of

Explanation for Incorrect Answer E :


Choice (E) is not correct. If then This is
only equal to or in this case when but the question asks for a
value of that will make the equation true for all values of

10

file://E:\新建文件夹\a4.htm 2006-11-12
The Official SAT Online Course 页码,7/15

In the equation above, is a constant. If the roots of the equation are and
what is the value of

(A)

(B)

(C)

(D)

(E)

ANSWERS AND EXPLANATIONS


Explanation for Correct Answer A :
Choice (A) is correct. Recall that a root of an equation is a number that,
when substituted for the variable in the equation, reduces the equation to an
identity. If and are the roots of the equation then
and Since the left-hand sides are
each equal to zero, it follows that Since is a constant, this is true
for all values of not merely and



d
Explanation for Incorrect Answer B :

re 用
Choice (B) is not correct. See the explanation for the correct response (A).

Explanation for Incorrect Answer C :


te
Choice (C) is not correct. See the explanation for the correct response (A).

Explanation for Incorrect Answer D :


is
Choice (D) is not correct. See the explanation for the correct response (A).

Explanation for Incorrect Answer E :


eg

Choice (E) is not correct. See the explanation for the correct response (A).


nR

11
Which of the following represents the area, of a circle as a function of its


diameter,

(A)


U

(B)

(C)

(D)

(E)

ANSWERS AND EXPLANATIONS


Explanation for Correct Answer E :
Choice (E) is correct. The area of a circle is given by the formula
where is the radius of the circle. The diameter of a circle, is twice the

radius, so Substituting for gives the equation

file://E:\新建文件夹\a4.htm 2006-11-12
The Official SAT Online Course 页码,8/15

Explanation for Incorrect Answer A :


Choice (A) is not correct. If is the radius of the circle, then
This is the circumference of the circle.

Explanation for Incorrect Answer B :


Choice (B) is not correct. If is the radius of the circle, then
This is twice the circumference of the circle.

Explanation for Incorrect Answer C :


Choice (C) is not correct. If is the radius of the circle, then

This is four times the area of the circle.

Explanation for Incorrect Answer D :


Choice (D) is not correct. If is the radius of the circle, then

This is twice the area of the circle.

12



ed 用
er

st


i


eg

The table above shows some values for the functions and If and are linear


functions, what is the value of
nR


(A)

(B)


(C)
U

(D)

(E)

ANSWERS AND EXPLANATIONS


Explanation for Correct Answer C :
Choice (C) is correct. Since is a linear function, it can be written in the
form From two of the data points in the table, you can write
two equations: and Substituting for
yields the equation which simplifies to and

further to so Since it follows that

and Knowing the values of and you can

calculate Therefore,

Similarly, since is a linear function, it can be written in the form


From two of the data points in the table, you can write two
equations: and Substituting for yields the

file://E:\新建文件夹\a4.htm 2006-11-12
The Official SAT Online Course 页码,9/15

equation which simplifies to and further to

so Since it follows that and

Knowing the values of and you can calculate

Therefore,

Therefore,

Explanation for Incorrect Answer A :


Choice (A) is not correct. See the explanation for the correct response (C).

Explanation for Incorrect Answer B :


Choice (B) is not correct. See the explanation for the correct response (C).

Explanation for Incorrect Answer D :


Choice (D) is not correct. See the explanation for the correct response (C).

Explanation for Incorrect Answer E :


Choice (E) is not correct. See the explanation for the correct response (C).


d
13

re 用

te

is


eg


nR



U

The depth of a lake is the difference between the altitude at the surface and at the
lowest point of the lake. If the five lakes in the graph above were listed in order from
the greatest depth to the least depth, which lake would be third in the list?

(A) Erie
(B) Huron
(C) Michigan

(D) Ontario

(E) Superior

ANSWERS AND EXPLANATIONS


Explanation for Correct Answer D :
Choice (D) is correct. To determine the lake with the third greatest depth,
you must determine the depth of each lake by subtracting the lowest point
from the highest point. The depth of Lake Superior is
the depth of Michigan is the depth of Huron is
the depth of Erie is and the depth of
Ontario is Ontario is the lake with the third greatest
depth, at feet.

file://E:\新建文件夹\a4.htm 2006-11-12
The Official SAT Online Course 页码,10/15

Explanation for Incorrect Answer A :


Choice (A) is not correct. See the explanation for the correct response (D).

Explanation for Incorrect Answer B :


Choice (B) is not correct. See the explanation for the correct response (D).

Explanation for Incorrect Answer C :


Choice (C) is not correct. See the explanation for the correct response (D).

Explanation for Incorrect Answer E :


Choice (E) is not correct. See the explanation for the correct response (D).

14

In the figure above, and If



is an integer, what is the greatest


possible value of

ed 用
(A)

(B)
er

(C)

(D)
t

is
(E)


eg

ANSWERS AND EXPLANATIONS


Explanation for Correct Answer B :


Choice (B) is correct. Since the fact that implies
nR

or The equation can be rewritten as


Substituting for in the previous inequality yields


it follows that
which can be simplified to Since is an integer,
must also be an integer, and the greatest integer that


U

satisfies the inequality is

Explanation for Incorrect Answer A :


Choice (A) is not correct. Although is a possible value for there are
greater possible values of and the question asks for the greatest possible
value.

Explanation for Incorrect Answer C :


Choice (C) is not correct. If then and
However, the problem states that

Explanation for Incorrect Answer D :


Choice (D) is not correct. If then and
However, the problem states that

Explanation for Incorrect Answer E :


Choice (E) is not correct. If then and
However, the problem states that

15
Molly is inches tall. At 10:00 A.M. one day, her shadow is inches long, and

file://E:\新建文件夹\a4.htm 2006-11-12
The Official SAT Online Course 页码,11/15

the shadow of a nearby tree is inches long. In terms of what is the height, in
inches, of the tree?
(A)

(B)

(C)

(D) 4s

(E)

ANSWERS AND EXPLANATIONS


Explanation for Correct Answer D :
Choice (D) is correct. The ratio of Molly’s height ( inches) to the length
of her shadow ( inches) is proportional to the ratio of the tree’s height (

) to the length of its shadow ( ). So This simplifies to

Explanation for Incorrect Answer A : !



d
Choice (A) is not correct. Height is proportional to the length of the shadow;
it is not the length of the shadow plus a constant.

re
Explanation for Incorrect Answer B :


Choice (B) is not correct. See the explanation for the correct response (D).


te
Explanation for Incorrect Answer C :
Choice (C) is not correct. The height of the tree is times the length of its


is
shadow, not the length of its shadow.


eg

Explanation for Incorrect Answer E :


Choice (E) is not correct. See the explanation for the correct response (D).


nR

16


If a number is chosen at random from the set what is the
probability
that it is a member of the solution set of both and


U

(A)

(B)

(C)

(D)

(E)

ANSWERS AND EXPLANATIONS


Explanation for Correct Answer C :
Choice (C) is correct. To determine this probability, you must first look at the
solution sets of the inequalities. The inequality simplifies to
and simplifies to Of the five numbers in the set, only
and are greater than and less than Therefore, the probability
of picking a number from the set that is in the solution set of both these

inequalities is

file://E:\新建文件夹\a4.htm 2006-11-12
The Official SAT Online Course 页码,12/15

Explanation for Incorrect Answer A :


Choice (A) is not correct. See the explanation for the correct response (C).

Explanation for Incorrect Answer B :


Choice (B) is not correct. See the explanation for the correct response (C).

Explanation for Incorrect Answer D :


Choice (D) is not correct. See the explanation for the correct response (C).

Explanation for Incorrect Answer E :


Choice (E) is not correct. See the explanation for the correct response (C).

17 If the length of is and the length of is which of the following could be


the length of

(A)

(B)

(C)


(D)

(E)


d
ANSWERS AND EXPLANATIONS

e 用
Explanation for Correct Answer A :
Choice (A) is correct. If and
er
lie on a line, the greatest possible
length of is If

业 and do not lie on the same line,


t
then a triangle is formed by these three points, and the length of must


is
be less than the sum of the lengths of and which is The only
choice that is less than or equal to is


eg


Explanation for Incorrect Answer B :
nR

Choice (B) is not correct. The greatest possible length of is equal to


Explanation for Incorrect Answer C :


U

Choice (C) is not correct. The greatest possible length of is equal to

Explanation for Incorrect Answer D :


Choice (D) is not correct. The greatest possible length of is equal to

Explanation for Incorrect Answer E :


Choice (E) is not correct. The greatest possible length of is equal to

18

file://E:\新建文件夹\a4.htm 2006-11-12
The Official SAT Online Course 页码,13/15

In triangle above, if and what is the area of triangle

(A)

(B)

(C)

(D)

(E)

ANSWERS AND EXPLANATIONS


Explanation for Correct Answer C :
Choice (C) is correct. If the base of triangle is then its height
is Substituting the values given in the problem, the area of triangle

is



Explanation for Incorrect Answer A :

d
Choice (A) is not correct. See the explanation for the correct response (C).

Explanation for Incorrect Answer B :

re 用
Choice (B) is not correct. This is the area of triangle


te
Explanation for Incorrect Answer D :
Choice (D) is not correct. This is the area of triangle


is
Explanation for Incorrect Answer E :
Choice (E) is not correct. See the explanation for the correct response (C).


eg


19
nR

If and are two different integers and the product is the square of an


integer, which of the following could be equal to

(A)


(B)
U

(C)

(D)

(E)

ANSWERS AND EXPLANATIONS


Explanation for Correct Answer D :
Choice (D) is correct. The prime factorization of is Thus, since

is the square of an integer, where is an

integer. It follows that If then

Explanation for Incorrect Answer A :

Choice (A) is not correct. which is not an integer.

The question asks for a value of that makes the square of an


integer.

file://E:\新建文件夹\a4.htm 2006-11-12
The Official SAT Online Course 页码,14/15

Explanation for Incorrect Answer B :

Choice (B) is not correct. which is not an

integer. The question asks for a value of that makes the square of
an integer.

Explanation for Incorrect Answer C :

Choice (C) is not correct. which is not an

integer. The question asks for a value of that makes the square of
an integer.

Explanation for Incorrect Answer E :

Choice (E) is not correct. which is not an

integer. The question asks for a value of that makes the square of
an integer.

20



d
re 用
On the number line above, the tick marks are equally spaced. Which of the lettered


te
points represents


is
(A) A
(B) B


eg

(C) C

(D) D

(E) E


nR


ANSWERS AND EXPLANATIONS


U

Explanation for Correct Answer E :


Choice (E) is correct. Regardless of the position of on the number line,

which is the average of and will always be exactly halfway

between and Since is units to the left of the point labeled

it follows that must be units to the right, or at the point labeled .

Explanation for Incorrect Answer A :


Choice (A) is not correct. See the explanation for the correct response (E).

Explanation for Incorrect Answer B :


Choice (B) is not correct. See the explanation for the correct response (E).

Explanation for Incorrect Answer C :


Choice (C) is not correct. See the explanation for the correct response (E).

Explanation for Incorrect Answer D :


Choice (D) is not correct. See the explanation for the correct response (E).

file://E:\新建文件夹\a4.htm 2006-11-12
The Official SAT Online Course 页码,15/15

Back to Score Report

Copyright © 2006 The College Board. All rights reserved. Privacy Policy Terms of Use Contact Us



d
re


te

is


eg


nR



U

file://E:\新建文件夹\a4.htm 2006-11-12
The Official SAT Online Course 页码,1/21

Help | Profile | My Organizer | My Bookmarks | Logout

Answers and Explanations

Test Sections Back to Score Report

Section 1 View Answers and Explanations


Section 2 Online - Practice Test #1

Section 3
1
Section 5
After winning the lottery, John bought sports cars, built a mansion, and wore designer
Section 6
suits; however, by thus ------- his -------, he alienated his friends.
Section 7
(A) enduring. . hardship
Section 8
Section 9 (B) flaunting . . prosperity
Section 10 (C) undermining. . image

(D) calculating . . successes

(E) moderating. . consumption



ANSWERS AND EXPLANATIONS

e d
Explanation for Correct Answer B :


Choice (B) is correct. "Flaunting" means displaying showily or pretentiously, and
"prosperity" means economic well-being. If one were to insert these terms into the
er
text, the sentence would read "After winning the lottery, John bought sports cars,


built a mansion, and wore designer suits; however, by thus flaunting his prosperity,
he alienated his friends." The word "thus" indicates that the missing terms will
t
paraphrase John's actions after he won the lottery, so it makes sense to say that


is
John's pretentious displays of wealth alienated, or aroused hostile feelings in, his
friends.


eg


Explanation for Incorrect Answer A :
nR

Choice (A) is incorrect. "Enduring" means undergoing or tolerating, and "hardship"


means misfortune. If one were to insert these terms into the text, the sentence


would read "After winning the lottery, John bought sports cars, built a mansion, and
wore designer suits; however, by thus enduring his hardship, he alienated his
friends." It makes little sense to say that winning a lottery is a misfortune that has


U

to be endured.

Explanation for Incorrect Answer C :


Choice (C) is incorrect. "Undermining" means weakening gradually and secretly,
and "image" means impression. If one were to insert these terms into the text, the
sentence would read "After winning the lottery, John bought sports cars, built a
mansion, and wore designer suits; however, by thus undermining his image, he
alienated his friends." It is more probable that John made these showy purchases to
build, not undermine, his image.

Explanation for Incorrect Answer D :


Choice (D) is incorrect. "Calculating" means figuring out or estimating, and
"successes" are things that turn out well. If one were to insert these terms into the
text, the sentence would read "After winning the lottery, John bought sports cars,
built a mansion, and wore designer suits; however, by thus calculating his
successes, he alienated his friends." Even though, John may have calculated his
successess, this probably would not have been why he alienated his friends.

Explanation for Incorrect Answer E :


Choice (E) is incorrect. "Moderating" means lessening in extent or intensity, and
"consumption," in this context, means spending money. If one were to insert these
terms into the text, the sentence would read "After winning the lottery, John
bought sports cars, built a mansion, and wore designer suits; however, by thus
moderating his consumption, he alienated his friends." Since the word "thus"
indicates that the missing terms will paraphrase John's actions after he won the
lottery, inserting these terms makes a completely illogical sentence. After winning
the lottery, John did not spend less money; he spent so much money that he
alienated his friends.

file://E:\新建文件夹\a5.htm 2006-11-12
The Official SAT Online Course 页码,2/21

2 The study of biology, once considered the key to solving nature’s mysteries, has
instead served to emphasize nature’s incredible -------.
(A) tranquility

(B) immobility
(C) consistency

(D) desirability

(E) complexity

ANSWERS AND EXPLANATIONS


Explanation for Correct Answer E :
Choice (E) is correct. “Complexity” means intricacy. If one were to insert this
term into the text, the sentence would read “The study of biology, once
considered the key to solving nature’s mysteries, has instead served to emphasize
nature’s incredible complexity.” Since the word “instead” indicates that what
follows will contrast in meaning with “solving nature’s mysteries,” it makes
sense to say that biology has not solved nature’s mysteries but has instead
uncovered new, perhaps unexplained, intricacies.



d
Explanation for Incorrect Answer A :

re
Choice (A) is incorrect. “Tranquility” means freedom from commotion or


disturbance. If one were to insert this term into the text, the sentence would read
“The study of biology, once considered the key to solving nature’s mysteries,
has instead served to emphasize nature’s incredible tranquility.” Since the word
te

“instead” indicates that what follows will contrast in meaning with “solving
nature’s mysteries,” it makes sense to say that biology, rather than solving
nature’s mysteries, has uncovered nature’s lack of commotion or disturbance.
is


Explanation for Incorrect Answer B :
Choice (B) is incorrect. “Immobility” means to be motionlessness. If one were to
eg


insert this term into the text, the sentence would read “The study of biology, once
considered the key to solving nature’s mysteries, has instead served to emphasize
nature’s incredible immobility.” Since the word “instead” indicates that what
follows will contrast in meaning with “solving nature’s mysteries,” it makes


sense to say that biology, rather than solving nature’s mysteries, biology has
nR

uncovered nature’s lack of movement.


Explanation for Incorrect Answer C :
Choice (C) is incorrect. “Consistency” means uniformity. If one were to insert
this term into the text, the sentence would read “The study of biology, once


U

considered the key to solving nature’s mysteries, has instead served to emphasize
nature’s incredible consistency.” Since the word “instead” indicates that what
follows will contrast in meaning with “solving nature’s mysteries,” it makes
sense to say that biology has uncovered nature’s uniformity, or predictability,
rather than solving its mysteries.

Explanation for Incorrect Answer D :


Choice (D) is incorrect. “Desirability” means pleasing. If one were to insert this
term into the text, the sentence would read “The study of biology, once
considered the key to solving nature’s mysteries, has instead served to emphasize
nature’s incredible desirability.” Since the word “instead” indicates that what
follows will contrast in meaning with “solving nature’s mysteries,” it makes
sense to say that biology has uncovered nature’s pleasing qualities rather than
solving its mysteries.

3 In 1575 Venetians instituted an annual celebration to ------- the end of the -------
that had struck the city.

(A) lament . . turmoil


(B) commemorate. . plague
(C) eulogize . . pestilence

(D) hail . . prosperity

file://E:\新建文件夹\a5.htm 2006-11-12
The Official SAT Online Course 页码,3/21

(E) solemnize. . fame

ANSWERS AND EXPLANATIONS


Explanation for Correct Answer B :
Choice (B) is correct. "Commemorate" means a remembrance, and a "plague" is an
epidemic disease. If one were to insert these terms into the text, the sentence
would read "In 1575 Venetians instituted an annual celebration to commemorate
the end of the plague that had struck the city." Since the first missing word refers
to the celebration's purpose, and the second to the event whose end was being
remembered, choice (B) makes sense: it is logical to rejoice over the memory of
the end of a plague.

Explanation for Incorrect Answer A :


Choice (A) is incorrect. "Lament" means to express sorrow, and "turmoil" means a
state of commotion. If one were to insert these terms into the text, the sentence
would read "In 1575 Venetians instituted an annual celebration to lament the end of
the turmoil that had struck the city." It would not make sense for citizens to be sad
about the end of a period of commotion or distress.


Explanation for Incorrect Answer C :
Choice (C) is incorrect. "Eulogize" means to praise, and "pestilence" is another word
for plague. If one were to insert these terms into the text, the sentence would read


d
"In 1575 Venetians instituted an annual celebration to eulogize the end of the
pestilence that had struck the city." Although citizens would certainly be happy that
a plague had come to an end, it is unlikely that they would arrange for a yearly

re 用
celebration merely to "praise" the plague's end.

Explanation for Incorrect Answer D :


te
Choice (D) is incorrect. "Hail" means to salute, and "prosperity" means economic
well-being. If one were to insert these terms into the text, the sentence would read
"In 1575 Venetians instituted an annual celebration to hail the end of the prosperity


is
that had struck the city." Venetians would be quite unlikely to salute the end of a
period of prosperity.


eg

Explanation for Incorrect Answer E :


Choice (E) is incorrect. "Solemnize" means to celebrate with dignity and gravity,
and "fame" means popular acclaim or renown. If one were to insert these terms


into the text, the sentence would read "In 1575 Venetians instituted an annual
celebration to solemnize the end of the fame that had struck the city." Although
nR

citizens might approach an annual celebration with solemnity and gravity, it does


not make sense to say that "fame" had struck the city.


U

4
Lena Horne’s singing style is such that she can invest even the most ------- lyrics
with dramatic meaning.

(A) harmonious
(B) sensational
(C) impeccable

(D) vapid

(E) esteemed

ANSWERS AND EXPLANATIONS


Explanation for Correct Answer D :
Choice (D) is correct. "Vapid" means dull and lacking interest. If one were to insert
this term into the text, the sentence would read "Lena Horne's singing style is such
that she can invest even the most vapid lyrics with dramatic meaning." Since the
word "even" is used as an intensifier in this sentence to indicate that the missing
term describes lyrics that are lacking in dramatic meaning, choice (D) works here.

Explanation for Incorrect Answer A :

file://E:\新建文件夹\a5.htm 2006-11-12
The Official SAT Online Course 页码,4/21

Choice (A) is incorrect. "Harmonious" means musically concordant or pleasing. If


one were to insert this term into the text, the sentence would read "Lena Horne's
singing style is such that she can invest even the most harmonious lyrics with
dramatic meaning." It makes no sense to speak of "harmonious lyrics"; harmony
has to do with music, not the words set to music.

Explanation for Incorrect Answer B :


Choice (B) is incorrect. "Sensational" means exaggeratedly dramatic. If one were to
insert this term into the text, the sentence would read "Lena Horne's singing style is
such that she can invest even the most sensational lyrics with dramatic meaning."
Choice (B) does not work here—"sensational" lyrics are already dramatic, even
excessively so.

Explanation for Incorrect Answer C :


Choice (C ) is incorrect. "Impeccable" means faultless. If one were to insert this
term into the text, the sentence would read "Lena Horne's singing style is such that
she can invest even the most impeccable lyrics with dramatic meaning." Since
"impeccable" lyrics are by definition faultless, it is illogical that they would need to
be enhanced by a singer.

Explanation for Incorrect Answer E :


Choice (E) is incorrect. "Esteemed" means highly regarded. If one were to insert
this term into the text, the sentence would read "Lena Horne's singing style is such
that she can invest even the most esteemed lyrics with dramatic meaning." If lyrics
were held in high regard, they would already be considered of high quality and


therefore would not need to be enhanced.


d
5
A long-standing theory about the migration of green turtles was ------- by an

work to the original researcher.


(A) instigated . . rancor re 用
innovative marine biologist who graciously defused potential ------- by dedicating her
te
(B) renounced. . approval
(C) displaced . . attribution 业

is
(D) enhanced . . alteration

(E) repudiated . . acrimony


eg


ANSWERS AND EXPLANATIONS
nR

Explanation for Correct Answer E :


Choice (E) is correct. "Repudiate" means, in this context, to reject as untrue, and
"acrimony" means a biting sharpness of speech or manner. If one were to insert
these terms into the text, the sentence would read "A long-standing theory about


the migration of green turtles was repudiated by an innovative marine biologist who
U

graciously defused potential acrimony by dedicating her work to the original


researcher." Since the first missing term refers to what the biologist did to the
theory, and the second missing term relates to consequences of research conducted
to reexamine the theory, choice (E) makes sense: the second researcher "defused,"
or made less potent, any bitterness that might arise out of her disproving the first
researcher's theory by dedicating the work to the first researcher, possibly
acknowledging a debt to the latter.

Explanation for Incorrect Answer A :


Choice (A) is incorrect. "To instigate" means to provoke or urge forward, and
"rancor" means ill will or bitterness. If one were to insert these terms into the text,
the sentence would read "A long-standing theory about the migration of green
turtles was instigated by an innovative marine biologist who graciously defused
potential rancor by dedicating her work to the original researcher." Although it
makes sense to say that the biologist defused potential ill will with a gracious
gesture, it does not make sense to say that she provoked or urged on the theory of
the original researcher and that her doing so might cause bitterness.

Explanation for Incorrect Answer B :


Choice (B) is incorrect. "Renounce" means to refuse to recognize or acknowledge
something further, and "approval" means acceptance of something as satisfactory.
If one were to insert these terms into the text, the sentence would read "A long-
standing theory about the migration of green turtles was renounced by an
innovative marine biologist who graciously defused potential approval by dedicating

file://E:\新建文件夹\a5.htm 2006-11-12
The Official SAT Online Course 页码,5/21

her work to the original "researcher." Although a theory might be renounced by a


forward-thinking biologist, it does not make sense to say that she avoided potential
approval, since it would be highly illogical to do so, especially with a gracious
gesture.

Explanation for Incorrect Answer C :


Choice (C ) is incorrect. "Displace" means to remove or to banish, and "attribution"
means the ascribing of a work to an author. If one were to insert these terms into
the text, the sentence would read "A long-standing theory about the migration of
green turtles was displaced by an innovative marine biologist who graciously
defused potential attribution by dedicating her work to the original researcher."
Although a theory might conceivably be displaced, or put aside, by an improved
theory, it makes no sense to speak of "defusing," or disarming, attribution of
authorship.

Explanation for Incorrect Answer D :


Choice (D) is incorrect. "Enhance" means to heighten or increase, and "alteration"
means modification. If one were to insert these terms into the text, the sentence
would read "A long-standing theory about the migration of green turtles was
enhanced by an innovative marine biologist who graciously defused potential
alteration by dedicating her work to the original researcher." Although a theory
might well be improved upon by new discoveries, it makes no sense to speak of
potential alteration, being defused

On the morning of June 13, 1998, a 4.6- !



billion-year-old

ed
extraterrestrial object streaked into Earth’s


atmosphere and
blew to pieces in the sky somewhere in the
er

neighborhood
of Nelda Wallace’s backyard. A dark
t
Line


basketball-size object
is

dropped with a loud ssshhht into Wallace’s


5


garden, and
eg

fragments pelted other properties—only the


first of many
nR

strange things soon to occur in town. For


meteorites are
more than just stars of science-fiction movies.
Scientists


U

covet them, private dealers scoop them up for


resale at
spiraling prices, and professional searchers
10
travel the
world to hunt them down. Nelda Wallace’s
town was
about to be invaded by meteorite dealers,
meteorite
fans, meteorite poachers, and other alien life-
forms.
6
The sentence in lines 1-4 (“On the morning . . . backyard”) is best characterized as

(A) ironic
(B) dramatic
(C) comical

(D) nostalgic

(E) celebratory

file://E:\新建文件夹\a5.htm 2006-11-12
The Official SAT Online Course 页码,6/21

ANSWERS AND EXPLANATIONS


Explanation for Correct Answer B :
Choice (B) is correct. This sentence describes an extraordinary event, emphasizing
its dramatic nature with terms like "streaked" and "blew to pieces." The sentence is
a forceful description of a spectacular event.

Explanation for Incorrect Answer A :


Choice (A) is incorrect. There is no irony in this sentence; it is a description of an
event.

Explanation for Incorrect Answer C :


Choice (C) is incorrect. The language in this sentence is straightforward, not
humorous.

Explanation for Incorrect Answer D :


Choice (D) is incorrect. Nostalgia is about remembering something in the past. This
sentence describes how an event occurred, not how people felt when remembering
it.


Explanation for Incorrect Answer E :
Choice (E) is incorrect. The event may be something to celebrate, but the sentence
describes the event objectively.


d
7

(A) re 用
The reference to the “alien life-forms” (line 13) primarily serves to

hint at the dangers posed by some unexpected visitors


te
(B) mock the public’s fascination with extraterrestrial beings
(C) indicate the dearth of reliable information about a subject


is
(D) acknowledge a lack of familiarity with a scientific phenomenon

(E) provide a humorous label for a certain kind of zealotry


eg


ANSWERS AND EXPLANATIONS
nR

Explanation for Correct Answer E :


Choice (E) is correct. The sentence is intended to mock gently the circuslike
atmosphere coming soon to Nelda Wallace's town. The author labels the unusual
visitors to the normally quiet town as "alien life-forms" to describe humorously the


types of people who are interested in meteorites.
U

Explanation for Incorrect Answer A :


Choice (A) is incorrect. There is nothing menacing or suggestive of danger in the
paragraph. "Alien life-forms" refers to human visitors, not to dangerous creatures.

Explanation for Incorrect Answer B :


Choice (B) is incorrect. The subtle mockery here is of those who will invade the
town in search of meteorites, not of the general public.

Explanation for Incorrect Answer C :


Choice (C ) is incorrect. The passage does not discuss the realibility of information
about meteorites.

Explanation for Incorrect Answer D :


Choice (D) is incorrect. The author of this paragraph clearly understands what
occurs when metorites fall to earth.

Apes raised by humans seem to pretend


more frequently

file://E:\新建文件夹\a5.htm 2006-11-12
The Official SAT Online Course 页码,7/21

than do apes in the wild. Animal handlers see


behaviors
they interpret as pretending practically every
day. But Anne
Russon, a psychologist, says she has found
Line
only about 20
recorded cases of possible pretending in free-
5
ranging
orangutans, culled from thousands of hours of
observation.
One possible reason, she noted in an e-mail
interview from
her field station in Borneo, is that researchers
have not been
looking for such behavior. But many
researchers believe
that interaction with humans—and the
10
encouragement to
pretend that comes with it—may play a major !
role in why

d
domesticated apes playact more.
8
re 用
Russon’s hypothesis would be most fully tested by which possible research project?


te
(A) Examining data from observations of pretending behavior in apes other
than orangutans
(B) Expanding ongoing observations of orangutans to include pretending


behavior
is
(C) Documenting pretending behavior among orangutans raised by humans

(D) Comparing specific pretending behaviors in free-ranging and domesticated


eg

orangutans
(E) Reviewing existing data on free-ranging orangutans to determine the


earliest record of pretending behavior
nR

ANSWERS AND EXPLANATIONS


Explanation for Correct Answer B :


Choice (B) is correct. Researchers had not been studying pretending behavior in
U

free-ranging orangutans; if they had done so, they would have had data to
compare with the data about orangutans raised by humans.

Explanation for Incorrect Answer A :


Choice (A) is incorrect. Orangutans are the only type of ape Russon has studied;
research on other apes would not be relevant.

Explanation for Incorrect Answer C :


Choice (C ) is incorrect. Lines 1-3 make it clear that most research on playacting
behaviors has focused on orangutans raised by humans, not on wild orangutans.

Explanation for Incorrect Answer D :


Choice (D) is incorrect. Only research on the free-ranging orangutans is needed to
fully test the hypothesis.

Explanation for Incorrect Answer E :


Choice (E) is incorrect. When such behavior was first recorded is not the issue; the
frequency of such behavior is the concern.

9
Which theoretical statement about pretending behavior in apes would be supported

file://E:\新建文件夹\a5.htm 2006-11-12
The Official SAT Online Course 页码,8/21

most fully by the “many researchers” mentioned in line 9?

(A) Having the ability to pretend has enabled apes, such as chimpanzees, to
be trained as performers.
(B) All types of apes, both wild and domesticated, can pretend with human
companions.
(C) Pretending behavior for wild apes may vary considerably by region and
population.
(D) Handlers of domesticated apes do not always have the rigorous
observational training of scientists.
(E) Wild apes living apart from humans pretend only rarely.

ANSWERS AND EXPLANATIONS


Explanation for Correct Answer E :
Choice (E) is correct. The implication of the paragraph's last sentence is that many
researchers feel that contact with humans determines the frequency of pretending
behavior in orangutans.

Explanation for Incorrect Answer A :


Choice (A) is incorrect. Performing might be a consequence of pretending behavior,
but it is not an explanation of that behavior.


Explanation for Incorrect Answer B :

ed
Choice (B) is incorrect. The researchers think that pretending behavior is learned
from humans, so they would not expect wild orangutans to demonstrate it.

Explanation for Incorrect Answer C :



er
Choice (C) is incorrect. Geographical or demographic influences are not discussed in


the passage.
st
Explanation for Incorrect Answer D :


Choice (D) is incorrect. The researchers in the passage are not assuming that the
apes' behavior has been misinterpreted; instead, they are arguing that it is real and
i

has been learned from humans.


eg


This excerpt is from a short story by a Japanese American writer. The narrator reflects
nR

on her family’s past as she helps her mother prepare to move from her home.


There’s a photograph of my mother
standing on the pier


U

in Honolulu in 1932, the year she left Hawaii to


attend the
University of California. She’s loaded to the
ears with leis.

Line She
’s wearing a fedora1 pulled smartly to the
side. She is
not smiling. Of my mother’s two years at the
5
university,
my grandmother recalled that she received
good grades
and never wore a kimono again. My second
cousin, with
whom my mother stayed when she first
arrived, said she
was surprisingly sophisticated—she liked hats.
My mother
10 said that she was homesick. Her favorite class

file://E:\新建文件夹\a5.htm 2006-11-12
The Official SAT Online Course 页码,9/21

was biology
and she entertained thoughts of becoming a
scientist. Her
father, however, wanted her to become a
teacher, and his
wishes prevailed, even though he would not
have forced
them upon her. She was a dutiful daughter.
During her second year, she lived near
15
campus with a
mathematics professor and his wife. In
exchange for room
and board she cleaned house, ironed, and
helped prepare
meals. One of the things that survives from this
period is a
black composition book entitled Recipes of

20
California. As

a child, I read it like a book of mysteries for
clues to a life

d
both alien and familiar. Some entries she had
copied by
re 用
hand; others she cut out of magazines and


pasted on the
te
page, sometimes with a picture or drawing. The


margins
is

contained her cryptic comments: “Saturday


bridge club,”


eg

25 “From Mary G. Do not give away.”


That book holds part of the answer to why
our family
nR

rituals didn’t fit the norm either of our


relatives or of the
larger community in which we grew up. At


U

home, we ate
in fear of the glass of spilled milk, the stray
elbow on the
table, the boarding house reach. At my
30
grandparents’, we
2
slurped our chasuke . We wore tailored dresses
and black
shoes with white socks; however, what we
longed for were
the lacy colorful dresses that other girls wore to
church on
Sunday. For six years, I marched to Japanese
language
school after my regular classes; however, we
35
only spoke
English at home. We talked too loudly and all at
once,

file://E:\新建文件夹\a5.htm 2006-11-12
The Official SAT Online Course 页码,10/21

which mortified my mother, but she was always


complaining
about Japanese indirectness. I know that she
smarted under
a system in which the older son is the center of
the familial
universe, but at thirteen I had a fit of jealous
40
rage over her
fawning attention to our only male cousin.
My sister has found a photograph of our
mother, a
round-faced and serious twelve or thirteen,
dressed in a
kimono and seated, on her knees, on the tatami
mat. She is
playing the koto, a difficult stringed instrument
45
thought to


teach girls discipline. Of course, everything
Japanese was


a lesson in discipline—flower arranging,

ed
embroidery,


everything. One summer my sister and I had to
take r

ikebana, the art of flower arrangement, at our
te
grandfather’s


school. The course was taught by Mrs. Oshima,
is
50
a soft-


spoken, terrifying woman, and my supplies
eg

were provided
by my grandmother, whose tastes ran to the
oversized.

nR

I remember little of that class and its principles.


What I
remember most clearly is having to walk home


U

carrying
one of our creations, which, more often than
55
not, towered
above our heads.
How do we choose among what we
experience, what
we are taught, what we run into by chance, or
what is
forced upon us? What is the principle of
selection? My
sisters and I are not bound by any of our
60
mother’s obli-
gations, nor do we follow the rituals that
seemed so
important. My sister once asked, do you realize
that when
she’s gone that’s it? She was talking about
how to make

file://E:\新建文件夹\a5.htm 2006-11-12
The Official SAT Online Course 页码,11/21

3
sushi , but it was a more profound question
nonetheless.

1 A fedora is a soft felt hat popular in the United States


in the 1930’s.

2 Chasuke is a rice and tea mixture.

3 Sushi is cold rice shaped into small cakes and


sometimes topped or wrapped with garnishes.

10 The thematic focus of the passage is on the


(A) conflicts between the narrator’s mother and grandmother
(B) challenge of balancing conflicting values and practices
(C) widespread assimilation of immigrants into the culture of the United States

(D) desirability of maintaining traditions

(E) !
irrelevance of traditional customs to modern society


d
ANSWERS AND EXPLANATIONS
Explanation for Correct Answer B : re 用

te
Choice (B) is correct. The passage offers examples of many American and Japanese
values and practices that must be balanced: table manners, clothing, and speech
are among the areas considered.


is


eg

Explanation for Incorrect Answer A :


Choice (A) is incorrect. Although the narrator is aware of different standards of


behavior in the two households, no conflict is ever mentioned.
nR

Explanation for Incorrect Answer C :


Choice (C) is incorrect. Although examples in the passage do illustrate some
assimilation of the narrator’s mother, the narrator does not discuss any
immigrants beyond her own family.


U

Explanation for Incorrect Answer D :


Choice (D) is incorrect. Although the narrator gives examples of Japanese traditions
valued by her grandmother and mother, the passage focuses on the challenge of
change rather than the desirability of maintaining traditions.

Explanation for Incorrect Answer E :


Choice (E) is incorrect. The passage does address both traditional customs and
modern society, but the issue of irrelevance is never raised.

11 The grandmother’s comments in lines 5-7 imply that her daughter’s experiences at
the university were characterized by

(A) success and camaraderie


(B) accomplishment and assimilation
(C) enlightenment and introspection

(D) diligence and homesickness

(E) scholarship and competition

ANSWERS AND EXPLANATIONS

file://E:\新建文件夹\a5.htm 2006-11-12
The Official SAT Online Course 页码,12/21

Explanation for Correct Answer B :


Choice (B) is correct. The fact that the daughter earns good grades shows her
academic accomplishment; that she never wears a kimono again shows that in her
manner of dress she is assimilated into American culture.

Explanation for Incorrect Answer A :


Choice (A) is incorrect. While the daughter does succeed in "[getting] good grades,"
no mention is made of friends or companions, which would evidence "camaraderie."

Explanation for Incorrect Answer C :


Choice (C) is incorrect. The grandmother’s comments about grades and dress do
not touch on either "enlightenment" (spiritual insight) or "introspection" (self-
examination).

Explanation for Incorrect Answer D :


Choice (D) is incorrect. Although "good grades" may have been a result of
"diligence," or hard work, the grandmother’s comments do not address
homesickness.

Explanation for Incorrect Answer E :


Choice (E) is incorrect. No competition with, for instance, other students is
necessarily suggested by the daughter’s "good grades," though scholarship may
have helped produce them.



ed
12 In line 11, the word "entertained" most nearly means

(A) regaled


(B) hosted
er
(C) flaunted


t
(D) harbored


is
(E) welcomed


eg

ANSWERS AND EXPLANATIONS


Explanation for Correct Answer D :
nR

Choice (D) is correct. The act of "entertaining" thoughts involves "harboring," or


holding them in the mind.



U

Explanation for Incorrect Answer A :


Choice (A) is incorrect. To "regale" someone is to provide him or her with great
enjoyment; it has nothing to do with holding thoughts in the mind.

Explanation for Incorrect Answer B :


Choice (B) is incorrect. In this context, the word "entertained" has to do with
thought, not with entertaining people.

Explanation for Incorrect Answer C :


Choice (C) is incorrect. The thoughts that the daughter entertains remain internal
and cannot therefore be "flaunted," or shown off.

Explanation for Incorrect Answer E :


Choice (E) is incorrect. Although the word "entertained" means that the daughter
thought about being a biologist, it does not suggest that she took great pleasure in
this.

13 The narrator’s statement in line 14 (“She . . . daughter”) serves to

(A) defend her mother’s interest in science


(B) justify her mother’s decision to leave home

file://E:\新建文件夹\a5.htm 2006-11-12
The Official SAT Online Course 页码,13/21

(C) explain why her mother became a teacher

(D) question the relevance of established customs

(E) rationalize her grandfather’s actions

ANSWERS AND EXPLANATIONS


Explanation for Correct Answer C :
Choice (C) is correct. The narrator’s mother becomes a teacher because “her
father wanted it” and because she allowed “his wishes to prevail.”

Explanation for Incorrect Answer A :


Choice (A) is incorrect. The narrator’s statement that her mother was “a dutiful
daughter” emphasizes the mother’s willingness to put aside her interest in
science and accept her father’s plans for her.

Explanation for Incorrect Answer B :


Choice (B) is incorrect. The narrator’s comment is made about a period of time
after her mother had already left home for college.

Explanation for Incorrect Answer D :


Choice (D) is incorrect. Although the narrator’s mother is a “dutiful daughter”


to her father, there is no suggestion that there is a tradition of her always acceding

ed
to his demands.


Explanation for Incorrect Answer E :
Choice (E) is incorrect. The narrator simply describes her mother’s response to
er
her grandfather’s wishes; she does not describe doing anything.


t

is
14

The narrator suggests that as a child she read her mother’s book of recipes in order


eg

to

(A) seek proof of her mother’s devotion to the family


(B) understand more fully the contradictions in her mother’s behavior
nR

(C) perpetuate the fantasy she created about her mother


(D) search for clues to her mother’s reluctance to discuss her past

(E) discover the cause of her mother’s unhappiness


U

ANSWERS AND EXPLANATIONS


Explanation for Correct Answer B :
Choice (B) is correct. The narrator reads the book in hopes of understanding her
mother, who is mysterious to her.

Explanation for Incorrect Answer A :


Choice (A) is incorrect. The narrator does not doubt her mother's loyalty to the
family; rather, she finds her puzzling as a person.

Explanation for Incorrect Answer C :


Choice (C) is incorrect. The narrator does not say that she had fantasies about her
mother.

Explanation for Incorrect Answer D :


Choice (D) is incorrect. The narrator does not suggest that her mother is unwilling
to talk about the past.

Explanation for Incorrect Answer E :


Choice (E) is incorrect. The narrator’s mother was homesick during her early time
in California, but the narrator offers no evidence of any unhappiness at the time she
read her mother’s recipe book.

file://E:\新建文件夹\a5.htm 2006-11-12
The Official SAT Online Course 页码,14/21

15 The description of the reaction of the mother to her children’s manner of speaking
(lines 36-38) highlights how she
(A) feared that her children’s naïveté would invite trouble
(B) shared her children’s distaste for Japanese language lessons
(C) was still imbued with the lessons of her culture

(D) insisted on maintaining a strong Japanese influence in her home

(E) wanted her children to be fluent in the Japanese language

ANSWERS AND EXPLANATIONS


Explanation for Correct Answer C :
Choice (C) is correct. Despite her complaints about "Japanese indirectness," the
mother is still embarrassed by her children’s loud voices and tendency to talk "all
at once."


Explanation for Incorrect Answer A :
Choice (A) is incorrect. The children are loud and speak "all at once," but they are
not naïve. The mother is "mortified" by their volume and manners but not fearful


d
for them.

re
Explanation for Incorrect Answer B :


Choice (B) is incorrect. Although the mother complains about "Japanese
indirectness," she does not express any dislike for the language lessons.
te

Explanation for Incorrect Answer D :
Choice (D) is incorrect. Only English is spoken in the narrator’s home, and the
passage describes the mother as having mixed feelings about Japanese customs.


is
Explanation for Incorrect Answer E :
Choice (E) is incorrect. The mother’s concern about the children’s way of


eg

speaking has to do with their volume and their talking simultaneously, not with
their Japanese language skills.


nR


16 The narrator repeats the word "everything" in lines 46-48 to

(A) explain the intensity of her competition with her sister


U

(B) characterize the diverse achievements of Japanese Americans


(C) describe her mother’s single-minded pursuit of perfection

(D) emphasize the extent to which discipline governed Japanese life

(E) highlight the extraordinary skill required to master the koto

ANSWERS AND EXPLANATIONS


Explanation for Correct Answer D :
Choice (D) is correct. The repetition of "everything" in a list of occupations requiring
great control calls attention to the pervasiveness of discipline in her mother’s
Japanese upbringing.

Explanation for Incorrect Answer A :


Choice (A) is incorrect. Although the sisters both studied ikebana, the narrator is
concerned with discipline, not competition.

Explanation for Incorrect Answer B :


Choice (B) is incorrect. The narrator lists representative activities not to show their
variety but to illustrate the range of behaviors that require discipline.

Explanation for Incorrect Answer C :

file://E:\新建文件夹\a5.htm 2006-11-12
The Official SAT Online Course 页码,15/21

Choice (C) is incorrect. The narrator is interested in the ways discipline pervaded
her mother’s life, not suggesting that her mother strove for perfection.

Explanation for Incorrect Answer E :


Choice (E) is incorrect. Although the narrator mentions that the koto was "thought
to teach girls discipline," she repeats "everything" to emphasize the extent to which
discipline was required in other activities as well.

17 In lines 53-56, the narrator’s description of childhood walks home from ikebana
class conveys a sense of
(A) adventure
(B) relief
(C) melancholy

(D) absurdity

(E) vitality

ANSWERS AND EXPLANATIONS


Explanation for Correct Answer D :
Choice (D) is correct. The picture of young girls dwarfed by flower arrangements
suggests a sense of "absurdity," or ridiculous incongruity.


ed 用
er
Explanation for Incorrect Answer A :
Choice (A) is incorrect. The description does not indicate any sense of excitement or


"adventure."
t
Explanation for Incorrect Answer B :


is
Choice (B) is incorrect. The narrator would not have been relieved to be walking
home with a flower arrangement that "more often than not, towered above" her
head.


eg

Explanation for Incorrect Answer C :


Choice (C) is incorrect. The description offers a picture that is odd but not sad.


nR

Explanation for Incorrect Answer E :


Choice (E) is incorrect. The description of the girls and the flower arrangements


does not carry any particular sense of "vitality," or energy.


U

18
To the narrator, her sister’s question (lines 62-63) implies that the

(A) mother represented the last true vestige of the sisters’ Japanese heritage

(B) mother should have made more of an effort to educate her daughters
about their background
(C) mother’s education in California extended beyond the confines of the
university
(D) sisters were saddened by their mother’s decision to move

(E) sisters would not regret the absence of traditional family rituals

ANSWERS AND EXPLANATIONS


Explanation for Correct Answer A :
Choice (A) is correct. The narrator understands her sister’s question to refer to
the making of sushi as well as to every other aspect of their Japanese heritage.

Explanation for Incorrect Answer B :


Choice (B) is incorrect. The rest of the passage makes clear the ways the mother
has made her daughters aware of their heritage and background.

file://E:\新建文件夹\a5.htm 2006-11-12
The Official SAT Online Course 页码,16/21

Explanation for Incorrect Answer C :


Choice (C) is incorrect. The question refers to the end of cultural influence, not to
the scope of the mother’s education.

Explanation for Incorrect Answer D :


Choice (D) is incorrect. The sister asked the question sometime in the past,
presumably before the mother thought of moving.

Explanation for Incorrect Answer E :


Choice (E) is incorrect. Though the sisters do not follow the traditional customs
themselves, the question strongly suggests that they will miss such customs.

The following passage is excerpted from a historian’s examination of European


attitudes toward childhood.

Medieval European art until about the


twelfth century
did not know childhood or did not attempt to
portray it.
It is hard to believe that this neglect was due to
incompe-
tence or incapacity; it seems more probable !
Line


d
that there was
no place for childhood in the medieval world. A
5
miniature
painted during the twelfth century provides usre 用

te
with a
striking example of the deformity that an artist


is
at that time
would inflict on the representation of children’


eg

s bodies.
The subject is a Biblical scene in which Jesus is
surrounded

nR

by little children. Yet the miniaturist has


10


grouped around
Jesus what are obviously eight men, without


any charac-
U

teristics of childhood; they have simply been


depicted on
a smaller scale. In a French miniature of the
late eleventh
century, three children brought to life by a
saint are also
reduced to a smaller scale than the adults,
15
without any
other difference in expression or features. A
painter would
not hesitate to give the body of a child the
musculature of
an adult.
In the world of pictorial formulas inherited
from
ancient Rome, right up to the end of the
20
thirteenth century,

file://E:\新建文件夹\a5.htm 2006-11-12
The Official SAT Online Course 页码,17/21

there are no children characterized by a special


expression,
but only adults on a reduced scale. This refusal
to accept
*
child morphology in art is to be found too in
most of the
ancient civilizations. A fine Sardinian bronze of
the ninth
century B.C. shows a mother holding in her
25
arms the bulky
body of her son. The museum catalog tells us:
“the little
masculine figure could also be a child which, in
accor-
dance with the formula adopted in ancient
times by other
peoples, had been represented as an adult.”
Everything in

fact would seem to suggest that the realistic


30

d
representation
of children or the idealization of childhood was
confined
re 用
to ancient Greek art. Representations of Eros,


te
the Greek
child god of love, proliferated in that Hellenistic


is
period,
but childhood disappeared from art together


eg

with the other


Hellenistic themes, and the subsequent
35


Romanesque art
nR

returned to the rejection of the special features


of

childhood.


This is no mere coincidence. Our starting
U

point in
this study is a world of pictorial representation
in which
childhood is unknown; literary historians such
40
as Calvé
have made the same observation about the
medieval epic,
in which child prodigies behave with the
courage and
physical strength of doughty warriors. This
undoubtedly
meant that the people of the tenth and eleventh
centuries
did not dwell on the image of childhood and
45
that the
image had neither interest nor even reality for
them. It

file://E:\新建文件夹\a5.htm 2006-11-12
The Official SAT Online Course 页码,18/21

suggests too that in the realm of real life, and


not simply
in that of aesthetic translation, childhood was a
period of
transition that passed quickly and that was just
as quickly
50 forgotten.

* Structure and form

19 The first two paragraphs (lines 1-37) primarily serve to


(A) argue against the depiction of children in artwork
(B) suggest that medieval Western art was particularly conservative
(C) describe the unrealistic portrayal of children in medieval art

(D) trace the evolution of realistic representation in Western art

(E) postulate a theory about the thematic focuses of medieval Western art

ANSWERS AND EXPLANATIONS !



ed
Explanation for Correct Answer C :


Choice (C) is correct. The first two paragraphs offer several examples in support of
the claim that medieval European art did not accurately portray children's specific
physical characteristics.
r

te

is
Explanation for Incorrect Answer A :
Choice (A) is incorrect. Lines 1-37 do not argue that children should not be


eg

portrayed; rather, they describe how they are portrayed.

Explanation for Incorrect Answer B :


Choice (B) is incorrect. Although the second paragraph suggests that medieval art
was conservative in that it preserved an old tradition, the main focus of the two
nR

paragraphs is on representations of children.


Explanation for Incorrect Answer D :
Choice (D) is incorrect. The first two paragraphs primarily supply examples of


unrealistic representations. The second paragraph mentions a period in Greek art
U

when realistic representation of children flourished, but it makes clear that this
period was followed by a return to lack of realism.

Explanation for Incorrect Answer E :


Choice (E) is incorrect. The purpose of the first two paragraphs is not to address
the themes of medieval art but to address the lack of accurate representations of
childhood. Possible theories about this lack of realistic childhood imagery are put
forward only in the third paragraph.

20
The author’s argument about the depiction of children in medieval art assumes that
the depictions

(A) suggest the connection between medieval art and religion


(B) prefigure the gradual shift to realism
(C) are too varied to support any one argument

(D) reflect earlier civilizations’ corruption

(E) offer an indication of commonly held attitudes

ANSWERS AND EXPLANATIONS

file://E:\新建文件夹\a5.htm 2006-11-12
The Official SAT Online Course 页码,19/21

Explanation for Correct Answer E :


Choice (E) is correct. The author's argument assumes that inaccurate depictions of
children in medieval art are a direct result of the era's widespread lack of interest in
the nature of children or in childhood images.

Explanation for Incorrect Answer A :


Choice (A) is incorrect. The author is not concerned with the connection between
religion and medieval art. The passage mentions Biblical scenes in art only to
examine the way in which children are depicted in them.

Explanation for Incorrect Answer B :


Choice (B) is incorrect. The author is concerned not with tracing the evolution of
artistic depictions of childhood but with indicating a direct connection between
artistic interpretations of children and real-life medieval attitudes toward them.

Explanation for Incorrect Answer C :


Choice (C) is incorrect. The basis of the author's argument is that depictions of
children in medieval art were not varied but were almost uniformly inaccurate.

Explanation for Incorrect Answer D :


Choice (D) is incorrect. Although the passage mentions childhood images that date
to the ninth century B.C., there is no indication that the author views these earlier
cultures as corrupt.



d
21 The author’s argument is developed primarily by

(A)

(B) re
quotations from literary sources

descriptions of visual evidence 用



te
(C) psychological analyses of medieval artists

(D) comparisons of modern and medieval images of the body


is
(E) reflections on the philosophical nature of childhood


eg

ANSWERS AND EXPLANATIONS


Explanation for Correct Answer B :
nR

Choice (B) is correct. Most of the passage is devoted to examples of depictions of


children in medieval paintings.



U

Explanation for Incorrect Answer A :


Choice (A) is incorrect. The author doesn't quote any literary sources and only
mentions in passing one literary historian.

Explanation for Incorrect Answer C :


Choice (C) is incorrect. Although the passage offers a fleeting sociological analysis
of medieval people—that childhood images "had neither interest nor even reality
for them"—the author does not provide any analyses of medieval artists.

Explanation for Incorrect Answer D :


Choice (D) is incorrect. The passage does not mention modern images of the body;
it discusses only medieval and ancient images of children's bodies.

Explanation for Incorrect Answer E :


Choice (E) is incorrect. The passage does not consider the nature of childhood;
rather, it infers from the way children are depicted in medieval art that medieval
Europeans did not consider childhood to have a philosophical nature worth
reflecting upon.

22 The last sentence of the passage (lines 46-50) primarily serves to

(A) define an important term that is central to the author’s argument

file://E:\新建文件夹\a5.htm 2006-11-12
The Official SAT Online Course 页码,20/21

(B) dismiss objections to the author’s thesis

(C) provide an explanation for the phenomenon discussed in the previous


paragraphs
(D) introduce examples from other time periods and other forms of
representational art
(E) summarize the views of other historians of medieval art

ANSWERS AND EXPLANATIONS


Explanation for Correct Answer C :
Choice (C) is correct. The last sentence offers an explanation for why children were
so rarely accurately depicted in medieval art—namely, because childhood was
considered a forgettable state that did not merit accurate depiction.

Explanation for Incorrect Answer A :


Choice (A) is incorrect. The last sentence does not define any of the terms of the
argument; it suggests an explanation for the examples of childhood depictions
listed earlier in the passage.

Explanation for Incorrect Answer B :


Choice (B) is incorrect. The last sentence expands on the author's thesis that
images of childhood held no interest for people of the tenth and eleventh centuries,


d
but no objections to this thesis are indicated in the passage.

re
Explanation for Incorrect Answer D :


Choice (D) is incorrect. The last sentence does not introduce any examples; it
provides an explanation for the examples of inaccurate childhood depictions in art
introduced earlier in the passage.


te
Explanation for Incorrect Answer E :
Choice (E) is incorrect. The historian Calvé is mentioned earlier in the same


is
paragraph; however, neither his nor any other historian's views are discussed in the
last sentence, which attempts to explain why childhood was virtually ignored in
medieval art.


eg

23


nR

In line 48, “translation” most nearly means


(A)

(B)
substitution

explanation


U

(C) representation

(D) transportation

(E) correction

ANSWERS AND EXPLANATIONS


Explanation for Correct Answer C :
Choice (C) is correct. In line 48, "translation" refers to the "representation," or
artistic depiction, of childhood.

Explanation for Incorrect Answer A :


Choice (A) is incorrect. In line 48, "translation" refers not to a "substitution" for life
but to a rendering of life that is unrealistic.

Explanation for Incorrect Answer B :


Choice (B) is incorrect. "Explanation" is not a possible meaning in the context of
line 48, since the "translation" doesn't attempt to explain or even recognize
childhood.

Explanation for Incorrect Answer D :


Choice (D) is incorrect. In the context of line 48, "translation" is not used to mean

file://E:\新建文件夹\a5.htm 2006-11-12
The Official SAT Online Course 页码,21/21

something that "transports" its viewers.

Explanation for Incorrect Answer E :


Choice (E) is incorrect. The "translation" discussed in line 48 is not a "correction" of
real life as much as it is a rendering of it.

24 The author offers which explanation for the way that medieval painters depicted
children?
(A) Children were discouraged from becoming artists’ models.
(B) Children were more difficult to paint than adults.
(C) Children had never been a subject of art in Western traditions.

(D) Childhood was not understood as a separate phase of life.

(E) Childhood was not recognized in medieval theology.

ANSWERS AND EXPLANATIONS


Explanation for Correct Answer D :
Choice (D) is correct. The passage indicates that "childhood was a period of


transition" that was seen not as a separate phase but as a quickly forgotten period
of pre-adulthood.


d
Explanation for Incorrect Answer A :

re 用
Choice (A) is incorrect. There is no mention in the passage of artists' models, either
children or adults.


te
Explanation for Incorrect Answer B :
Choice (B) is incorrect. Children may well have been more difficult to paint than


is
adults, but the passage indicates that "incompetence or incapacity" (lines 3-4) on
the part of an artist was not a likely explanation for the absence of accurate
childhood depictions.


eg

Explanation for Incorrect Answer C :


Choice (C) is incorrect. The passage points out that "representations of Eros, the


Greek child god of love," were plentiful in ancient Greek paintings.
nR

Explanation for Incorrect Answer E :


Choice (E) is incorrect. According to the passage, one medieval painting was a
depiction of Jesus surrounded by small children, indicating that childhood was
indeed recognized in medieval theology (even if painters depicted children as small


U

adults).

Back to Score Report

Copyright © 2006 The College Board. All rights reserved. Privacy Policy Terms of Use Contact Us

file://E:\新建文件夹\a5.htm 2006-11-12
The Official SAT Online Course 页码,1/9

Help | Profile | My Organizer | My Bookmarks | Logout

Answers and Explanations

Test Sections Back to Score Report

Section 1 View Answers and Explanations


Section 2 Online - Practice Test #1

Section 3
1 Which of the following triples does NOT satisfy the equation
Section 5
Section 6
(A)
Section 7
Section 8 (B)

Section 9 (C)
Section 10
(D)

(E)

ANSWERS AND EXPLANATIONS !



d
Explanation for Correct Answer E :
Choice (E) is correct. The triple

re 用
results in

Therefore, this triple does not satisfy the equation.


te

is
Explanation for Incorrect Answer A :
Choice (A) is not correct. The triple results in


eg

However, the question asks for a triple that does not


satisfy the equation.


Explanation for Incorrect Answer B :
nR

Choice (B) is not correct. The triple results in


satisfy the equation.
However, the question asks for a triple that does not


U

Explanation for Incorrect Answer C :


Choice (C) is not correct. The triple results in However,
the question asks for a triple that does not satisfy the equation.

Explanation for Incorrect Answer D :


Choice (D) is not correct. The triple results in
However, the question asks for a triple that does not satisfy the equation.

2 An amusement park charges more for an adult’s admission than for a child’s
admission. If a group of adults and children spent on admission, what is
the price of admission for one child?

(A)

(B)

(C)

(D)

(E)

ANSWERS AND EXPLANATIONS

file://E:\新建文件夹\a6.htm 2006-11-12
The Official SAT Online Course 页码,2/9

Explanation for Correct Answer B :


Choice (B) is correct. Let be the price of admission, in dollars, for a child. The
price of admission for an adult is then Since the total cost of admission for
adults and children is it follows that This

simplifies to Thus, and

Explanation for Incorrect Answer A :


Choice (A) is not correct. See the explanation for the correct response (B).

Explanation for Incorrect Answer C :


Choice (C) is not correct. See the explanation for the correct response (B).

Explanation for Incorrect Answer D :


Choice (D) is not correct. See the explanation for the correct response (B).

Explanation for Incorrect Answer E :


Choice (E) is not correct. See the explanation for the correct response (B).

3


d
re 用

te
The figure above shows four apartments in a building. In this building, each


apartment is occupied by only one person. Alice lives next to Sam, and Paul lives next
is
to Alice and Dara. In which apartment could Alice live?


eg

(A) only
(B) only
(C)


only
nR

(D) or


(E) or


U

ANSWERS AND EXPLANATIONS


Explanation for Correct Answer D :
Choice (D) is correct. It is given that Alice lives next to Sam. From the fact that
Paul lives next to Alice, you can conclude that Alice must live in an apartment that
has neighbors on both sides (Sam and Paul do not live in the same apartment). The
only two apartments that satisfy this condition are apartments 2 and 3. Both of
these apartments are possible, because the following two arrangements meet all
the conditions given in the problem. Sam could live in apartment 1, Alice in
apartment 2, Paul in apartment 3, and Dara in apartment 4. Dara could live in
apartment 1, Paul in apartment 2, Alice in apartment 3, and Sam in apartment 4.

Explanation for Incorrect Answer A :


Choice (A) is not correct. Apartment 1 only has one neighboring apartment, and
Alice must live in an apartment with two neighboring apartments.

Explanation for Incorrect Answer B :


Choice (B) is not correct. While apartment 2 is a valid answer, apartment 3 is also a
valid answer.

Explanation for Incorrect Answer C :


Choice (C) is not correct. While apartment 3 is a valid answer, apartment 2 is also a
valid answer.

file://E:\新建文件夹\a6.htm 2006-11-12
The Official SAT Online Course 页码,3/9

Explanation for Incorrect Answer E :


Choice (E) is not correct. Neither apartment 1 nor apartment 4 is a valid choice for
where Alice could live, because Alice must live in an apartment that has two
neighboring apartments.

4 What is the ratio of the radius of a circle to the circumference of the circle?
(A)

(B)

(C)

(D)

(E)

ANSWERS AND EXPLANATIONS


Explanation for Correct Answer A :
Choice (A) is correct. The circumference of a circle with radius is equal to
Thus, the ratio of the radius to the circumference is This simplifies to



d
Explanation for Incorrect Answer B :

re
Choice (B) is not correct. This is the ratio of the diameter to the circumference.

Explanation for Incorrect Answer C :




Choice (C) is not correct. This is the ratio of the radius to the area.
te
Explanation for Incorrect Answer D :


Choice (D) is not correct. This is the ratio of the circumference to the diameter.
is

Explanation for Incorrect Answer E :


eg

Choice (E) is not correct. This is the ratio of the circumference to the radius.


nR



U

The graph above shows various temperatures from 10 A.M. to 6 P.M. of a given day.
Which of the following situations best fits the information on the graph?

(A) It rained a little, and then the Sun came out and warmed things up.

(B) The mild temperature was lowered by a heavy rain in the morning, and
the temperature dropped lower by evening.
(C) It was more windy in the morning than it was in the evening, and the
temperature was mild throughout.
(D) The morning was cold, but the Sun later came out and raised the
temperature.
(E) The temperature decreased at a constant rate from 10 A.M. to 6 P.M.

ANSWERS AND EXPLANATIONS


Explanation for Correct Answer B :
Choice (B) is correct. The graph shows a sharp decrease in temperature in the

file://E:\新建文件夹\a6.htm 2006-11-12
The Official SAT Online Course 页码,4/9

morning and a slight further decrease in temperature in the afternoon. The situation
described by choice (B) best fits this information.

Explanation for Incorrect Answer A :


Choice (A) is not correct. If this description were true, you would expect to see an
increase in temperature throughout the day. The graph, however, shows a decrease
in temperature.

Explanation for Incorrect Answer C :


Choice (C) is not correct. If this description were true, you would expect to see
roughly constant temperatures throughout the day. The graph, however, shows a
decrease in temperature.

Explanation for Incorrect Answer D :


Choice (D) is not correct. If this description were true, you would expect to see an
increase in temperature in the afternoon. The graph, however, shows a decrease in
temperature in the afternoon.

Explanation for Incorrect Answer E :


Choice (E) is not correct. If this description were true, you would expect to see the
temperature decrease at a constant rate throughout the day. The graph, however,
shows a sharper decrease in temperature in the morning than in the afternoon.



d
6 Rita’s dog weighed pounds when she bought it. Over the next several years, the
dog’s weight increased by percent per year. Which of the following functions
gives the weight,

(A)
re
in pounds, of the dog after


years of weight gain at this rate?


te
(B)


(C)
is

(D)


eg

(E)


nR

ANSWERS AND EXPLANATIONS


Explanation for Correct Answer D :
Choice (D) is correct. The dog’s weight increases by percent per year. Each


year, therefore, the dog’s new weight is the dog’s weight the previous year plus
U

times the dog’s weight the previous year. This is the same as multiplying the
previous weight by Therefore, after years, the initial weight of pounds
has increased to pounds.

Explanation for Incorrect Answer A :


Choice (A) is not correct. See the explanation for the correct response (D).

Explanation for Incorrect Answer B :


Choice (B) is not correct. See the explanation for the correct response (D).

Explanation for Incorrect Answer C :


Choice (C) is not correct. See the explanation for the correct response (D).

Explanation for Incorrect Answer E :


Choice (E) is not correct. See the explanation for the correct response (D).

If all four interior angles of quadrilateral have the same measure, which of the
following statements must be true?

file://E:\新建文件夹\a6.htm 2006-11-12
The Official SAT Online Course 页码,5/9

I. All sides of have equal length.

II. The diagonals of are perpendicular.

III. The measure of each interior angle is

(A) None
(B) I only
(C) II only

(D) III only

(E) I, II, and III

ANSWERS AND EXPLANATIONS


Explanation for Correct Answer D :
Choice (D) is correct.

I. This statement is not necessarily true. A rectangle that is not a square is an


example of a quadrilateral in which all four interior angles do have the same
measure but the four sides do not all have the same length.


II. This statement is not necessarily true. A rectangle that is not a square is an
example of a quadrilateral in which all four interior angles do have the same
measure but the diagonals are not perpendicular.


d
III. This statement must be true. The sum of the measures of the interior angles
in a quadrilateral is If all four interior angles have the same measure,

re
they each have a measure of



te

is
Explanation for Incorrect Answer A :
Choice (A) is not correct. Statement III must be true.


eg

Explanation for Incorrect Answer B :


Choice (B) is not correct. Statement I is not true for rectangles that are not
squares.


nR

Explanation for Incorrect Answer C :


Choice (C) is not correct. Statement II is not true for rectangles that are not
squares



Explanation for Incorrect Answer E :
U

Choice (E) is not correct. While statement III must be true, neither statement I nor
II is necessarily true.

8
If and what is in terms of

(A)

(B)

(C)

(D)

(E)

ANSWERS AND EXPLANATIONS


Explanation for Correct Answer C :
Choice (C) is correct. Substituting for in the equation yields the
equation From this, it follows that

file://E:\新建文件夹\a6.htm 2006-11-12
The Official SAT Online Course 页码,6/9

Explanation for Incorrect Answer A :

Choice (A) is not correct.

Explanation for Incorrect Answer B :

Choice (B) is not correct.

Explanation for Incorrect Answer D :

Choice (D) is not correct.

Explanation for Incorrect Answer E :

Choice (E) is not correct.

9


ed 用
er

st


A snack machine has buttons arranged as shown above. If a selection is made by
choosing a letter followed by a one-digit number, what is the greatest number of
different selections that could be made?
i


eg

Your Response:


nR

Correct Response(s): 54


Explanation:

The correct answer is 54. To determine the number of possible different selections,


U

you multiply the number of possible choices of a letter by the number of possible
choices of a number. Therefore, the greatest number of different selections is 6 ×
x 9 = 54 selections.

10 If and and are positive integers, what is one possible value of

Your Response:

Correct Response(s): 3 or 6 or 9

Explanation:

The three possible correct answers are and It is given that and are
positive integers. Since is even and is odd, must be odd. If
then and If then and If then
and If then but it is given that must be positive.
So there are three possible values of and

11
A rectangular-shaped field has a perimeter of feet and a width of feet. What

file://E:\新建文件夹\a6.htm 2006-11-12
The Official SAT Online Course 页码,7/9

is the area of the field in square feet?

Your Response:

Correct Response(s): 9600

Explanation:

The correct answer is To determine the area of the field, you must first
determine its length. The formula for the perimeter of a rectangle is
The perimeter is given as and the width is given as Therefore,
which simplifies to feet. The area of a rectangle is the
length multiplied by the width, so the area equals square feet.

12
If what is the value of

Your Response:

Correct Response(s): 2



Explanation:

d
The correct answer is

be solved for
. The equation

as follows:
re 用
can


te

is


eg


nR

13 If


divided by one-half is , what is the value of


U

Your Response:

Correct Response(s): 25

Explanation:

The correct answer is Dividing by one-half is the same as multiplying by


two. So and

14

file://E:\新建文件夹\a6.htm 2006-11-12
The Official SAT Online Course 页码,8/9

In above, what is the length of

Your Response:

Correct Response(s): 9

Explanation:

The correct answer is Since has a measure of and the sum of the
measures of angles and is it follows that must
have a measure of Therefore, the measure of is
This identifies as a special right triangle, which
has the property that the shortest side (opposite the angle) is half as long as

the hypotenuse of the triangle. Therefore,

15 The sum of the positive odd integers less than is subtracted from the sum of the


positive even integers less than or equal to . What is the resulting difference?


d
Your Response:

e
Correct Response(s): 50


Explanation:
er
The correct answer is


The sum of the even integers less than or equal to
t
is The sum of the odd integers less than


is
is You can see that the first term in the first sum is
more than the first term in the second sum, the second term in the first sum is
more than the second term in the second sum, and so on. Since there are such


eg

pairs of terms, the difference between the two sums is

You can also approach this problem algebraically by considering the fact that every


positive odd integer less than can be written as and every positive
nR

even integer less than or equal to can be written as where is greater


than and less than or equal to Since there are terms in each sum and


the difference between each pair and is the difference between the
two sums is


U

16

The function above is defined in terms of another function for all values of
where is a constant. If is a number for which and what does
equal?

Your Response:

Correct Response(s): 3.5, 7/2

Explanation:

The correct answer is or Substituting the given values for and

into the given equation yields the equation This equation simplifies to

and so The answer can be entered into the grid as either a

fraction or a decimal.

file://E:\新建文件夹\a6.htm 2006-11-12
The Official SAT Online Course 页码,9/9

17 Fifty percent of the songs played on a certain radio station are minutes long,
percent are minutes long, and percent are minutes long. What is the
average (arithmetic mean) number of minutes per song played on this radio station?

Your Response:

Correct Response(s): 3.4, 17/5

Explanation:

The correct answer is or To determine the average length of songs

played on this radio station, you can take the appropriate fraction of each of the
lengths and add them together. Since of the songs are minutes long,
are minutes long, and are minutes long, the average length is
minutes. This value can also be expressed and
gridded as

18

A large solid cube is assembled by gluing together identical unpainted small cubic


blocks. All six faces of the large cube are then painted red. If exactly 27 of the small

d
cubic blocks that make up the large cube have no red paint on them, how many small
cubic blocks make up the large cube?

Your Response: re 用

te
Correct Response(s): 125


is
Explanation:


The correct answer is The core of the large cube is a cube of unpainted
eg

blocks, or a cube with length, width, and height Around this core is a shell of
blocks, one block thick, all of which are painted. This shell adds block to each


side of the core cube, or blocks total to each dimension of the core cube, making
the length, width, and height of the large cube each equal to A cube of this size
nR

is made up of blocks.



U

Back to Score Report

Copyright © 2006 The College Board. All rights reserved. Privacy Policy Terms of Use Contact Us

file://E:\新建文件夹\a6.htm 2006-11-12
The Official SAT Online Course 页码,1/21

Help | Profile | My Organizer | My Bookmarks | Logout

Answers and Explanations

Test Sections Back to Score Report

Section 1 View Answers and Explanations


Section 2 Online - Practice Test #1

Section 3
1 William Faulkner, being that he was a Southern writer, used Mississippi as a setting
Section 5 for most of his novels.
Section 6
(A) being that he was a Southern writer
Section 7
Section 8 (B) a Southern writer
Section 9 (C) while a writer from the South
Section 10
(D) in origin a writer of the South

(E) because of him being a writer from the South

ANSWERS AND EXPLANATIONS




ed
Explanation for Correct Answer B :
Choice (B) is correct. It avoids the error of the original by reducing the subordinate


clause introduced by "being that" to the modifying phrase "a Southern writer."

r

te
Explanation for Incorrect Answer A :


is
Choice (A) involves the use of an improper idiom. The phrase "being that" is
improperly used to introduce a subordinate clause ("he was a Southern writer")
where it would be more idiomatic to say, "William Faulkner, who was a Southern


eg

writer."

Explanation for Incorrect Answer C :


Choice (C) is unsatisfactory because it results in an illogical sentence. The word
nR

"while" indicates a contrast in ideas where there is no contrast.


Explanation for Incorrect Answer D :
Choice (D) involves the use of an improper idiom. It provides the modifying phrase
"in origin a writer of the South" where it would be more idiomatic to use a phrase


U

such as "a writer originally from the South."

Explanation for Incorrect Answer E :


Choice (E) results in an unsatisfactory sentence. The sentence would be clearer if
the awkward phrase "because of him being" were deleted and the simple modifying
phrase "a writer from the South" were left.

2
Tadpoles hatch and metamorphose into small replicas of adult frogs although
remaining in their aquatic birthplace.

(A) although remaining


(B) while remaining
(C) in spite of it remaining

(D) due to their remaining

(E) in the course of which they remain

ANSWERS AND EXPLANATIONS


Explanation for Correct Answer B :
Choice (B) is correct. It avoids the error of the original by replacing the adverb
“although,” which indicates contrast, with the adverb “while,” which indicates

file://E:\新建文件夹\a7.htm 2006-11-12
The Official SAT Online Course 页码,2/21

the time during which tadpoles change.

Explanation for Incorrect Answer A :


Choice (A) involves improper modification. The adverb "although" illogically
indicates a contrast between the two parts of the sentence, which are not
contrasting ideas.

Explanation for Incorrect Answer C :


Choice (C) involves the use of a vague pronoun. It is not clear what the pronoun
“it” refers to.

Explanation for Incorrect Answer D :


Choice (D) results in an illogical sentence. The phrase "due to" illogically indicates
that the development of tadpoles into frogs is caused by their remaining in water.

Explanation for Incorrect Answer E :


Choice (E) involves the use of a vague pronoun. The sentence is unsatisfactory
because the pronoun "which" appears to refer to an entire clause ("Tadpoles...
frogs").

3

You cannot expect to treat your friends badly and no one notices.


(A) and no one notices

ed
(B) and have no one notice


(C) without notice by someone

r
(D) without notice by no one


te
(E) without the result of somebody noticing


is
ANSWERS AND EXPLANATIONS
Explanation for Correct Answer B :


eg

Choice (B) is correct. It avoids the error of the original by replacing the phrase "and
no one notices," with the more idiomatic and parallel phrase "and have no one
notice."


nR


Explanation for Incorrect Answer A :


Choice (A) involves the improper use of an idiom. It provides the phrase "and no
U

one notices" where it would be more idiomatic to use the phrase "and have no one
notice."

Explanation for Incorrect Answer C :


Choice (C) involves improper diction. The sentence uses the word "notice" as a
noun. This use results in an illogical sentence because the noun "notice" has a
different meaning from the verb "notice."

Explanation for Incorrect Answer D :


Choice (D) is unsatisfactory because the use of the preposition "without," in
combination with the phrase "no one," results in a double negative.

Explanation for Incorrect Answer E :


Choice (E) involves wordiness. The phrase "the result of" is awkward and
unnecessary.

4 The memoirs of President Clinton begin with his childhood in Arkansas and culminate
in his rise to the presidency.

(A) begin with his childhood in Arkansas and culminate


(B) that begin with his childhood in Arkansas and culminate
(C) have begun with his childhood in Arkansas and culminate

file://E:\新建文件夹\a7.htm 2006-11-12
The Official SAT Online Course 页码,3/21

(D) have begun with his childhood in Arkansas and culminating

(E) began with his childhood in Arkansas and are culminated

ANSWERS AND EXPLANATIONS


Explanation for Correct Answer A :
Choice (A) is correct. The verbs "begin" and "culminate" are both in present tense,
and the two verbs join with prepositional phrases to produce parallel constructions.

Explanation for Incorrect Answer B :


Choice (B) creates a sentence fragment. The only verbs (“begin” and
“culminate”) are in a dependent clause introduced by “that,” so a complete
thought is not stated.

Explanation for Incorrect Answer C :


Choice (C) involves incorrect tense sequence. The present perfect tense of the verb
"have begun" is not consistent with the present tense of "culminate."

Explanation for Incorrect Answer D :


Choice (D) fails to maintain parallelism. The verbal "culminating" is not parallel with
the earlier verb "have begun."


ed
Explanation for Incorrect Answer E :
Choice (E) exhibits incorrect tense sequence. The past tense of the verb "began" is
not consistent with the present tense of "are culminated."

r 用
te
5



Because of ignoring its potential, biofeedback is a medical therapy most physicians
is
reject.

(A) Because of ignoring its potential, biofeedback is a medical therapy most


eg

physicians reject.
(B) Biofeedback is rejected by most physicians because of their ignoring its
potential.


(C) Most physicians, because of ignoring the potential of biofeedback, and
nR

rejecting it.
(D) Most physicians reject biofeedback because they ignore its potential.


(E) A medical therapy rejected by most physicians, caused by ignoring its
potential, is biofeedback.


U

ANSWERS AND EXPLANATIONS


Explanation for Correct Answer D :
Choice (D) is correct. It avoids the error of the original by using a clause with a
clearly identified actor—the pronoun "they" (which refers to "most physicians").

Explanation for Incorrect Answer A :


Choice (A) involves loose modification and inappropriate word order. The
introductory phrase, "Because of ignoring its potential," is far removed from the
only noun in the sentence it could logically modify—"physicians."

Explanation for Incorrect Answer B :


Choice (B) uses an inappropriate idiom. After the conjunction "because," the phrase
"of their ignoring" is less idiomatic than "they ignore."

Explanation for Incorrect Answer C :


Choice (C) creates a fragment. Since the phrase contains no verb (only the verbals
"ignoring" and "rejecting"), it does not state a complete thought.

Explanation for Incorrect Answer E :


Choice (E) displays improper modification. The verbal "caused" does not logically
modify the noun immediately before it, "physicians."

file://E:\新建文件夹\a7.htm 2006-11-12
The Official SAT Online Course 页码,4/21

6 In Death of a Salesman Willy Loman mistakenly believes that his sons have no flaws,
believing which leads to many problems for the entire family.
(A) believing which leads
(B) a belief that leads
(C) and which is to lead

(D) the belief of which leads

(E) his believing this leads

ANSWERS AND EXPLANATIONS


Explanation for Correct Answer B :
Choice (B) is correct. It avoids the error of the original by replacing the vague
pronoun "which" with the noun "belief" to identify Willy Loman's mistaken idea.

Explanation for Incorrect Answer A :


Choice (A) exhibits loose pronoun reference. The pronoun "which" refers to the idea
expressed by an entire clause ("that his sons have no flaws"), not to any specific
noun.


ed
Explanation for Incorrect Answer C :
Choice (C) involves loose pronoun reference. The pronoun "which" refers to the idea


expressed by an entire clause ("that his sons have no flaws"), not to any specific
noun.
er

Explanation for Incorrect Answer D :
Choice (D) displays wordiness. The phrase "of which" is not needed.
t

is
Explanation for Incorrect Answer E :
Choice (E) uses improper coordination. It joins two complete thoughts ("In Death . .
. no flaws" and "his believing . . . entire family") with only a comma.


eg


nR

Clara Barton, an American nurse, whose influence as a reformer in the field of health
care almost equals that of Florence Nightingale.


(A) Clara Barton, an American nurse, whose influence as a reformer


Clara Barton, who was an American nurse and whose influence as a
U

(B)
reformer
(C) An American with influence as a nursing reformer, Clara Barton

(D) Clara Barton was an American nurse whose influence as a reformer

(E) An American, Clara Barton who was a nursing reformer and whose
influence

ANSWERS AND EXPLANATIONS


Explanation for Correct Answer D :
Choice (D) is correct. It avoids the error of the original by forming an independent
clause with “Clara Barton” as the subject and “was” as the verb.

Explanation for Incorrect Answer A :


Choice (A) creates a sentence fragment. Since the only verb, “equals,” is within
a dependent clause, the sentence does not state a complete thought.

Explanation for Incorrect Answer B :


Choice (B) produces a sentence fragment. Since the only verb, “equals,” is within
a dependent clause, the sentence does not state a complete thought.

Explanation for Incorrect Answer C :

file://E:\新建文件夹\a7.htm 2006-11-12
The Official SAT Online Course 页码,5/21

Choice (C) makes an illogical comparison. It compares a person, “Clara Barton,”


with the influence of another person (“that of Florence Nightingale”).

Explanation for Incorrect Answer E :


Choice (E) involves a sentence fragment. The only verbs, "was" and "equals," are in
dependent clauses, so a complete thought is not stated.

8 During the summer months, several thousand people a day visit the park, which is
known for its waterfalls and rock formations.

(A) During the summer months, several thousand people a day visit the park,
which is known for its waterfalls and rock formations.
(B) Known for its waterfalls and rock formations, several thousand people a
day visit the park during the summer months.
(C) Several thousand people a day visit the park during the summer months
known for its waterfalls and rock formations.
(D) Several thousand people had visited the park a day, which is known for its
waterfalls and rock formations during the summer months.
(E) During the summer months, knowing its waterfalls and rock formations,
several thousand people a day visit the park.

ANSWERS AND EXPLANATIONS


Explanation for Correct Answer A :


Choice (A) is correct. The dependent clause introduced by "which" comes

ed
immediately after the noun it modifies, "park," and the adverb phrase "During the
summer months" comes near the verb it modifies, "visit."


er
Explanation for Incorrect Answer B :


st
Choice (B) displays improper modification. The introductory phrase ("Known for its


waterfalls and rock formations") cannot logically modify the noun that immediately
follows, "people."
i


eg

Explanation for Incorrect Answer C :


Choice (C) involves improper modification and incorrect word order. Since the
phrase "known for its waterfalls and rock formations" cannot logically modify the


noun "months," the phrase needs to be placed next to the noun it does apparently
modify, "park."
nR


Explanation for Incorrect Answer D :
Choice (D) uses incorrect word order. Since the adverbial phrase "during the
summer months" tells when people went to the park (not when it was known for its


waterfalls and rocks), the phrase is improperly separated from the verb it modifies,
U

"had visited."

Explanation for Incorrect Answer E :


Choice (E) exhibits incorrect word order. Since the adverbial phrase "during the
summer months" tells when people go to the park (not when they know about its
waterfalls and rocks), the phrase is improperly separated from the verb it modifies,
"visit."

9
Although the candidate promised both to cut taxes and improve services, he failed to
keep either of them after the election.

(A) Although the candidate promised both to cut taxes and improve services,
he
(B) The candidate, having promised both to cut taxes and improve services,

(C) Although the candidate made promises both to cut taxes and improve
services, he
(D) Having promised, first, to cut taxes and, second, to improve services, the
candidate
(E) The candidate’s promises were both to cut taxes and improve services,
he

ANSWERS AND EXPLANATIONS

file://E:\新建文件夹\a7.htm 2006-11-12
The Official SAT Online Course 页码,6/21

Explanation for Correct Answer C :


Choice (C) is correct. It avoids the error of the original by including the noun
"promises," to which the pronoun "them" can refer.

Explanation for Incorrect Answer A :


Choice (A) exhibits vague pronoun reference. The sentence contains no noun to
which the pronoun "them" (apparently meaning "promises") can refer.

Explanation for Incorrect Answer B :


Choice (B) displays vague pronoun reference. The pronoun "them" (apparently
meaning "promises") does not refer to any specific noun in the sentence.

Explanation for Incorrect Answer D :


Choice (D) involves vague pronoun reference. The sentence provides no noun to
which the pronoun "them" (apparently meaning "promises") can refer.

Explanation for Incorrect Answer E :


Choice (E) uses improper coordination. It joins two complete thoughts ("The
candidate's . . . improve services" and "he failed . . . after the election") with only a
comma.



d
10 The students’ fieldwork in the state forest proved more exciting and more dangerous
than any of them had anticipated, having to be rescued by helicopter during a fire.

(A)

(B)
anticipated, having to be

anticipated; when they had to be


re 用

te
(C) anticipated: they had to be

(D) anticipated: among which was their


is
(E) anticipated, and so they had been


eg

ANSWERS AND EXPLANATIONS


Explanation for Correct Answer C :


Choice (C) is correct. It avoids the error of the original by describing the need to be
nR

rescued in an independent clause with the pronoun "they" as its subject.



U

Explanation for Incorrect Answer A :


Choice (A) displays improper modification. The phrase "having to be
rescued" (presumably referring to students) has no noun or pronoun to modify.

Explanation for Incorrect Answer B :


Choice (B) uses improper coordination. It uses a semicolon to join unequal
sentence parts (a complete thought before the semicolon and a dependent clause
after it).

Explanation for Incorrect Answer D :


Choice (D) involves improper modification. The sentence contains no noun that the
connecting pronoun "which" can modify.

Explanation for Incorrect Answer E :


Choice (E) exhibits ineffective coordination. It uses the connecting phrase "and so"
to join two complete thoughts, but this linkage incorrectly suggests that the first
clause describes a cause and the second a result.

11 Chinese watercolors have become more popular than American and European artists
who are their contemporaries.

(A) American and European artists who are their contemporaries


(B) contemporary American and European artists

file://E:\新建文件夹\a7.htm 2006-11-12
The Official SAT Online Course 页码,7/21

(C) those by contemporary American and European artists of the period

(D) those of American and European pictures of the same period

(E) those by contemporary American and European artists

ANSWERS AND EXPLANATIONS


Explanation for Correct Answer E :
Choice (E) is correct. It avoids the error of the original by including the words
"those by" so that watercolors from China are compared with watercolors from
Europe and America.

Explanation for Incorrect Answer A :


Choice (A) contains an illogical comparison. It compares things, "watercolors," with
people, "American and European artists."

Explanation for Incorrect Answer B :


Choice (B) makes an illogical comparison. It compares things, "watercolors," with
people, "American and European artists."

Explanation for Incorrect Answer C :


Choice (C) displays wordiness. The phrase "of the period" is not needed since it


merely repeats the time specified earlier by the adjective "contemporary."

d
Explanation for Incorrect Answer D :

re 用
Choice (D) exhibits wordiness. The phrase "those of" is not needed and confusing
since the phrase also contains the noun "pictures."


te
12


is
By next year the old vaudeville theater had been converted into two small theaters


eg

in which films can be shown. No error


nR


ANSWERS AND EXPLANATIONS
Corrected Sentence: By next year the old vaudeville theatre will have been


U

converted into two small theaters in which films can be shown.

Explanation for Correct Answer A :


The error in this sentence occurs at (A), which involves an improper sequence of
tenses. The pluperfect verb phrase, "had been," does not agree with the phrase "By
next year" or with the tense of the verb phrase "can be shown" later in the
sentence.

Explanation for Incorrect Answer B :


There is no error at (B). The adjective "small" properly modifies the noun
"theaters."

Explanation for Incorrect Answer C :


There is no error at (C). The preposition "in" properly combines with the pronoun
"which" to refer to the two small theaters where the films can be shown.

Explanation for Incorrect Answer D :


There is no error at (D). The word "shown," a past participle of the verb "to show,"
properly combines with the phrase "can be" to produce an appropriate verb phrase.

Explanation for Incorrect Answer E : There is an error in the sentence.

file://E:\新建文件夹\a7.htm 2006-11-12
The Official SAT Online Course 页码,8/21

13
studying have become increasingly
Some scholars the writings of Emily Brontë

she No error
interested in the relationships between her siblings and .

ANSWERS AND EXPLANATIONS


Corrected Sentence: Some scholars studying the writings of Emily Brontë have
become increasingly interested in the relationships between her siblings and her.

Explanation for Correct Answer D :


The error in this sentence occurs at (D), where an improper pronoun case is used.
The nominative pronoun "she" is used in the prepositional phrase introduced by the
word "between" instead of the objective pronoun "her."


Explanation for Incorrect Answer A :
There is no error at (A). The participle “studying” properly introduces the
modifying phrase, “studying the writings of Emily Brontë,” which modifies


d
“scholars.”

re
Explanation for Incorrect Answer B :


There is no error at (B). The plural verb phrase "have become" agrees with its
plural subject, "scholars."


te
Explanation for Incorrect Answer C :
There is no error at (C). The adverb "increasingly" properly modifies the adjective
"interested."


is

Explanation for Incorrect Answer E : There is an error in the sentence.


eg


14
nR


At the 1984 Olympic Games, John Moffet and Pablo Morales, who were swimmers on


U

the United States team, set world records. No error

ANSWERS AND EXPLANATIONS


Corrected Sentence:

Explanation for Correct Answer E : There is no error in this sentence.

Explanation for Incorrect Answer A :


There is no error at (A). The pronoun "who," which can be either singular or plural,
agrees with its plural subject, "John Moffet and Pablo Morales," and is properly used
to introduce a subordinate clause.

Explanation for Incorrect Answer B :


There is no error at (B). The plural verb "were" agrees with its plural subject,
"who."

Explanation for Incorrect Answer C :


There is no error at (C). The past-tense verb "set" functions logically within the
1984 time frame of the sentence.

file://E:\新建文件夹\a7.htm 2006-11-12
The Official SAT Online Course 页码,9/21

Explanation for Incorrect Answer D :


There is no error at (D). The plural noun phrase, "world records," properly indicates
that Moffet and Morales had separate record-setting performances.

15
plus have written as well as
Both Dorothy Sayers Carolyn Heilbrun scholarly works

diversity of
popular murder mysteries, demonstrating a talents and interests.

No error

ANSWERS AND EXPLANATIONS


Corrected Sentence: Both Dorothy Sayers and Carolyn Heilbrun have written
scholarly works as well as popular murder mysteries, demonstrating a diversity of
talents and interests.

Explanation for Correct Answer A :


The error in this sentence occurs at (A), where the idiom is inappropriate. The


connecting word "plus" is not the correct term for use in a phrase beginning with

ed
"both."


er

Explanation for Incorrect Answer B :
There is no error at (B). The plural verb "have" agrees with its plural subject
t
("Dorothy Sayers" and "Carolyn Heilbrun"), and the present perfect tense is


is
appropriate to indicate action begun in the past and continuing to the present.

Explanation for Incorrect Answer C :


eg

There is no error at (C). The phrase "as well as" functions properly after the noun
"works" to indicate the addition of another item, "popular murder mysteries."


Explanation for Incorrect Answer D :
nR

There is no error at (D). The noun "diversity" functions correctly as the object of
the verbal "demonstrating," and "of" is the appropriate preposition to introduce a


phrase modifying "diversity."

Explanation for Incorrect Answer E : There is an error in the sentence.


U

16

Although the coach had predicted that the team would have a winning season, the

fans were surprised by the success of the young, inexperienced players. No error

ANSWERS AND EXPLANATIONS


Corrected Sentence:

Explanation for Correct Answer E : There is no error in this sentence.

Explanation for Incorrect Answer A :


There is no error at (A). The subordinating conjunction "Although" properly
introduces a subordinate adverbial clause.

file://E:\新建文件夹\a7.htm 2006-11-12
The Official SAT Online Course 页码,10/21

Explanation for Incorrect Answer B :


There is no error at (B). The word "predicted" combines with the word "had" to
produce an appropriate tense of the verb. The verb tense indicates that the coach's
prediction happened before the fans' reaction. The relative pronoun "that" properly
introduces the noun clause to state what the coach predicted.

Explanation for Incorrect Answer C :


There is no error at (C). The plural verb "were" agrees with its plural subject,
"fans," and its tense combines logically with those of the other verbs in the
sentence.

Explanation for Incorrect Answer D :


There is no error at (D). The prepositional phrase "of... players" correctly modifies
the noun "success."

17
must compete
Because they with a large chain of super-stores that can afford to

charge very low rates for certain items, the owners of small hardware stores know


that you are unlikely to make much profit and may, in fact, go bankrupt. No error


ed 用
ANSWERS AND EXPLANATIONS
er
Corrected Sentence: Because they must compete with a large chain of super-


stores that can afford to charge very low rates for certain items, the owners of
small hardware stores know that they are unlikely to make much profit and may, in
fact, go bankrupt.
st


Explanation for Correct Answer B :
i

The error in this sentence occurs at (B), where there is a shift of pronoun. The


eg

second-person pronoun "you" does not agree with the third-person pronoun "they"
earlier in the sentence (which in turn refers to the plural subject of the sentence,
"owners").


nR


Explanation for Incorrect Answer A :
There is no error at (A). The verb phrase "must compete" agrees with its subject,


U

"they."

Explanation for Incorrect Answer C :


There is no error at (C). The infinitive “to make” modifies the adjective
“unlikely” to produce an appropriate idiom. Furthermore, the adjective “much”
properly modifies the noun “profit.”

Explanation for Incorrect Answer D :


There is no error at (D). The verb phrase "may... go bankrupt," introduced by the
word "and," is properly parallel with the preceding verb phrase "are unlikely...
profit."

Explanation for Incorrect Answer E : There is an error in the sentence.

18

Although science offers the hope of preventing serious genetic diseases, there is

difficult ethical questions raised by the possibility of altering human heredity. No error

file://E:\新建文件夹\a7.htm 2006-11-12
The Official SAT Online Course 页码,11/21

ANSWERS AND EXPLANATIONS


Corrected Sentence: Although science offers the hope of preventing serious
genetic diseases, there are difficult ethical questions raised by the possibility of
altering human heredity.

Explanation for Correct Answer B :


The error in this sentence occurs at (B), where there is a problem with subject-verb
agreement. In this inverted construction, the singular verb "is" does not agree with
its plural subject, "questions."

Explanation for Incorrect Answer A :


There is no error at (A). The preposition "of" and the verbal "preventing" correctly
introduce a phrase that modifies the previous noun, "hope."

Explanation for Incorrect Answer C :


There is no error at (C). The verbal "raised" and the preposition "by" correctly begin
a phrase that modifies the noun "questions."

Explanation for Incorrect Answer D :


There is no error at (D). The noun "possibility" is used correctly as the object of the
preposition "by," and the preposition "of" correctly begins another phrase that
modifies "possibility."


d
Explanation for Incorrect Answer E : There is an error in the sentence.

e 用
er

19
In the nineteenth century, careers in business and law were prestigious, but
t

is
it did not require practitioners to hold college degrees. No error


eg

ANSWERS

用 AND EXPLANATIONS
nR


Corrected Sentence: In the nineteenth century, careers in business and law were
prestigious, but they did not require practitioners to hold college degrees.


Explanation for Correct Answer B :
U

The error in this sentence occurs at (B), where the number of the pronoun is
incorrect. The singular pronoun "it" does not agree with the plural noun "careers,"
to which it refers.

Explanation for Incorrect Answer A :


There is no error at (A). The plural verb "were" agrees with its plural subject,
"careers."

Explanation for Incorrect Answer C :


There is no error at (C). The verb in past tense, "did require," is consistent with the
earlier past-tense verb, "were," and the adverb "not" is the appropriate negative
word.

Explanation for Incorrect Answer D :


There is no error at (D). The verbal "to hold" is the appropriate phrase to use
between the nouns "practitioners" and "college degrees."

Explanation for Incorrect Answer E : There is an error in the sentence.

20

file://E:\新建文件夹\a7.htm 2006-11-12
The Official SAT Online Course 页码,12/21

to improve their
Chess players find that playing against a computer is helpful skills,

even though won No error


no chess-playing computer has yet a championship.

ANSWERS AND EXPLANATIONS


Corrected Sentence: Chess players find that playing against a computer is helpful
in improving their skills, even though no chess-playing computer has yet won a
championship.

Explanation for Correct Answer A :


The error in this sentence occurs at (A), where an improper idiom is used. The
infinitive “to improve” is used in combination with the verb phrase “is helpful,”
where it would be more idiomatic to use the phrase “in improving.”

Explanation for Incorrect Answer B :


There is no error at (B). The plural possessive pronoun "their" is properly used to
refer to the subject, "chess players."


ed
Explanation for Incorrect Answer C :
There is no error at (C). The subordinating conjunction “even though” properly
introduces the subordinate adverbial clause, “even though no chess-playing . . .


championship.” er
Explanation for Incorrect Answer D :


There is no error at (D). The word "won," the past participle of the verb "to win,"
combines with the word "has" to produce the appropriate verb tense.
st


Explanation for Incorrect Answer E : There is an error in the sentence.
i


eg

21


Reaching lengths of twelve inches, banana slugs are the much larger of all the slug
nR


species that inhabit North America. No error


U

ANSWERS AND EXPLANATIONS


Corrected Sentence: Reaching lengths of twelve inches, banana slugs are the
largest of all the slug species that inhabit North America.

Explanation for Correct Answer C :


The error in this sentence occurs at (C), where the comparative term is incorrect.
The phrase "much larger" implies a comparison between only two items, not a
comparison of banana slugs with all other slug species in North America.

Explanation for Incorrect Answer A :


There is no error at (A). The verbal "Reaching" properly introduces a participial
phrase that modifies the noun "slugs."

Explanation for Incorrect Answer B :


There is no error at (B). The plural verb "are" agrees with its plural subject, "slugs."

Explanation for Incorrect Answer D :


There is no error at (D). The plural verb "inhabit" agrees with the plural noun
phrase "all the slug species," and the present tense of "inhabit" is consistent with

file://E:\新建文件夹\a7.htm 2006-11-12
The Official SAT Online Course 页码,13/21

the tense of the earlier verb "are."

Explanation for Incorrect Answer E : There is an error in the sentence.

22
proudly most popular invention, a watch
Maya Madera wore her sister’s for winter

that flashes the temperature in


campers lighted numerals and sends out a loud alarm

pressing a button. No error


when

ANSWERS AND EXPLANATIONS


Corrected Sentence: Maya Madera proudly wore her sister’s most popular
invention, a watch for winter campers that flashes the temperature in lighted
numerals and sends out a loud alarm when one presses a button.

Explanation for Correct Answer D :



The error in this sentence occurs at (D), where modification is improper. The phrase


"pressing a button" cannot logically modify the noun just before it, "alarm," or any

d
other word in the sentence.

re 用

te
Explanation for Incorrect Answer A :
There is no error at (A). The adverb "proudly" (telling how) is used correctly to
modify the verb "wore."


is

Explanation for Incorrect Answer B :


There is no error at (B). The noun phrase "most popular invention" functions


eg

correctly as the direct object of the verb "wore," and the noun "watch" is placed
properly as an appositive to "invention."


Explanation for Incorrect Answer C :
nR

There is no error at (C). The dependent clause "that flashes the temperature"
correctly serves as an adjective describing the watch.


Explanation for Incorrect Answer E : There is an error in the sentence.


U

23

Obviously, Whistler’s paintings, unlike Klee, are conventional in their subject matter.

No error

ANSWERS AND EXPLANATIONS


Corrected Sentence: Obviously, Whistler’s paintings, unlike those of Klee, are
conventional in their subject matter.

Explanation for Correct Answer B :


The error in this sentence occurs at (B), where the comparison is not logical. Works
of art ("Whistler's paintings") are improperly compared with an artist ("Klee").

Explanation for Incorrect Answer A :

file://E:\新建文件夹\a7.htm 2006-11-12
The Official SAT Online Course 页码,14/21

There is no error at (A). The adverb "obviously" correctly modifies "are" and
indicates a high degree of certainty.

Explanation for Incorrect Answer C :


There is no error at (C). The adjective "conventional" is used appropriately after the
linking verb "are," and the preposition "in" correctly introduces a phrase that
modifies "conventional."

Explanation for Incorrect Answer D :


There is no error at (D). The plural pronoun "their" agrees with the plural noun
"paintings," to which it refers.

Explanation for Incorrect Answer E : There is an error in the sentence.

24
A newly formed organization of homeowners and business people have met withthe

transportation department to voice its concerns about plans fora shopping mall in the

community. No error



ed
ANSWERS AND EXPLANATIONS


Corrected Sentence: A newly formed organization of homeowners and business
er
people has met with the transportation department to voice its concerns about


plans for a shopping mall in the community.
t
Explanation for Correct Answer A :


is
The error in this sentence occurs at (A), where there is a problem with subject-verb
agreement. The plural verb "have" agrees with the interrupting nouns
("homeowners and business people") but not with its singular subject,


eg

"organization."


nR

Explanation for Incorrect Answer B :


There is no error at (B). The infinitive "to voice" correctly introduces a verbal
phrase that modifies "met" and indicates the purpose of the meeting.


U

Explanation for Incorrect Answer C :


There is no error at (C). The noun "concerns" serves correctly as the direct object
of the infinitive "to voice," and the preposition "about" is appropriate to begin a
phrase that modifies "concerns."

Explanation for Incorrect Answer D :


There is no error at (D). The noun "plans" serves correctly as the object of the
preposition "about," and the preposition "for" appropriately begins another
prepositional phrase that modifies "plans."

Explanation for Incorrect Answer E : There is an error in the sentence.

25

Although she considers her chemistry research complete, she has heeded her

professor’s advice and is conducting three additional experiments. No error

ANSWERS AND EXPLANATIONS

file://E:\新建文件夹\a7.htm 2006-11-12
The Official SAT Online Course 页码,15/21

Corrected Sentence:

Explanation for Correct Answer E : There is no error in this sentence.

Explanation for Incorrect Answer A :


There is no error at (A). The subordinating conjunction "Although" properly
introduces the subordinate adverbial clause, "she considers... complete."

Explanation for Incorrect Answer B :


There is no error at (B). The adjective "complete" properly modifies the noun
phrase, "her chemistry research."

Explanation for Incorrect Answer C :


There is no error at (C). The singular verb phrase "has heeded" agrees with its
singular subject, "she."

Explanation for Incorrect Answer D :


There is no error at (D). The singular verb phrase "is conducting" agrees with its
singular subject, "she," and properly introduces the second half of the compound
predicate of the main clause.

26 !

d
Princeton University officials first broke with a tradition of awarding honorary degrees

re 用
only to men when they awarded it to author Willa Cather. No error


te

is
ANSWERS AND EXPLANATIONS
Corrected Sentence: Princeton University officials first broke with a tradition of


eg

awarding honorary degrees only to men when they awarded such a degree to
author Willa Cather.


Explanation for Correct Answer D :
nR

The error in this sentence occurs at (D), where the pronoun reference is vague. The
sentence contains no singular noun to which the pronoun "it" (presumably meaning


an honorary degree) can refer.


U

Explanation for Incorrect Answer A :


There is no error at (A). The past tense verb "broke" is used correctly to describe
action already completed, and the preposition "with" is used properly to begin a
phrase modifying "broke."

Explanation for Incorrect Answer B :


There is no error at (B). The preposition "of" is used correctly to begin a phrase
modifying "tradition," and the verbal "awarding" functions properly as the object of
the preposition.

Explanation for Incorrect Answer C :


There is no error at (C). The prepositional phrase "to men" is used correctly to
identify those awarded honorary degrees.

Explanation for Incorrect Answer E : There is an error in the sentence.

27

His love of politics led him to volunteer in local campaigns as well as a job in a

file://E:\新建文件夹\a7.htm 2006-11-12
The Official SAT Online Course 页码,16/21

No error
government office in the state capital.

ANSWERS AND EXPLANATIONS


Corrected Sentence: His love of politics led him to volunteer in local campaigns as
well as to work in a government office in the state capital.

Explanation for Correct Answer D :


The error in this sentence occurs at (D), where there is a flaw in parallelism. The
noun phrase "a job" is not parallel with the earlier verbal phrase "to volunteer."

Explanation for Incorrect Answer A :


There is no error at (A). The past-tense verb "led" correctly describes actions
already completed.

Explanation for Incorrect Answer B :


There is no error at (B). Since the pronoun "him" is the direct object of the verb
"led," the objective case is correct.



Explanation for Incorrect Answer C :

d
There is no error at (C). The connecting phrase "as well" functions correctly to
introduce an additional activity.

re 用
Explanation for Incorrect Answer E : There is an error in the sentence.


te
28


is
Now that Michiko finished the research, she feels reasonably confident about writing


eg

her paper on the rise of the progressive movement in the United States. No error


nR


ANSWERS AND EXPLANATIONS


U

Corrected Sentence: Now that Michiko has finished the research, she feels
reasonably confident about writing her paper on the rise of the progressive
movement in the United States.

Explanation for Correct Answer A :


The error in this sentence occurs at (A), where the tense sequence is incorrect. The
past tense of the verb, "finished," is inappropriate because the adverb "now"
indicates action begun in the past but continuing until the present (action described
correctly by the present perfect tense).

Explanation for Incorrect Answer B :


There is no error at (B). The adjective "confident" is used correctly after the verb
"feels," and the adverb "reasonably" properly modifies "confident."

Explanation for Incorrect Answer C :


There is no error at (C). The prepositional phrase "about writing" functions properly
as an adverb modifying the adjective "confident."

Explanation for Incorrect Answer D :


There is no error at (D). The noun "rise" serves appropriately as the object of the
preposition "on," and the preposition "of" introduces another prepositional phrase
that modifies "rise."

Explanation for Incorrect Answer E : There is an error in the sentence.

file://E:\新建文件夹\a7.htm 2006-11-12
The Official SAT Online Course 页码,17/21

29 usually causes
The condition known as laryngitis the vocal cords and surrounding

thus to move freely No error


tissue to swell, preventing the cords .

ANSWERS AND EXPLANATIONS


Corrected Sentence: The condition known as laryngitis usually causes the vocal
cords and surrounding tissue to swell, thus preventing the cords from moving
freely.

Explanation for Correct Answer C :


The error in this sentence occurs at (C), where the idiom is inappropriate. After the
verbal "preventing" the phrase "to move" is not idiomatic.

Explanation for Incorrect Answer A :



There is no error at (A). The adverb "usually" correctly modifies the verb "causes,"


ed
and that singular verb agrees with its singular subject, "condition."

Explanation for Incorrect Answer B :


There is no error at (B). The connecting word "thus" is used correctly to link a
er
cause (swelling) with its effect (prevention of movement).

Explanation for Incorrect Answer D :


There is no error at (D). The adverb "freely" is used correctly to tell how the cords
st
normally move


Explanation for Incorrect Answer E : There is an error in the sentence.
i
eg



(1) A significant problem all across our state is garbage. (2) Our landfills are
nR

full. (3) It seems that we must either find new sites for landfills or employ other
methods of disposal, like incineration. (4) Unfortunately, there are drawbacks to


every solution that they think of. (5) Polluted runoff water often results from
landfills. (6) With incineration of trash, you get air pollution. (7) People are
criticized for not wanting to live near a polluting waste disposal facility, but


U

really, can you blame them?

(8) Recycling can be an effective solution, but owners of apartment complexes


and other businesses complain that recycling adds to their expenses. (9) Local
governments enjoy the benefits of taxes collected from business and
industry. (10) They tend to shy away from pressuring such heavy contributors
to recycle.

(11) Perhaps those of us being concerned should encourage debate about what
other levels of government can do to solve the problems of waste disposal.
(12) We should make a particular effort to cut down on the manufacture and
use of things that will not decompose quickly. (13) Certainly we should press
individuals, industries, and all levels of government to take responsible action
while we can still see green grass and trees between the mountains of waste.

30
Which of the following would fit most logically between sentences 1 and 2?

(A) A sentence citing examples of states that have used up available landfills
(B) A sentence citing examples of successful alternatives to landfills
(C) A sentence citing the number of new landfills in the state

(D) A sentence citing the number of illnesses blamed on polluted water in the
state

file://E:\新建文件夹\a7.htm 2006-11-12
The Official SAT Online Course 页码,18/21

(E) A sentence citing the average amount of trash disposed of annually by


each person in the state

ANSWERS AND EXPLANATIONS


Explanation for Correct Answer E :
Choice (E) is correct. Information about the amount of trash disposed of annually in
the state explains how the state's landfills came to be full.

Explanation for Incorrect Answer A :


Choice (A) is unsatisfactory because it deals with other states; it does not explain
how landfills in the writer's own state became full.

Explanation for Incorrect Answer B :


Choice (B) is unsatisfactory because what is needed is a sentence about landfills,
not alternatives to landfills.

Explanation for Incorrect Answer C :


Choice (C) is unsatisfactory because the information that would fit most logically is
not about the number of landfills but the amount of garbage that goes into them.

Explanation for Incorrect Answer D :



Choice (D) is unsatisfactory because polluted water, a byproduct of landfills, is


d
discussed later in the essay; what is needed here is information about how the
landfills themselves became full.

re 用

te
31
Which of the following is the best way to phrase the underlined portion of sentence 4
(reproduced below) ?


Unfortunately, there are drawbacks to every solution that they think of.
is

(A) (as it is now)


eg

(B) that was thought of


(C) that they have previously come up with


nR

(D) to which there are proposals


(E) that has been proposed


U

ANSWERS AND EXPLANATIONS


Explanation for Correct Answer E :
Choice (E) is correct. It avoids the error of the original by providing a passive verb
phrase, "has been proposed," and gets rid of the vague pronoun "they."

Explanation for Incorrect Answer A :


Choice (A) is unsatisfactory because it involves the use of a vague pronoun. Neither
the sentence nor its context mentions anyone to whom the pronoun "they" can
logically refer.

Explanation for Incorrect Answer B :


Choice (B) is unsatisfactory because the past tense of the verb does not fit logically
into the context, which describes the situation entirely in the present tense.

Explanation for Incorrect Answer C :


Choice (C) is unsatisfactory because there is nothing in the sentence to which the
pronoun “they” can logically refer. Furthermore, the adverb “previously” does
not make sense in the context of the sentence.

Explanation for Incorrect Answer D :


Choice (D) is unsatisfactory because it results in an illogical sentence. It makes no
sense to speak of proposals that are made to solutions; proposals are made not to
solutions, but to people or groups of people.

file://E:\新建文件夹\a7.htm 2006-11-12
The Official SAT Online Course 页码,19/21

32 Which of the following is the best way to revise and combine sentences 5 and 6
(reproduced below) ?
Polluted runoff water often results from landfills. With incineration of trash, you get
air pollution.

(A) With landfills, polluted runoff water will result, and whereas with
incineration of trash, you get air pollution.
(B) While on the one hand are landfills and polluted runoff water, on the other
hand you have air pollution in the case of incineration of trash.
(C) Landfills often produce polluted runoff water, and trash incineration
creates air pollution.
(D) Landfills and incineration that produce water and air pollution.

(E) Runoff water is from new landfills; from incineration of trash, there is air
pollution.

ANSWERS AND EXPLANATIONS


Explanation for Correct Answer C :
Choice (C) is correct. It combines the two sentences using the same grammatical
structure in both clauses and linking them with the word "and."



d
Explanation for Incorrect Answer A :

re 用
Choice (A) is unsatisfactory because it involves improper coordination: the two
clauses are improperly linked with both "and" and "whereas."
te

Explanation for Incorrect Answer B :
Choice (B) is unsatisfactory because the wordy phrases “While on the one hand . .
. on the other hand” illogically indicate a contrast between the two ideas. Also, the
sentence does not make clear the link between landfills and polluted runoff water; it


is
merely says that both exist.


eg

Explanation for Incorrect Answer D :


Choice (D) is unsatisfactory because it is a fragment; the sentence has no main
verb.


Explanation for Incorrect Answer E :
nR

Choice (E) is unsatisfactory because the verbs "is" and "is" do not indicate causal


connections; also, the word "new" changes the meaning of the first clause, probably
making that statement inaccurate.


U

33
If sentence 8 were rewritten to begin with the clause “Although recycling can be an
effective solution,” the next words would most logically be

(A) and owners of apartment complexes and other businesses complain


(B) yet owners of apartment complexes and other businesses complain
(C) owners of apartment complexes and other businesses complain

(D) mostly owners of apartment complexes and other businesses are


complaining
(E) owners of apartment complexes and other business complained

ANSWERS AND EXPLANATIONS


Explanation for Correct Answer C :
Choice (C) is correct. The main clause completes the sentence by describing a
negative reaction and needs no connecting word to link it with the introductory
dependent clause.

file://E:\新建文件夹\a7.htm 2006-11-12
The Official SAT Online Course 页码,20/21

Explanation for Incorrect Answer A :


Choice (A) is unsatisfactory because the coordinating conjunction "and" is used
incorrectly to join an independent clause ("owners of apartment complexes and
other businesses complain") to an introductory dependent clause (“Although
recycling can be an effective solution”).

Explanation for Incorrect Answer B :


Choice (B) is unsatisfactory because the coordinating conjunction "yet" is used
incorrectly to join an independent clause ("owners of apartment complexes and
other businesses complain") to an introductory dependent clause (“Although
recycling can be an effective solution”). The contrast signalled by "yet" is
unnecessary since "Although" is now at the beginning of the sentence.

Explanation for Incorrect Answer D :


Choice (D) is unsatisfactory because the adverb "mostly" is unnecessary and alters
the meaning of the original.

Explanation for Incorrect Answer E :


Choice (E) is unsatisfactory because the past tense of the verb "complained" is
inconsistent with the present tense of the earlier verb "can be."

34 In context, which of the following is the best way to combine sentences 9 and 10?

(A)

Local governments enjoy the benefits of taxes collected from business and
industry, as they tend to shy away from pressuring such heavy


ed
contributors to recycle.
Because local governments enjoy the benefits of taxes collected from
(B) business and industry, they tend to shy away from pressuring such heavy


contributors to recycle.
er
However, local governments enjoy the benefits of taxes collected from
(C) business and industry, they tend to shy away from pressuring such heavy


contributors to recycle.
In addition to enjoying the benefits of taxes collected from business and
t
(D) industry, local governments tend to shy away from pressuring business


is
and industry into recycling
(E) Local governments, enjoying the benefits of taxes collected from business
and industry, they tend to shy away from pressure to recycle.


eg

ANSWERS AND EXPLANATIONS


nR

Explanation for Correct Answer B :


Choice (B) is correct. The subordinating conjunction "Because" clearly establishes a


cause-effect relationship between the benefits described in the introductory clause
and the tendency described in the main clause.


U

Explanation for Incorrect Answer A :


Choice (A) is unsatisfactory because the conjunction "as" may suggest only that the
two conditions exist at the same time, not that one causes the other.

Explanation for Incorrect Answer C :


Choice (C) is unsatisfactory because it uses incorrect coordination. Two complete
thoughts ("However, local governments enjoy . . . from business and industry" and
"they tend . . . to recycle") are joined by only a comma.

Explanation for Incorrect Answer D :


Choice (D) is unsatisfactory because the phrase "In addition to" does not suggest a
cause-effect relationship.

Explanation for Incorrect Answer E :


Choice (E) is unsatisfactory because the pronoun "they" unnecessarily repeats the
subject of the sentence, "Local governments."

35 Which of the following is the best phrasing for the underlined portion of sentence 11
(reproduced below)?
Perhaps those of us being concerned should encourage debate about what other
levels of government can do to solve the problems of waste disposal.

file://E:\新建文件夹\a7.htm 2006-11-12
The Official SAT Online Course 页码,21/21

(A) (as it is now)


(B) those of us who are concerned
(C) those concerned ones of us

(D) we, being among those who are concerned,

(E) we who are those being concerned

ANSWERS AND EXPLANATIONS


Explanation for Correct Answer B :
Choice (B) is correct. The dependent clause "who are concerned" is an appropriate
idiom to follow the pronoun "us," and the entire phrase uses no unnecessary words.

Explanation for Incorrect Answer A :


Choice (A) is unsatisfactory because, after the pronoun "us," the verbal phrase
"being concerned" is less idiomatic than the dependent clause "who are oncerned."


Explanation for Incorrect Answer C :
Choice (C) is unsatisfactory because it inappropriately separates the pronoun
"those" from the prepositional phrase "of us."


ed
Explanation for Incorrect Answer D :
Choice (D) is unsatisfactory because the phrase "being among those" is not needed.

Explanation for Incorrect Answer E :


r 用
Choice (E) is unsatisfactory because the words "those being" are not necessary.


te

is


eg

Back to Score Report


Copyright © 2006 The College Board. All rights reserved. Privacy Policy Terms of Use Contact Us
nR



U

file://E:\新建文件夹\a7.htm 2006-11-12
The Official SAT Online Course 页码,1/16

Help | Profile | My Organizer | My Bookmarks | Logout

Answers and Explanations

Test Sections Back to Score Report

Section 1 View Answers and Explanations


Section 2 Online - Practice Test #1

Section 3
1
Section 5 Most pioneers ------- this valley on their journey to the West because its rugged
terrain and frequent landslides made it a ------- place for travelers.
Section 6
Section 7 (A) flanked. . fascinating
Section 8 (B) avoided. . necessary
Section 9 (C) encompassed. . curious
Section 10
(D) enjoyed. . troublesome

(E) skirted. . hazardous

ANSWERS AND EXPLANATIONS !



d
Explanation for Correct Answer E :

re
Choice (E) is correct. "Skirted" means avoided, and "hazardous" means dangerous.


If one were to insert these terms into the text, the sentence would read "Most
pioneers skirted this valley on their journey to the West because its rugged terrain
and frequent landslide made it a hazardous place for travelers." Since the word


te
"because" indicates that the activity in the first part of the sentence is directly
affected by the factors mentioned in the second part of the sentence, choice (E)
makes sense: The pioneers avoided the valley as a direct result of their awareness


is
of its dangerous terrain.


eg

Explanation for Incorrect Answer A :


Choice (A) is incorrect. "Flanked" means moved around the side of, and
nR

"fascinating" means alluring. If one were to insert these terms into the text, the
sentence would read "Most pioneers flanked this valley on their journey to the West


because its rugged terrain and frequent landslides made it a fascinating place for
travelers." It is unlikely that settlers would find frequent landslides "fascinating,"
and even if they had, choosing to move around the side of the valley would not


U

have been a reasonable expression of this fascination.

Explanation for Incorrect Answer B :


Choice (B) is incorrect. "Avoided" means kept away from, and "necessary" means
essential. If one were to insert these terms into the text, the sentence would read
"Most pioneers avoided this valley on their journey to the West because its rugged
terrain and frequent landslides made it a necessary place for travelers." Although it
would have been reasonable for pioneers to avoid the dangerous valley, it makes
no sense to say that they avoided it because its hazards made it a "necessary place
for travelers."

Explanation for Incorrect Answer C :


Choice (C) is incorrect. "Encompassed" means enclosed, and "curious," in this
context, means unusual. If one were to insert these terms into the text, the
sentence would read "Most pioneers encompassed this valley on their journey to
the West because its rugged terrain and frequent landslides made it a curious place
for travelers." It is not reasonable to characterize the threat of injury or death as
"curious," and "encompassing," or surrounding, the valley during the pioneers'
journey would not have been a logical result of their discovering the valley's
danger.

Explanation for Incorrect Answer D :


Choice (D) is incorrect. "Enjoyed" means took pleasure in, and "troublesome"
means difficult. If one were to insert these terms into the text, the sentence would
read "Most pioneers enjoyed this valley on their journey to the West because its
rugged terrain and frequent landslides made it a troublesome place for travelers."
Dangerous conditions would certainly make the valley a difficult place for travelers,
but they would almost certainly not find the place pleasurable.

file://E:\新建文件夹\a8.htm 2006-11-12
The Official SAT Online Course 页码,2/16

2 Ballads often praise popular figures who have performed feats that many perceive as
-------, such as defending the poor or resisting ------- authority.
(A) modest. . acceptable
(B) inescapable . . legitimate
(C) insufficient . . overpowering

(D) admirable. . unjust

(E) unbelievable. . tolerable

ANSWERS AND EXPLANATIONS


Explanation for Correct Answer D :
Choice (D) is correct. "Admirable" means deserving approval, and "unjust" means
unfair. If one were to insert these terms into the text, the sentence would read
"Ballads often praise popular figures who have performed feats that many perceive
as admirable, such as defending the poor or resisting unjust authority." Because
the figures mentioned in the first part of the sentence are praised for their "feats,"
or notable deeds, the first missing term will have a positive meaning. The second
missing term, used to describe the type of authority that these figures were praised


for resisting, will have a negative meaning. Since feats such as helping the poor
and resisting unjust authority are seen as admirable and therefore are praised by
balladeers, choice (D) is correct.


r
Explanation for Incorrect Answer A :

ed 用
Choice (A) is incorrect. "Modest" means humble, and "acceptable" means adequate.


te
If one were to insert these terms into the text, the sentence would read "Ballads
often praise popular figures who have performed feats that many perceive as
modest, such as defending the poor or resisting acceptable authority." Resisting


is
"acceptable authority" would not be viewed as a modest, or humble feat, nor would
performers of such modest feats be likely to become "popular figures" to be praised
in a ballad.


eg

Explanation for Incorrect Answer B :


Choice (B) is incorrect. "Inescapable" means unavoidable, and "legitimate" means


approved by law. If one were to insert these terms into the text, the sentence
nR

would read "Ballads often praise popular figures who have performed feats that
many perceive as inescapable, such as defending the poor or resisting legitimate


authority." Resisting "legitimate authority" would not be seen as unavoidable;
therefore, this type of behavior would not be likely to be praised.


U

Explanation for Incorrect Answer C :


Choice (C ) is incorrect. "Insufficient" means inadequate, and something that is
"overpowering" is characterized by overwhelming intensity. If one were to insert
these terms into the text, the sentence would read "Ballads often praise popular
figures who have performed feats that many perceive as insufficient, such as
defending the poor or resisting overpowering authority." Even though figures might
resist "overpowering authority," it is not reasonable to suggest that these figures
would also perform feats that are considered insufficient and that they would be
popular and deserving of praise.

Explanation for Incorrect Answer E :


Choice (E) is incorrect. "Unbelievable" means too improbable for belief, and
"tolerable" means fairly good. If one were to insert these terms into the text, the
sentence would read "Ballads often praise popular figures who have performed
feats that many perceive as unbelievable, such as defending the poor or resisting
tolerable authority." Resisting tolerable authority would hardly be seen as an
unbelievable feat and, if it were, those performing the feat would not be popular
figures.

3 The serious purpose of the paper-airplane flying contest, which attracted many novel
and sometimes truly ------- entries, was to determine whether any -------
aerodynamic designs could be discovered.

(A) unorthodox . . conventional

file://E:\新建文件夹\a8.htm 2006-11-12
The Official SAT Online Course 页码,3/16

(B) bizarre. . revolutionary


(C) derivative. . imaginative

(D) mundane . . predictable

(E) ungainly. . aesthetic

ANSWERS AND EXPLANATIONS


Explanation for Correct Answer B :
Choice (B) is correct. "Bizarre" means oddly eccentric, and "revolutionary" means
marked by radical change. If one were to insert these terms into the text, the
sentence would read "The serious purpose of the paper-airplane flying contest,
which attracted many novel and sometimes truly bizarre entries, was to determine
whether any revolutionary aerodynamic designs could be discovered." The structure
of the sentence indicates that the correct response is the only pairing where the
first missing term parallels the word "novel," or new, and makes sense in the first
part of the sentence without contradicting the meaning of the second missing word.
Since it is logical that the organizers of the contest hoped to discover a
revolutionary design from among the entries that were so new that they seemed
bizarre, choice (B) makes sense.

Explanation for Incorrect Answer A :




Choice (A) is incorrect. "Unorthodox" means extraordinary, and "conventional"

ed
means ordinary. If one were to insert these terms into the text, the sentence would
read "The serious purpose of the paper-airplane flying contest, which attracted


many novel and sometimes truly unorthodox entries, was to determine whether any
conventional aerodynamic designs could be discovered." The terms "unorthodox"

r
and "conventional" have opposite meanings. Therefore, the organizers could not
have expected to find a conventional, or traditional, design by choosing from


te
among novel and unconventional entries.


Explanation for Incorrect Answer C :
is
Choice (C) is incorrect. "Derivative" means unoriginal, and "imaginative" means
creative. If one were to insert these terms into the text, the sentence would read
"The serious purpose of the paper-airplane flying contest, which attracted many


eg

novel and sometimes truly derivative entries, was to determine whether any
imaginative aerodynamic designs could be discovered." If the entries were based on
already established designs, the contest would not have been likely to yield many


imaginative or original designs, so choice (C) does not make sense.
nR

Explanation for Incorrect Answer D :


Choice (D) is incorrect. "Mundane" means ordinary, and "predictable" means easily
foreseen, or routine. If one were to insert these terms into the text, the sentence
would read "The serious purpose of the paper-airplane flying contest, which


U

attracted many novel and sometimes truly mundane entries, was to determine
whether any predictable aerodynamic designs could be discovered." It would not
make sense for the organizers of the contest to try to discover predictable designs,
since the point of discovery is to find something new; ordinary, entries are unlikely
to be novel. For these reasons, choice (D) does not make sense.

Explanation for Incorrect Answer E :


Choice (E) is incorrect. "Ungainly" means graceless and unwieldly, and "aesthetic"
means tastefully attractive. If one were to insert these terms into the text, the
sentence would read "The serious purpose of the paper-airplane flying contest,
which attracted many novel and sometimes truly ungainly entries, was to determine
whether any aesthetic aerodynamic designs could be discovered." The terms
"ungainly" and "aesthetic" have opposite meaning, therefore a contest that
attracted graceless entries would not be likely to yield tastefully attractive designs.

4 Carson presents her case so strongly and logically that only the prejudiced or the
------- will attempt to ------- her.

(A) impartial. . defy


(B) doubtful. . champion
(C) gullible. . believe

(D) obstinate . . contradict

file://E:\新建文件夹\a8.htm 2006-11-12
The Official SAT Online Course 页码,4/16

(E) irrational . . follow

ANSWERS AND EXPLANATIONS


Explanation for Correct Answer D :
Choice (D) is correct. "Obstinate" means stubborn, and "contradict" means to
assert the opposite of. If one were to insert these terms into the text, the sentence
would read "Carson presents her case so strongly and logically that only the
prejudiced or the obstinate will attempt to contradict her." The sentence is
structured so that the first missing word will be a negative one that parallels
"prejudiced," or having a preconceived opinion, and the second missing word will be
something that someone with the negative characteristic described by the first
missing word would attempt to do. Choice (D) makes sense, because only a
stubborn person would attempt to contradict Carson's strong and logical case.

Explanation for Incorrect Answer A :


Choice (A) is incorrect. "Impartial" means fair, and "defy" means to challenge. If
one were to insert these terms into the text, the sentence would read "Carson
presents her case so strongly and logically that only the prejudiced or the impartial
will attempt to defy her." Since an impartial, person would not be likely to challenge


a strong and logical argument, choice (A) does not make sense.

Explanation for Incorrect Answer B :


d
Choice (B) is incorrect. "Doubtful" means undecided or skeptical, and "to champion"
means to support. If one were to insert these terms into the text, the sentence
would read "Carson presents her case so strongly and logically that only the

re 用
prejudiced or the doubtful will attempt to champion her. Someone who is doubtful
about the strength of Carson's case is not likely to support her.


te
Explanation for Incorrect Answer C :
Choice (C) is incorrect. "Gullible" means easily fooled, and "believe" means to
accept as true. If one were to insert these terms into the text, the sentence would


is
read "Carson presents her case so strongly and logically that only the prejudiced or
the gullible will attempt to believe her." If only someone who can be easily fooled is
likely to believe Carson's case, then her case cannot be described as strong and


eg

logical.

Explanation for Incorrect Answer E :


Choice (E) is incorrect. "Irrational" means not led by reason or logic, and "follow,"
in this context, means to agree with or adhere to. If Carson presents her case
nR

strongly and logically, it would not make sense that someone who is irrational, or


illogical, would necessarily tend to agree with her.


U

5
Over the years the anthropologist’s opinions had -------: he refused to tolerate new
ideas and nothing could change his mind.

(A) digressed
(B) proliferated
(C) ossified

(D) germinated

(E) incubated

ANSWERS AND EXPLANATIONS


Explanation for Correct Answer C :
Choice (C) is correct. "Ossified" means became rigidly set. If one were to insert this
term into the text, the sentence would read "Over the years the anthropologist's
opinions had ossified: he refused to tolerate new ideas and nothing could change
his mind." Since the colon indicates that the phrases on either side of it have the
same meaning, the word "ossified" makes sense: by ignoring new ideas, the
anthropologist has allowed his opinions to become rigid and unchanging.

file://E:\新建文件夹\a8.htm 2006-11-12
The Official SAT Online Course 页码,5/16

Explanation for Incorrect Answer A :


Choice (A) is incorrect. "Digressed" means strayed from the subject. If one were to
insert this term into the text, the sentence would read "Over the years the
anthropologists's opinions had digressed: he refused to tolerate new ideas and
nothing could change his mind." Since the colon in this sentence indicates that the
missing term should mean the same as mentally rigid and inflexible, "digressed"
does not make sense here.

Explanation for Incorrect Answer B :


Choice (B) is incorrect. "Proliferated" means multiplied rapidly. If one were to insert
this term into the text, the sentence would read "Over the years the
anthropologist's opinions had proliferated: he refused to tolerate new ideas and
nothing could change his mind." The colon in this sentence indicates that the
missing term should reflect the anthropologist's mental inflexibility, but the term
"proliferated" instead suggests that his opinions rapidly multiplied.

Explanation for Incorrect Answer D :


Choice (D) is incorrect. "Germinated" means sprouted. If one were to insert this
term into the text, the sentence would read "Over the years the anthropologist's
opinions had germinated: he refused to tolerate new ideas and nothing could
change his mind." The term "germinated" does not make sense in this context
because the anthropologist was not open to new ideas.

Explanation for Incorrect Answer E :


Choice (E) is incorrect. "Incubated" means formed or considered slowly. If one were


to insert this term into the text, the sentence would read "Over the years the
anthropologist's opinions had incubated: he refused to tolerate new ideas and
nothing could change his mind." "Germination" does not make sense here because


the anthropologist "refused to tolerate new ideas."

ed 用
er
As the first ------- of the political campaign, the senator unleashed a spirited verbal
attack on her leading opponent.

(A) salvo


t
(B) encore


is
(C) palliative

(D) concession


eg

(E) demurral


nR

ANSWERS AND EXPLANATIONS


Explanation for Correct Answer A :
Choice (A) is correct. A "salvo" is a forceful assault. If one were to insert this term
into the text, the sentence would read "As the first salvo of the political campaign,


U

the senator unleashed a spirited verbal attack on her leading opponent." It makes
sense that the senator would launch a verbal "assault" on her opponent at the
opening of a political campaign.

Explanation for Incorrect Answer B :


Choice (B) is incorrect. An "encore" is an additional performance at an audience's
request . If one were to insert this term into the text, the sentence would read "As
the first encore of the political campaign, the senator unleashed a spirited verbal
attack on her leading opponent." It does not make sense to say that the senator led
off a political campaign with an additional performance.

Explanation for Incorrect Answer C :


Choice (C) is incorrect. A "palliative" is something that soothes. If one were to
insert this term into the text, the sentence would read "As the first palliative of the
political campaign, the senator unleashed a spirited verbal attack on her leading
opponent." It does not make sense to say that a politician would "attack" her
opponent with something meant to soothe.

Explanation for Incorrect Answer D :


Choice (D) is incorrect. A "concession" is something yielded or granted. If one were
to insert this term into the text, the sentence would read "As the first concession of
the political campaign, the senator unleashed a spirited verbal attack on her leading
opponent." It does not make sense to say that a politician attacked her opponent
by yielding or granting something to the opponent.

file://E:\新建文件夹\a8.htm 2006-11-12
The Official SAT Online Course 页码,6/16

Explanation for Incorrect Answer E :


Choice (E) is incorrect. A "demurral" is a mild expression of disagreement. If one
were to insert this term into the text, the sentence would read "As the first
demurral of the political campaign, the senator unleashed a spirited verbal attack on
her leading opponent." This sentence makes no sense because a demurral would
not be described as a spirited attack.

The following passages are taken from testimony given before congressional
committees about how government funding affects the arts. The author of the first
passage is a writer and radio entertainer; the author of the second passage is a
novelist and critic.

Passage 1
All governments have honored artists when
they are
old and saintly and successful and almost dead,
but twenty-
five years ago Congress decided to boldly and
blindly
support the arts—support the act of creation !

Line
itself—and

ed
to encourage artists who are young and


5
dangerous and
r
unknown and very much alive. This courageous


te
legislation
has changed American life.


Forty years ago, if American men or women
is

meant to


eg

have artistic careers, they got on the train to


New York.


Today, you can be a violinist in North Carolina,
10
nR

a writer in


Iowa, a painter in Utah. This is a small and
lovely revolu-


tion that the National Endowment for the Arts
U

(NEA) has
helped to bring about. The Endowment has
fostered thou-
sands of artistic works—many of which will
outlive you
and me—but even more important, the
15
Endowment has
changed how we think about the arts. Today,
no American
family can be secure against the danger that
one of its chil-
dren may decide to become an artist.
I grew up in a family that never attended
concerts or
museums, never bought books. I never
20
imagined that a
person could be a writer.

file://E:\新建文件夹\a8.htm 2006-11-12
The Official SAT Online Course 页码,7/16

Twice in my life, at crucial times, grants


from the
Endowment made it possible for me to be a
writer. The
first, in 1969, arrived when I was young, broke,
married
with a baby, living on very little cash and a big
25
vegetable
garden. I was writing for The New Yorker at the
time, but
they weren’t aware of it. I wrote every
morning and every
night. I often had fantasies of finding a patron—
a beggar
would appear at my door, I’d give him an egg
salad sand-
wich, and suddenly he’d turn into a man in a


30
pinstripe suit,
Prince Bob from a philanthropic foundation. But
instead, I

d
got a letter offering me a job for one month in
the Writers
re 用
in the Schools program in Minneapolis, funded


e
by the
NEA, which sent young writers into the schools
st


to read
35 and teach.
i


In 1974 a grant from the NEA enabled me
eg

and my col-


leagues at a public radio station to start a new
radio series.
nR

By the time the show became popular, the


Endowment had
vanished from the credits, its job done.


U

When you’re starting out, it seems like


40
nobody wants
to give you a dime, and then, when you have
big success
and have everything you could ever want,
people can’t do
enough for you. The Endowment is there at the
beginning,
and that’s the beauty of it.

Passage 2
I love my country’s government for its
45
attempt in a pre-
carious world to sustain a peaceful order in
which work can
be done and happiness can be pursued, not for
the good of

file://E:\新建文件夹\a8.htm 2006-11-12
The Official SAT Online Course 页码,8/16

the state, but in a state that exists for our


good.
I love my government not least for the
extent to which
it leaves me alone. My personal ambition has
50
been simply
to live by the work of my pen. This is not a very
fastidious
ambition. If I were aware of large amounts of
federal money
available to purveyors of the written word, I
would attempt
to gain access to it and hope to please the
administrators of
this fund as I hope to please magazine editors
55
and book
buyers.


But I would rather have as my patron a host
of anony-


mous citizens digging into their own pockets for

ed
the price


of a book or a magazine than a small body of
enlightened
er

and responsible people administering public
60
funds. I would
t

rather chance my personal vision of the truth
is

striking home
here and there in the chaos of publication that


eg

exists than
attempt to filter it through a few sets of official,
honorable,

nR

and public-spirited scrutinizers.


65

The realms of scientific research are now
inextricably


U

involved with government funding. Can we fear


that the
humanities might become similarly dependent?
If I try to
think of who in the last century has most
brilliantly illumi-
nated our sense of humanity, which I take to be
the end
purpose of the humanities, I think of Freud and
70
Kafka, of
Proust and Joyce, of Whitman, of Henry James.
I wonder
how many of these brave, strange, stubborn
spirits would
have wanted subsidies from their governments.
How can public-salaried officials not think in
terms of
75 respectability, of social optimism, of broad and

file://E:\新建文件夹\a8.htm 2006-11-12
The Official SAT Online Course 页码,9/16

uncontro-
versial appeal? How can legislators, asked to
vote tax money
away, not begin to think of guidelines that
insidiously edge
toward censorship?
If government money becomes an
increasingly impor-
tant presence in the financing of the
80
humanities, is there
a danger, I respectfully ask, of humanists
becoming
politicians?
7 The argument in Passage 1 is supported primarily by

(A) a theory of how art is created


(B) the author’s personal experiences


(C) examples of renowned artists

(D) evidence of the harmful effects of arts funding


ed
(E) emotional appeals to uphold the rights of citizens

ANSWERS AND EXPLANATIONS



er

Explanation for Correct Answer B :
Choice (B) is correct. The author tells about being given two grants that "made it
possible for me to be a writer."
t

is


eg

Explanation for Incorrect Answer A :


Choice (A) is incorrect. The author is concerned with discussing the effect of


funding on artists, not with discussing how art is created.
nR

Explanation for Incorrect Answer C :


Choice (C) is incorrect. The author of Passage 1 does not name any renowned
artists.


U

Explanation for Incorrect Answer D :


Choice (D) is incorrect. Far from providing evidence of harmful effects of arts
funding, the author expresses gratitude for the positive effects of funding.

Explanation for Incorrect Answer E :


Choice (E) is incorrect. The author does not suggest that citizens have a right to
arts funding.

8 Which is a likely response by the author of Passage 2 to the description of artists as


“dangerous” (line 5) in Passage 1?

(A) Practical concerns rarely prevent artists from speaking out on


controversial issues.
(B) Artists should not be permitted to undermine the values of their society.

(C) Artists will cease taking risks if they come to depend on government
money.
(D) The future of the arts in the United States depends on whether young
artists can continue to shock the public.
(E) Experienced artists know better how to excite the public than do young
artists.

ANSWERS AND EXPLANATIONS

file://E:\新建文件夹\a8.htm 2006-11-12
The Official SAT Online Course 页码,10/16

Explanation for Correct Answer C :


Choice (C) is correct. The author of Passage 2 suggests that being offered large
amounts of government money could cause artists to "hope to please the
administrators of this fund" who "think in terms of respectability." In other words,
being offered government money would make artists less likely to take artistic risks.

Explanation for Incorrect Answer A :


Choice (A) is incorrect. The author of Passage 2 indicates that dependence on public
funding tends to make artists too eager to please "official, honorable" fund
administrators.

Explanation for Incorrect Answer B :


Choice (B) is incorrect. By indicating that censorship of artists is unacceptable, the
author of Passage 2 implies that freedom of artistic expression is more important
than such values as "respectability" or "broad and uncontroversial appeal."

Explanation for Incorrect Answer D :


Choice (D) is incorrect. There is no evidence that the author of Passage 2 believes
art must be shocking in order to be good.

Explanation for Incorrect Answer E :


Choice (E) is incorrect. The author of Passage 2 makes no comparison between the


effect on the public of the works of experienced artists and those of young artists.


ed 用
9 In lines 10-11, Passage 1, the author refers to North Carolina, Iowa, and Utah to

(A) prove that certain environments support creativity better than others
er

(B) support the argument by mentioning how few exceptions there are to it
(C) show that opportunities for artistic success are now widely available
st


(D) suggest that legislators should listen to their constituents

(E) offer a parallel between arts funding and democracy


i


eg

ANSWERS AND EXPLANATIONS


nR

Explanation for Correct Answer C :


Choice (C) is correct. The author uses these examples—states in several parts of


the United States—to show that opportunities for artistic success are less limited
than they once were.


U

Explanation for Incorrect Answer A :


Choice (A) is incorrect. The examples point out that many different kinds of places
support artistic creativity, not that such creativity is limited to certain places.

Explanation for Incorrect Answer B :


Choice (B) is incorrect. The examples support the author's argument by
exemplifying the variety encouraged by government grants. They are not presented
as exceptions to the argument.

Explanation for Incorrect Answer D :


Choice (D) is incorrect. The author of Passage 1 doesn't discuss the relationship
between legislators and their constituents.

Explanation for Incorrect Answer E :


Choice (E) is incorrect. The author of Passage 1 does not comment on any
relationship between arts funding and the democratic form of government.

10 The statement in16-18, Passage 1 (“Today . . . artist”) is best described as an


example of

(A) an ironic comment

file://E:\新建文件夹\a8.htm 2006-11-12
The Official SAT Online Course 页码,11/16

(B) an emotional plea


(C) a moral pronouncement

(D) a definition of a key concept

(E) a generalization supported by research

ANSWERS AND EXPLANATIONS


Explanation for Correct Answer A :
Choice (A) is correct. An "ironic comment" is one that is humorous because it
presents an "incongruity," a situation that is unexpected in a particular context.

Explanation for Incorrect Answer B :


Choice (B) is incorrect. The tone of the sentence is humorous, rather than pleading
or emotional.

Explanation for Incorrect Answer C :


Choice (C) is incorrect. A "moral pronouncement" is a formal or authoritative
statement about distinctions between right and wrong; it is unlikely to be humorous


or to include incongruity.

Explanation for Incorrect Answer D :


d
Choice (D) is incorrect. The sentence does not present a definition of any kind.

re
Explanation for Incorrect Answer E :


Choice (E) is incorrect. Although the likelihood that a family's child might become
an artist could be a subject for research, the author does not mention any such
research.


te

is
11
The “man in a pinstripe suit” (line 30, Passage 1) is


eg

(A) a fastidious bureaucrat


(B) a character in a novel


(C) a famous writer
nR

(D) an anonymous critic


(E) an imagined benefactor


U

ANSWERS AND EXPLANATIONS


Explanation for Correct Answer E :
Choice (E) is correct. A "benefactor" is one who gives benefits or gifts, and a
"fantasy," like the man in the pinstripe suit, is something imagined.

Explanation for Incorrect Answer A :


Choice (A) is incorrect. Although the pinstripe suit could be seen as a stereotype
that suggests a fastidious bureaucrat, a government administrator with high
standards, the man is better described as an imagined benefactor in the context of
the sentence.

Explanation for Incorrect Answer B :


Choice (B) is incorrect. The man is described as a character in a fantasy about
funding, not in a novel.

Explanation for Incorrect Answer C :


Choice (C) is incorrect. The man is described as being from a philanthropic
foundation, a group that distributes funds; he is not described as a writer.

Explanation for Incorrect Answer D :


Choice (D) is incorrect. A man from a philanthropic foundation is more likely to
distribute funds rather than to serve as a critic.

file://E:\新建文件夹\a8.htm 2006-11-12
The Official SAT Online Course 页码,12/16

12 Lines 40-43 (“When . . . for you”) suggest that the author of Passage 1 would agree
with which of the following observations?
(A) Those who never succeed value success most.
(B) The love of money is the root of all evil.
(C) Nothing succeeds like success.

(D) Make a virtue of necessity.

(E) Time is money.

ANSWERS AND EXPLANATIONS


Explanation for Correct Answer C :
Choice (C) is correct. The sentence indicates that artists who are already successful
are likely to continue to be rewarded: being successful leads to more success.
Another way of saying this is "Nothing succeeds like success."

Explanation for Incorrect Answer A :


Choice (A) is incorrect. The sentence has to do with others' reactions to artists'
success, not with the value artists place on success.


d
Explanation for Incorrect Answer B :

re
Choice (B) is incorrect. The sentence does not state or suggest anything about the
relationship between money and evil.



te
Explanation for Incorrect Answer D :
Choice (D) is incorrect. The author does not suggest that artists should look for the
advantages of having no funding.


is
Explanation for Incorrect Answer E :
Choice (E) is incorrect. The sentence has nothing to do with the value of artists'


time.
eg


nR

13


The author of Passage 2 would most likely criticize the author of Passage 1 on the
grounds that


(A) humor detracts from the seriousness of the issue being discussed
U

(B) public funding is often given to artists who do not need it


(C) it is invalid to assume that artists can also be teachers

(D) taxes will be too high if the government supports the arts

(E) public funding is just as likely to inhibit artists as it is to encourage them

ANSWERS AND EXPLANATIONS


Explanation for Correct Answer E :
Choice (E) is correct. The author of Passage 2 expresses concern that artists might
be tempted to try to please those who provide grants. Such temptation, it can be
inferred, might inhibit artists by steering them away from controversial projects.

Explanation for Incorrect Answer A :


Choice (A) is incorrect. Although the author of Passage 1 uses humor more freely in
making an argument than does the author of Passage 2, there is no indication that
the author of Passage 2 is critical of using humor.

Explanation for Incorrect Answer B :


Choice (B) is incorrect. The author of Passage 2 is not concerned with artists' need
but with their tendency to try to please funding administrators.

file://E:\新建文件夹\a8.htm 2006-11-12
The Official SAT Online Course 页码,13/16

Explanation for Incorrect Answer C :


Choice (C) is incorrect. The author of Passage 2 does not comment at all on the
ability of artists to be teachers.

Explanation for Incorrect Answer D :


Choice (D) is incorrect. The author of Passage 2 does not express concern about the
effect of funding for the arts on taxes.

14 In line 57, Passage 2, “host” most nearly means

(A) large number


(B) sustaining organism
(C) provider

(D) proprietor

(E) sponsor

ANSWERS AND EXPLANATIONS


Explanation for Correct Answer A :


Choice (A) is correct. It would take a "large number" of such citizens to support the
author.


d
Explanation for Incorrect Answer B :

re 用
Choice (B) is incorrect. The word "host" is used to mean a "large number," not an
organism.


te
Explanation for Incorrect Answer C :
Choice (C) is incorrect. Although a "host" would, in a sense, be a "provider" for the


is
author, it is clear that the author is speaking of a large number of people.

Explanation for Incorrect Answer D :


eg

Choice (D) is incorrect. This term does not make sense in the context of the
sentence: the author's patron wouldn't be a "proprietor" of anonymous citizens.


Explanation for Incorrect Answer E :
nR

Choice (E) is incorrect. In the context of this sentence, it does not make sense to
say that the author's patron is a sponsor of anonymous citizens.



U

15

The question in lines 66-67 suggests that the author of Passage 2 believes that
federal funding of scientific research

(A) encourages deceit


(B) undermines autonomy
(C) encourages an unhealthy competitiveness

(D) develops a superficial sense of loyalty

(E) spends public money under false pretenses

ANSWERS AND EXPLANATIONS


Explanation for Correct Answer B :
Choice (B) is correct. Funding that deprives the recipient of independence
"undermines autonomy," or weakens one's ability to make one's own decisions.

Explanation for Incorrect Answer A :


Choice (A) is incorrect. The context doesn't suggest that deceit is involved in
funding in any way.

file://E:\新建文件夹\a8.htm 2006-11-12
The Official SAT Online Course 页码,14/16

Explanation for Incorrect Answer C :


Choice (C) is incorrect. Nothing in the passage suggests that a lack of independence
fosters competitive behavior.

Explanation for Incorrect Answer D :


Choice (D) is incorrect. The context does not concern researchers' feelings of loyalty
but their dependence on government funding.

Explanation for Incorrect Answer E :


Choice (E) is incorrect. The author's remarks about scientific research have nothing
to do with the possibility of misrepresentation.

16 The author of Passage 2 most likely thinks that the individuals named in lines 70-71
would have
(A) supported the idea of providing artists with government funding

(B) avoided writing about controversial topics if doing so brought them more
funding
(C) been even more influential if they had received government funding

(D) embraced the romantic image of the starving artist

(E) refused to submit their creativity to outside control

ANSWERS AND EXPLANATIONS !



d
Explanation for Correct Answer E :

e
Choice (E) is correct. Since the author suggests in lines 65-67 that government


funding may limit independence, it can be inferred that these individuals would not
er
want to give control of their creativity to anyone.


t

is
Explanation for Incorrect Answer A :
Choice (A) is incorrect. The author questions whether these individuals would want


eg

government funding themselves but does not suggest anything about their opinion
of government funding for other artists.


Explanation for Incorrect Answer B :
Choice (B) is incorrect Since the context questions whether these individuals would
nR

have wanted government funding, it is unlikely that the author would suspect them


of avoiding controversy in order to get money.

Explanation for Incorrect Answer C :


Choice (C) is incorrect. There is no indication in Passage 2 that government funding
U

would have made these very influential individuals even more influential.

Explanation for Incorrect Answer D :


Choice (D) is incorrect. The fact that the individuals did not receive government
funding does not necessarily mean that they would have thought the image of
starvation to be romantic.

17
The final sentence of Passage 2 serves to

(A) emphasize the moral dilemmas that artists face when selling their work
(B) indicate why artists are so often in need of finan-cial support

(C) suggest that the public should not have to subsidize the art preferred by
bureaucrats
(D) warn of the likelihood of artistic compromise

(E) link arts funding in the United States with other social programs

ANSWERS AND EXPLANATIONS


Explanation for Correct Answer D :
Choice (D) is correct. It can be inferred that if humanists became "politicians," that
is, if artistic decisions were made on the basis of political considerations, artistic

file://E:\新建文件夹\a8.htm 2006-11-12
The Official SAT Online Course 页码,15/16

values would be compromised, or endangered.

Explanation for Incorrect Answer A :


Choice (A) is incorrect. In the passage, no mention has been made of the decisions
artists face when selling their work to such people as magazine editors and book
buyers.

Explanation for Incorrect Answer B :


Choice (B) is incorrect. The question is about the effects of government financial
support, not the reasons why it is needed.

Explanation for Incorrect Answer C :


Choice (C) is incorrect. The question, like the passage, is primarily concerned with
the relationship between government funding and artists, not with the involvement
of the public.

Explanation for Incorrect Answer E :


Choice (E) is incorrect. There is no mention in the passage of the relationship
between funding for the arts and other social programs.

18
would most likely question? !
Which of the following is an assumption in Passage 2 that the author of Passage 1


d
(A) Public funding of the arts increases the danger of censorship.

re
(B) Artistic creation should not involve taking financial risks.


(C) Artists appreciate financial support more if they earn it after suffering
hardships.


te
(D) Administrators of arts funding have higher artistic standards than the
general public.
(E) Democracy is conducive to artistic self-expression.


is

ANSWERS AND EXPLANATIONS


eg

Explanation for Correct Answer A :


Choice (A) is correct. Passage 2 suggests that legislators who make funding


decisions might use guidelines that "insidiously edge toward censorship." Passage 1
nR

expresses no concern about any negative effects of funding on artists.



U

Explanation for Incorrect Answer B :


Choice (B) is incorrect. Neither author presents an opinion about whether artistic
creation should be free of financial risk.

Explanation for Incorrect Answer C :


Choice (C) is incorrect. The author of Passage 2 is concerned with the effect of
government funding on artists' artistic choices, not with the reasons artists
appreciate being funded.

Explanation for Incorrect Answer D :


Choice (D) is incorrect. Passage 2 doesn't describe the artistic standards that
administrators of arts funding have, though the passage does suggest that those
standards, whatever they are, may be more conservative than those of the book-
buying public.

Explanation for Incorrect Answer E :


Choice (E) is incorrect. Neither author expresses an opinion about the effect of
democracy on artistic self-expression.

19
Which of the following situations would support the position taken in Passage 1 and
provide examples contrary to the argument in Passage 2?

I. A federally sponsored photographer displayed an innovative collection of photographs

file://E:\新建文件夹\a8.htm 2006-11-12
The Official SAT Online Course 页码,16/16

that offended both the sponsor and a large segment of the public.

II. The most original works of a certain brilliant composer were those commissioned by
kings.

III. A theatrical troupe from Harlem achieved prominence by drawing large audiences
from its local community.

(A) I only
(B) III only
(C) I and II only

(D) II and III only

(E) I, II, and III

ANSWERS AND EXPLANATIONS


Explanation for Correct Answer C :
Choice (C) is correct. In both situation I and situation II, governments support acts
of creation; in each case, artistic expression remains unaffected.

Explanation for Incorrect Answer A : !



d
Choice (A) is incorrect. Situation I supports the praise of government funding in
Passage 1 and counters the argument in Passage 2 that funding can hinder
creativity. However, both situations that meet the criteria must be included in the
correct answer.

Explanation for Incorrect Answer B : re 用



te
Choice (B) is incorrect. Since situation III does not have anything to do with
government funding, it is irrelevant to the discussions in Passage 1 and Passage 2.


is
Explanation for Incorrect Answer D :
Choice (D) is incorrect. Whereas situation II supports the position taken in Passage
1 in favor of arts funding and provides an example contrary to the argument in


eg

Passage 2 that funding may inhibit artists, situation III has nothing to do with
funding.


Explanation for Incorrect Answer E :
nR

Choice (E) is incorrect. Although situations I and II support the position taken in
Passage 1 and provide examples contrary to the argument in Passage 2, situation


III does not meet these criteria and so it cannot be included in the correct answer.


U

Back to Score Report

Copyright © 2006 The College Board. All rights reserved. Privacy Policy Terms of Use Contact Us

file://E:\新建文件夹\a8.htm 2006-11-12
The Official SAT Online Course 页码,1/12

Help | Profile | My Organizer | My Bookmarks | Logout

Answers and Explanations

Test Sections Back to Score Report

Section 1 View Answers and Explanations


Section 2 Online - Practice Test #1

Section 3
1 A community college charges an activity fee of per student and has a student
Section 5
body of students.
Section 6
If every student pays the fee, what is the total amount in activity fees collected from
Section 7 the students?
Section 8
(A)
Section 9
Section 10 (B)

(C)

(D)

(E)



ed
ANSWERS AND EXPLANATIONS


Explanation for Correct Answer D :
Choice (D) is correct. To find the total amount of money paid by the
r
students, you multiply the number of students by the amount each student


te
pays:


is


eg

Explanation for Incorrect Answer A :


Choice (A) is not correct. See the explanation for the correct response (D).


Explanation for Incorrect Answer B :
Choice (B) is not correct. See the explanation for the correct response (D).
nR


Explanation for Incorrect Answer C :
Choice (C) is not correct. See the explanation for the correct response (D).


U

Explanation for Incorrect Answer E :


Choice (E) is not correct. See the explanation for the correct response (D).

In the figure above, and


If what is the value of

(A)

(B)

(C)

(D)

file://E:\新建文件夹\a9.htm 2006-11-12
The Official SAT Online Course 页码,2/12

(E)

ANSWERS AND EXPLANATIONS


Explanation for Correct Answer C :
Choice (C) is correct. Since and the measure of is it
follows that Since and the measure of is
it follows that

Explanation for Incorrect Answer A :


Choice (A) is not correct. This is the value of

Explanation for Incorrect Answer B :


Choice (B) is not correct. See the explanation for the correct response (C).

Explanation for Incorrect Answer D :


Choice (D) is not correct. See the explanation for the correct response (C).

Explanation for Incorrect Answer E :


Choice (E) is not correct. See the explanation for the correct response (C).


d
3

re 用

te

is


eg


nR


Some common nail sizes and their corresponding lengths are shown in the table


above. If nail sizes from 2d up to 10d increase by a constant length for each increase
U

of 1d in size, what would be the length, in inches, of a 6d nail?

(A)

(B)

(C)

(D)

(E)

ANSWERS AND EXPLANATIONS


Explanation for Correct Answer C :
Choice (C) is correct. Comparing the first two entries on the chart, you can

see that for a increase in nail size, the length increases by inch. To

get to the length of a nail from the given length of a nail, there is a

increase in size, and so the length must increase by inch.

file://E:\新建文件夹\a9.htm 2006-11-12
The Official SAT Online Course 页码,3/12

Therefore, the length of a nail is inches.

Explanation for Incorrect Answer A :


Choice (A) is not correct. See the explanation for the correct response (C).

Explanation for Incorrect Answer B :


Choice (B) is not correct. See the explanation for the correct response (C).

Explanation for Incorrect Answer D :


Choice (D) is not correct. See the explanation for the correct response (C).

Explanation for Incorrect Answer E :


Choice (E) is not correct. See the explanation for the correct response (C).

4 In a sequence of numbers, the first number is and each number after the first is
more than times the preceding number. What is the fourth number in the
sequence?


(A)

(B)


d
(C)

(D)

(E)

re 用

te
ANSWERS AND EXPLANATIONS


is
Explanation for Correct Answer D :
Choice (D) is correct. If the first number in the sequence is then following
the stated rule, the second number is The third number is


eg

and the fourth is


nR


Explanation for Incorrect Answer A :
Choice (A) is not correct. This is the number you get if you think the fourth
term is However, you should plug the preceding number of the


U

sequence into this expression, not the number of the term.

Explanation for Incorrect Answer B :


Choice (B) is not correct. See the explanation for the correct response (D).

Explanation for Incorrect Answer C :


Choice (C) is not correct. See the explanation for the correct response (D).

Explanation for Incorrect Answer E :


Choice (E) is not correct. This is the fifth number in the sequence.

5
If and what is in terms of

(A)

(B)

(C)

(D)

(E)

file://E:\新建文件夹\a9.htm 2006-11-12
The Official SAT Online Course 页码,4/12

ANSWERS AND EXPLANATIONS


Explanation for Correct Answer E :
Choice (E) is correct. Since the expression can be substituted for
in the equation This yields

Explanation for Incorrect Answer A :


Choice (A) is not correct. See the explanation for the correct response (E).

Explanation for Incorrect Answer B :


Choice (B) is not correct. See the explanation for the correct response (E).

Explanation for Incorrect Answer C :


Choice (C) is not correct. See the explanation for the correct response (E).

Explanation for Incorrect Answer D :


Choice (D) is not correct. See the explanation for the correct response (E).

6 If the average (arithmetic mean) of 5 and


what is the average of and !
is 7 and the average of and is


ed
(A)

(B)

(C)
r 用
te
(D)

(E)


is

ANSWERS AND EXPLANATIONS


eg


Explanation for Correct Answer C :
Choice (C) is correct. Since the average of and is it follows that


therefore, Since the average of and is it
nR


follows that therefore, Thus, the average of and

is


U

Explanation for Incorrect Answer A :


Choice (A) is not correct. is the correct value of but the question asks
for the average of and

Explanation for Incorrect Answer B :


Choice (B) is not correct. See the explanation for the correct response (C).

Explanation for Incorrect Answer D :


Choice (D) is not correct. is the correct value of but the question asks
for the average of and

Explanation for Incorrect Answer E :


Choice (E) is not correct. The sum of and is but the question asks
for the average of and

7
If then

file://E:\新建文件夹\a9.htm 2006-11-12
The Official SAT Online Course 页码,5/12

(A)

(B)

(C)

(D)

(E)

ANSWERS AND EXPLANATIONS


Explanation for Correct Answer A :

Choice (A) is correct. Because you are subtracting from itself, the

expression in parentheses is equal to zero. When is multiplied by the


result is

Explanation for Incorrect Answer B :


Choice (B) is not correct. See the explanation for the correct response (A).

Explanation for Incorrect Answer C :



Choice (C) is not correct. See the explanation for the correct response (A).


ed
Explanation for Incorrect Answer D :
Choice (D) is not correct. See the explanation for the correct response (A).

Explanation for Incorrect Answer E :



er
Choice (E) is not correct. See the explanation for the correct response (A).


st


i


eg


nR



U

The figure on the left is called an ell. The lengths of some of its sides are given,
and all the angles are right angles. For any positive integer n, an n-ell is the
figure formed by positioning n ells adjacent to each other as shown in the 3 -ell
on the right.
8

What is the perimeter of the -ell?

(A)

(B)

(C)

(D)

(E)

ANSWERS AND EXPLANATIONS


Explanation for Correct Answer A :
Choice (A) is correct. The 3-ell has one side of length sides of length
and sides of length Therefore, its perimeter is

file://E:\新建文件夹\a9.htm 2006-11-12
The Official SAT Online Course 页码,6/12

Explanation for Incorrect Answer B :


Choice (B) is not correct. See the explanation for the correct response (A).

Explanation for Incorrect Answer C :


Choice (C) is not correct. is the perimeter of the 3-ell plus the lengths of
the dashed line segments inside the 3-ell. The dashed lines are not part of
the perimeter and should not be counted.

Explanation for Incorrect Answer D :


Choice (D) is not correct. See the explanation for the correct response (A).

Explanation for Incorrect Answer E :


Choice (E) is not correct. is times the perimeter of an ell, but that is
not equal to the perimeter of a 3-ell. In calculating you count twice the
line segments shown as dashed on the 3-ell, but these are not part of the
perimeter of the 3-ell and should not be counted.

9 The perimeter of an -ell is and the perimeter of a -ell is What is the


perimeter of a -ell?
(A)


(B)

(C)


d
(D)

(E)

ANSWERS AND EXPLANATIONS re 用


te
Explanation for Correct Answer A :

Choice (A) is correct. When the 80-ell and the 20-ell are put together to form


is
the 100-ell, they have a common border of length but this border is not
part of the perimeter of the 100-ell. Because the length of this border is
counted twice in adding the perimeters of the 80-ell and the 20-ell, the
eg


perimeter of the 100-ell is less than the sum of the two perimeters; that
is, the perimeter is


nR


Explanation for Incorrect Answer B :
Choice (B) is not correct. See the explanation for the correct response (A).


U

Explanation for Incorrect Answer C :


Choice (C) is not correct. Simply adding the perimeter of the 80-ell and the
20-ell will not give the perimeter of the 100-ell because of the excess length
of the common border.

Explanation for Incorrect Answer D :


Choice (D) is not correct. See the explanation for the correct response (A).

Explanation for Incorrect Answer E :


Choice (E) is not correct. See the explanation for the correct response (A).

10

Which of the following graphs best represents the information in the table above?

file://E:\新建文件夹\a9.htm 2006-11-12
The Official SAT Online Course 页码,7/12

(A)

(B)

(C)



d
re 用

te
(D)


is


eg


nR


(E)


U

ANSWERS AND EXPLANATIONS


Explanation for Correct Answer B :
Choice (B) is correct. From the age of months to the age of months,
the growth of the snake is very rapid, so the graph should have a large
positive slope. This growth is not constant, however; from months to
months, the growth, while still positive, has tapered off greatly. Therefore,
the graph should have a positive slope close to from months to
months. The only graph shown with both of these characteristics is graph
(B).

Explanation for Incorrect Answer A :


Choice (A) is not correct. This graph shows a constant positive growth rate,
but from the data in the table, you can see that the growth rate is not
constant.

Explanation for Incorrect Answer C :


Choice (C) is not correct. This graph shows a constant negative growth rate
(analogous to a shrinking snake). According to the data in the table, not only

file://E:\新建文件夹\a9.htm 2006-11-12
The Official SAT Online Course 页码,8/12

is the growth rate not constant, it is also positive.

Explanation for Incorrect Answer D :


Choice (D) is not correct. This graph shows a growth rate that starts out
slowly and then speeds up. According to the data in the table, just the
opposite happens: the growth rate begins very quickly and then slows down.

Explanation for Incorrect Answer E :


Choice (E) is not correct. This graph shows a growth rate that begins very
quickly and then reverses and turns past to a negative value. This is
analogous to the case where a snake grows quickly, then grows slowly, and
eventually begins to shrink. From the data in the table, you can see that this
is not accurate.

11 1. Add to
2. Multiply the sum by
3. Subtract from the product.

If the steps above are followed in order, which of the following is a simplified
expression for the result?
(A)

(B)

(C)


(D)

d
(E)

ANSWERS AND EXPLANATIONS


re 用

te
Explanation for Correct Answer E :
Choice (E) is correct. After step one, the expression is Multiplying


by two yields the expression and subtracting
is
gives the expression This simplifies to


eg


Explanation for Incorrect Answer A :
nR

Choice (A) is not correct. See the explanation for the correct response (E).


Explanation for Incorrect Answer B :
Choice (B) is not correct. See the explanation for the correct response (E).


U

Explanation for Incorrect Answer C :


Choice (C) is not correct. See the explanation for the correct response (E).

Explanation for Incorrect Answer D :


Choice (D) is not correct. See the explanation for the correct response (E).

12 If is a positive integer, which of the following is equivalent to

(A)

(B)

(C)

(D)

(E)

ANSWERS AND EXPLANATIONS


Explanation for Correct Answer A :
Choice (A) is correct. Adding two identical terms is equivalent to multiplying
the term by

file://E:\新建文件夹\a9.htm 2006-11-12
The Official SAT Online Course 页码,9/12

Explanation for Incorrect Answer B :


Choice (B) is not correct. This would be the answer if the expression given
were

Explanation for Incorrect Answer C :


Choice (C) is not correct. This would be the answer if the expression given
were

Explanation for Incorrect Answer D :


Choice (D) is not correct. This would be the answer if the expression given
were

Explanation for Incorrect Answer E :


Choice (E) is not correct. This would be the answer if the expression given
were

13 During a one-cent sale, a shopper pays the regular price for a bottle of vegetable oil
and pays


for a second bottle. If the regular price of the vegetable oil is
how much per bottle does the shopper save by buying two bottles at this sale?


ed
(A)

(B)


(C) er
(D)

(E)


st
ANSWERS

AND EXPLANATIONS
i


eg

Explanation for Correct Answer B :


Choice (B) is correct. Normally, the shopper pays per bottle of
vegetable oil. During the sale, the shopper pays a total of for two


nR

bottles of oil, which is per bottle of oil. The shopper saves


the difference between the two prices, or per bottle
of oil.


U

Explanation for Incorrect Answer A :


Choice (A) is not correct. is the price of the second bottle of vegetable
oil during the sale.

Explanation for Incorrect Answer C :


Choice (C) is not correct. is the average price of a bottle of vegetable
oil during the sale, but the question asks for the amount saved per bottle
during the sale.

Explanation for Incorrect Answer D :


Choice (D) is not correct. See the explanation for the correct response (B).

Explanation for Incorrect Answer E :


Choice (E) is not correct. is the regular price of a bottle of vegetable
oil.

14
If what is the value of

file://E:\新建文件夹\a9.htm 2006-11-12
The Official SAT Online Course 页码,10/12

(A)

(B)

(C)

(D)

(E)

ANSWERS AND EXPLANATIONS


Explanation for Correct Answer E :
Choice (E) is correct. Cross multiply to get which simplifies

to Dividing both sides of this equation by gives


Explanation for Incorrect Answer A :
Choice (A) is not correct. See the explanation for the correct response (E).


Explanation for Incorrect Answer B :

ed
Choice (B) is not correct. See the explanation for the correct response (E).


Explanation for Incorrect Answer C :
Choice (C) is not correct. See the explanation for the correct response (E).
er

Explanation for Incorrect Answer D :
Choice (D) is not correct. See the explanation for the correct response (E).
st


i

15


eg

A right circular cylinder has a base of circumference If the volume of the cylinder
is what is the height?


(A)
nR

(B)


(C)

(D)


U

(E)

ANSWERS AND EXPLANATIONS


Explanation for Correct Answer B :
Choice (B) is correct. The volume of a right circular cylinder is given by the
formula where is the radius of the base and is the height of
the cylinder. If the base has a circumference of it must have a diameter
of and thus a radius of You can then write the volume equation as

which simplifies to

Explanation for Incorrect Answer A :


Choice (A) is not correct. is the radius of the cylinder, not the height.

Explanation for Incorrect Answer C :


Choice (C) is not correct. See the explanation for the correct response (B).

Explanation for Incorrect Answer D :


Choice (D) is not correct. If the area of the base of the cylinder were the
height of the cylinder would be but the question states that the
circumference of the base is not the area.

file://E:\新建文件夹\a9.htm 2006-11-12
The Official SAT Online Course 页码,11/12

Explanation for Incorrect Answer E :


Choice (E) is not correct. See the explanation for the correct response (B).

16 In the -coordinate plane, the graph of intersects line at and

What is the least possible value of the slope of


(A)

(B)

(C)

(D)

(E)

ANSWERS AND EXPLANATIONS


Explanation for Correct Answer D :
Choice (D) is correct. The slope of line equals

or To find values for and

and lie on the graph of !


you can use the equation of the parabola. Notice that since the points
their coordinates can be


d
substituted for and in the equation Substituting for
and for in the equation gives This simplifies to

and for
and further to
in the equation gives
re 用 so Substituting for
This simplifies to
te

and further to so

Since there are two values for each variable, there are four possible values


is
for the expression and
To find the least possible value of the slope of line


eg

you must compare all four possible values of the slope, which is equal to

and The least possible value of the

slope is


nR



U

Explanation for Incorrect Answer A :


Choice (A) is not correct. While the slope of line could be this is not the
least possible value of the slope, and the question asks for the least possible
value.

Explanation for Incorrect Answer B :


Choice (B) is not correct. While the slope of line could be this is not the
least possible value of the slope, and the question asks for the least possible
value.

Explanation for Incorrect Answer C :


Choice (C) is not correct. While the slope of line could be this is not
the least possible value of the slope, and the question asks for the least
possible value.

Explanation for Incorrect Answer E :


Choice (E) is not correct. See the explanation for the correct response (D).

Back to Score Report

Copyright © 2006 The College Board. All rights reserved. Privacy Policy Terms of Use Contact Us

file://E:\新建文件夹\a9.htm 2006-11-12
The Official SAT Online Course 页码,12/12



r

ed

te

is


eg


nR



U

file://E:\新建文件夹\a9.htm 2006-11-12
The Official SAT Online Course 页码,1/9

Help | Profile | My Organizer | My Bookmarks | Logout

Answers and Explanations

Test Sections Back to Score Report

Section 1 View Answers and Explanations


Section 2 Online - Practice Test #1

Section 3
1 In their zeal to make beachfront living widely available, developers have overbuilt,
Section 5 thereby they endanger fragile coastlines.
Section 6
(A) overbuilt, thereby they endanger fragile coastlines
Section 7
Section 8 (B) overbuilt they endanger fragile coastlines as a result
Section 9 (C) overbuilt and thereby have endangered fragile coastlines
Section 10
(D) overbuilt; fragile coastlines endangered thereby

(E) overbuilt, the fragile coastlines are endangered by this

ANSWERS AND EXPLANATIONS




d
Explanation for Correct Answer C :
Choice (C) is correct. It avoids the error of the original by changing the second

e 用
independent clause to a verb phrase ("have endangered") that is parallel to the
earlier verb phrase ("have overbuilt") in the new compound predicate.
er

t

is
Explanation for Incorrect Answer A :
Choice (A) uses improper coordination. It joins two complete thoughts ("In their
zeal . . . developers have overbuilt" and "thereby they endanger fragile coastlines")


eg

with only a comma.

Explanation for Incorrect Answer B :


Choice (B) displays improper coordination. It places one complete thought ("they
nR

endanger fragile coastlines as a result") immediately after another ("In their zeal . .
. developers have overbuilt") with no conjunction or punctuation between them.


Explanation for Incorrect Answer D :
Choice (D) exhibits improper coordination. It uses a semicolon to join unequal


U

sentence parts (a complete thought before the semicolon and a phrase after it).

Explanation for Incorrect Answer E :


Choice (E) involves improper coordination. It joins two complete thoughts ("In their
zeal . . . developers have overbuilt" and "the fragile coastlines are endangered by
this") with only a comma.

2
Hawaii’s Haleakala, being more than 10,000 feet high, and the world’s largest
dormant volcano.

(A) Haleakala, being more than 10,000 feet high and


(B) Haleakala, more than 10,000 feet high, it is
(C) Haleakala which is more than 10,000 feet high, being

(D) Haleakala, more than 10,000 feet high, is

(E) Haleakala, more than 10,000 feet high; it is

ANSWERS AND EXPLANATIONS


Explanation for Correct Answer D :
Choice (D) is correct. It avoids the error of the original by providing the verb "is" to
complete the predicate of the sentence.

file://E:\新建文件夹\a10.htm 2006-11-12
The Official SAT Online Course 页码,2/9

Explanation for Incorrect Answer A :


Choice (A) is a sentence fragment. The sentence is grammatically incomplete
because it has no main verb.

Explanation for Incorrect Answer B :


Choice (B) is unsatisfactory because the word "it" introduces an independent clause
("it is... volcano"). The opening phrase ("Haleakala... high") now has no verb and is
left unattached to the sentence.

Explanation for Incorrect Answer C :


Choice (C) results in a sentence fragment. The use of an improper verb form
("being" instead of "is") leaves the sentence grammatically incomplete.

Explanation for Incorrect Answer E :


Choice (E) involves improper coordination. The clause preceding the semicolon is
incomplete because it has no main verb.

3 I do not blame Leslie for her anger yesterday, being it was her plan and she should
have credit for it.

(A)

yesterday, being it was her plan and she should have credit for it


(B) yesterday, being that she should have credit for it when it was her plan

d
(C) yesterday when it was her plan, for which they should give her credit for it

e 用
(D) yesterday; since she should receive credit, it being her plan

(E)
er
yesterday: it was her plan, and she should have received credit for it

ANSWERS AND EXPLANATIONS



st


Explanation for Correct Answer E :
Choice (E) is correct. It avoids the error of the original by using a colon between
i


eg

the main clauses as is appropriate when the second clause explains the idea in the
first clause.


nR


Explanation for Incorrect Answer A :
Choice (A) involves the use of an improper idiom. The word "being" is used to
introduce the clause, "it was her plan," where it would be more idiomatic to


U

introduce it with "because" or to separate the clauses with a colon.

Explanation for Incorrect Answer B :


Choice (B) is unsatisfactory because it involves the use of an improper idiom. The
clause, "she should have credit for it," is introduced by the phrase, "being that,"
where it would be more idiomatic to introduce it with "since" or "because."

Explanation for Incorrect Answer C :


Choice (C) involves the use of a vague pronoun. It is not clear to whom the
pronoun "they" refers.

Explanation for Incorrect Answer D :


Choice (D) involves a lack of clarity. The confusing word order makes the meaning
of the sentence unclear.

4 At the time at which temperatures approach absolute zero, or -459.7° F, metals


become highly conductive, and their volume shrinks dramatically.

(A) At the time at which temperatures approach


(B) When temperatures approach
(C) Since temperatures approached

(D) At the point temperatures had approached

file://E:\新建文件夹\a10.htm 2006-11-12
The Official SAT Online Course 页码,3/9

(E) While temperatures approaching

ANSWERS AND EXPLANATIONS


Explanation for Correct Answer B :
Choice (B) is correct. It avoids the error of the original by providing the adverb
"When" instead of the repetitive adverbial phrase, "At the time at which."

Explanation for Incorrect Answer A :


Choice (A) involves unnecessary repetition. The phrase "At the time at which" is
used to introduce the subordinate clause, where the adverb "When" would be
simpler and clearer.

Explanation for Incorrect Answer C :


Choice (C) involves an improper sequence of verb tenses. It provides a past-tense
verb ("approached") instead of the present-tense verb that is needed to match the
present-tense verbs in the main clause of the sentence.

Explanation for Incorrect Answer D :


Choice (D) is unsatisfactory because it involves an improper sequence of verb
tenses. The pluperfect verb "had approached" does not match the present tense of
the other verbs in the sentence.


d
Explanation for Incorrect Answer E :
Choice (E) involves the use of an improper idiom. It provides the phrase "While

re
temperatures approaching absolute zero," where it would be more idiomatic to use
a clause introduced by the adverb "When."



te
5 Participants in the executive leadership workshop expect a program of outstanding


is
speakers and gaining information about new approaches to management.

(A) and gaining information


eg

(B) as well as information


(C) as well as being informed


(D) and also being informed
nR

(E) in addition, they expect to gain information


ANSWERS AND EXPLANATIONS


U

Explanation for Correct Answer B :


Choice (B) is correct. It avoids the error of the original by using two parallel nouns,
"information" and "program," to name the two things participants expect.

Explanation for Incorrect Answer A :


Choice (A) fails to maintain parallelism. The verbal phrase "gaining information" is
not parallel with the earlier noun "program."

Explanation for Incorrect Answer C :


Choice (C) does not maintain parallelism. The verbal phrase "being informed" is not
parallel with the earlier noun "program."

Explanation for Incorrect Answer D :


Choice (D) has a flaw in parallelism. The verbal phrase "being informed" is not
parallel with the earlier noun "program."

Explanation for Incorrect Answer E :


Choice (E) exhibits improper coordination. It places one complete thought ("in
addition . . . gain information") immediately after another ("Participants . . . of
outstanding speakers") with no conjunction or punctuation to connect them.

file://E:\新建文件夹\a10.htm 2006-11-12
The Official SAT Online Course 页码,4/9

6 One of the unforeseen consequences of the editor’s management style is that it


leaves so little room for innovation.
(A) that it leaves so little room for innovation
(B) that they leave so little room for innovation
(C) that sufficient room is not left for their innovation

(D) that innovation has so little room left from it

(E) to leave so little room for innovation

ANSWERS AND EXPLANATIONS


Explanation for Correct Answer A :
Choice (A) is correct. It appropriately uses a subordinate noun clause, introduced
by the relative pronoun "that," to describe a result of the editor's management
style.

Explanation for Incorrect Answer B :


Choice (B) involves noun-pronoun disagreement. It improperly uses the plural
pronoun "they" to refer to the singular phrase "the editor's management style."


ed
Explanation for Incorrect Answer C :
Choice (C) involves the use of a vague pronoun. There is nothing in the sentence to
which the pronoun "their" can logically refer.

r
Explanation for Incorrect Answer D :

Choice (D) results in an illogical sentence. The phrase "innovation has so little room


te
left from it" makes no sense.


Explanation for Incorrect Answer E :
is
Choice (E) involves the use of an improper idiom. It provides an infinitive phrase
("to leave... innovation") where it would be more idiomatic to use a subordinate
clause introduced by the relative pronoun "that."


eg


nR

7
For decades, African American music has inspired musicians throughout the world,


including in Russia.

(A) including in Russia


(B)
U

including those of Russia


(C) this includes Russia

(D) one of which is Russia

(E) one example being Russia

ANSWERS AND EXPLANATIONS


Explanation for Correct Answer B :
Choice (B) is correct. It avoids the error of the original by changing the vague
phrase "in Russia" to "those of Russia," where the pronoun "those" clearly refers to
musicians.

Explanation for Incorrect Answer A :


Choice (A) involves the use of a vague modifying phrase. It is not clear whether the
phrase "in Russia" refers to inspiration or to musicians.

Explanation for Incorrect Answer C :


Choice (C) involves the use of a vague pronoun. It is not clear what the pronoun
"this" refers to.

Explanation for Incorrect Answer D :

file://E:\新建文件夹\a10.htm 2006-11-12
The Official SAT Online Course 页码,5/9

Choice (D) results in an illogical sentence. There is nothing in the sentence to which
the pronoun "which" can logically refer.

Explanation for Incorrect Answer E :


Choice (E) is unsatisfactory because it results in an illogical sentence. There is
nothing in the sentence of which Russia could logically be an example.

8 By the end of the eighteenth century, watchmaking technology had greatly improved,
and they were standard equipment for military personnel.
(A) and they were standard equipment for military personnel
(B) so it was standard equipment for military personnel to have watches
(C) with watches included in the standard equipment for military personnel

(D) and watches had become standard equipment for military personnel

(E) and for military personnel it was standard equipment

ANSWERS AND EXPLANATIONS


Explanation for Correct Answer D :


Choice (D) is correct. It avoids the error of the original by providing the noun
"watches" in place of the vague pronoun "they."


ed 用
Explanation for Incorrect Answer A :
Choice (A) involves the use of a vague pronoun. There is no plural noun in the
r
sentence to which the pronoun "they" can logically refer.


te
Explanation for Incorrect Answer B :
Choice (B) involves the use of an improper idiom. It uses the phrase "it was


is
standard equipment for... watches" where it would be more idiomatic to say
"watches were standard equipment."


eg

Explanation for Incorrect Answer C :


Choice (C) results in an illogical sentence. The linking word "with" illogically
indicates that the issuing of watches to military personnel was a part of the


improvement in watchmaking technology rather than a result of it.
nR

Explanation for Incorrect Answer E :


Choice (E) involves the use of a vague pronoun. It is not clear what the pronoun
"it" refers to.


U

9 Nancy and Carlos will represent Central High in the swimming competition, their work
in this having been excellent this year.

(A) competition, their work in this having been excellent this year
(B) competition, they have done excellent work this year in this
(C) competition, for this year they have done excellent work in this

(D) competition, for their swimming has been excellent this year

(E) competition, their work as swimmers having been excellent this year

ANSWERS AND EXPLANATIONS


Explanation for Correct Answer D :
Choice (D) is correct. It avoids the error of the original by using the specific noun
"swimming" to replace a wordy phrase ("work in this") containing a vague pronoun,
"this."

Explanation for Incorrect Answer A :


Choice (A) involves wordiness and use of a vague pronoun. In the phrase "work in

file://E:\新建文件夹\a10.htm 2006-11-12
The Official SAT Online Course 页码,6/9

this," the meaning of the pronoun "this" is unclear.

Explanation for Incorrect Answer B :


Choice (B) displays improper coordination. It joins two complete thoughts ("Nancy
and Carlos will represent . . . swimming competition" and "they have done excellent
work this year in this") with only a comma.

Explanation for Incorrect Answer C :


Choice (C) uses an ambiguous pronoun. The pronoun "this" could refer either to
swimming or to swimming competition.

Explanation for Incorrect Answer E :


Choice (E) exhibits wordiness. The phrase "work as swimmers" can be reduced to
one word, "swimming."

10 After 1907, residents of the Omaha Reservation could use the hospital in Walthill,
Nebraska, it was established by Dr. Susan LaFlesche Picotte, an Omaha Indian.

(A) hospital in Walthill, Nebraska, it was established by Dr. Susan LaFlesche


Picotte, an Omaha Indian
(B) hospital; it was in Walthill, Nebraska and established by Dr. Susan
LaFlesche Picotte, an Omaha Indian
(C) hospital that has been established by Dr. Susan LaFlesche Picotte, an
Omaha Indian, in Walthill, Nebraska


(D) Walthill, Nebraska, hospital where an Omaha Indian, Dr. Susan LaFlesche
Picotte, established it


d
(E) hospital established in Walthill, Nebraska, by Dr. Susan LaFlesche Picotte,
an Omaha Indian

ANSWERS AND EXPLANATIONS re 用



te
Explanation for Correct Answer E :
Choice (E) is correct. It avoids the error of the original by replacing the clause


introduced by "it was" with an appropriate modifying phrase, "established... Omaha
is
Indian," to describe the hospital.


eg


Explanation for Incorrect Answer A :
nR

Choice (A) involves improper coordination. Two complete thoughts are linked by
only a comma.


Explanation for Incorrect Answer B :
Choice (B) involves awkward and illogical phrasing. The sentence does not make


U

sense because the word "and" makes the hospital appear to have existed before it
was established.

Explanation for Incorrect Answer C :


Choice (C) involves an improper sequence of tenses. The tense of the verb phrase
“has been established” does not combine logically with the tense of the verb
phrase “could use” earlier in the sentence.

Explanation for Incorrect Answer D :


Choice (D) involves the use of a vague pronoun. It is not clear what the pronoun
"it" refers to.

11

Eating food that has a high concentration of fat causes essentially the same reaction
in the stomach than if you eat too fast.

(A) than if you eat


(B) than to eat
(C) as if one eats

(D) as eating

(E) as it does when eating

file://E:\新建文件夹\a10.htm 2006-11-12
The Official SAT Online Course 页码,7/9

ANSWERS AND EXPLANATIONS


Explanation for Correct Answer D :
Choice (D) is correct. It avoids the error of the original by using the word "as,"
which pairs with the earlier word "same" to create an appropriate idiom indicating
similarity.

Explanation for Incorrect Answer A :


Choice (A) uses an improper idiom. After the word “same,” the conjunction
“than” is inappropriate because it indicates difference rather than similarity.

Explanation for Incorrect Answer B :


Choice (B) contains an improper idiom. After the word "same," the conjunction
"than" is inappropriate because it indicates difference rather than similarity.

Explanation for Incorrect Answer C :


Choice (C) fails to maintain parallelism. The clause "if one eats" is not parallel with
the earlier verbal "eating."

Explanation for Incorrect Answer E :


Choice (E) exhibits wordiness. The words "it does when" are not needed.


d
12 Not one of the students in the advanced chemistry class have passed a single test

(A)
re
with a grade better than a C, but the second half of the course will be easier.


have passed a single test with a grade better than a C


te
(B) have managed to pass a single test with better than a C grade
(C) have passed a single test any better than a grade of C


is
(D) has passed having better than a C grade on a single test

(E) has passed a single test with better than a C grade


eg

ANSWERS AND EXPLANATIONS


Explanation for Correct Answer E :
nR

Choice (E) is correct. It avoids the error of the original by making the singular verb


"has" agree with its singular subject, "one."


U

Explanation for Incorrect Answer A :


Choice (A) has an error in subject-verb agreement. Even though the plural verb
"have" agrees with the interrupting noun "students," it does not agree with its
singular subject, "one."

Explanation for Incorrect Answer B :


Choice (B) contains an error in subject-verb agreement. The plural verb "have"
agrees with the interrupting noun "students," but not with with its singular subject,
"one."

Explanation for Incorrect Answer C :


Choice (C) exhibits an error in subject-verb agreement. While the plural verb
"have" does agree with the interrupting noun "students," it does not agree with its
singular subject, "one."

Explanation for Incorrect Answer D :


Choice (D) displays loose modification. This word order fails to make clear exactly
what the verbal phrase "having better than a C grade on a single test" modifies.

13 In neighborhoods throughout the United States, one can encounter hundreds of


different rope-jumping games, each with its own rules.

file://E:\新建文件夹\a10.htm 2006-11-12
The Official SAT Online Course 页码,8/9

(A) each with its own rules


(B) each having their own rules
(C) when they each have their own rules

(D) which has its own rules

(E) they each have rules of their own

ANSWERS AND EXPLANATIONS


Explanation for Correct Answer A :
Choice (A) is correct. The singular pronoun "its" agrees with the earlier singular
pronoun "each."

Explanation for Incorrect Answer B :


Choice (B) involves a shift in pronoun number. The plural pronoun "their" does not
agree with the earlier singular pronoun "each."

Explanation for Incorrect Answer C :


Choice (C) uses ineffective subordination. The conjunction "when" incorrectly
suggests that the main relationship between the two clauses is temporal.


ed
Explanation for Incorrect Answer D :
Choice (D) displays disagreement of subject and verb and also of noun and
pronoun. Although the pronoun "which" refers to the plural noun "games," neither


the singular verb "has" nor the singular pronoun "its" agrees with that noun.
er
Explanation for Incorrect Answer E :


Choice (E) uses improper coordination. It joins two complete thoughts ("In
neighborhoods . . . rope-jumping games" and " they each . . . of their own") with
st
only a comma.


i


eg

14

A flurry of do-it-yourself books on the market today are inspiring homeowners to do


their own repairs.
nR

(A) are inspiring homeowners to do their own repairs


(B) are inspiring to homeowners about their own repairs
(C) is inspiring homeowners into doing their own repairing


U

(D) is inspiring homeowners to do their own repairs

(E) inspiring homeowners to repair their own homes

ANSWERS AND EXPLANATIONS


Explanation for Correct Answer D :
Choice (D) is correct. It avoids the error of the original by making the singular verb
"is" agree with its singular subject, "flurry."

Explanation for Incorrect Answer A :


Choice (A) has an error in subject-verb agreement. Even though the plural verb
"are" agrees with the interrupting noun "books," it does not agree with its singular
subject, "flurry."

Explanation for Incorrect Answer B :


Choice (B) contains an error in subject-verb agreement. The plural verb "are"
agrees with the interrupting noun "books," but not with with its singular subject,
"flurry."

Explanation for Incorrect Answer C :


Choice (C) uses inappropriate idioms. After the verb "is inspiring," the phrase "into
doing" is less effective than "to do," and "repairing" is less effective than "repairs."

file://E:\新建文件夹\a10.htm 2006-11-12
The Official SAT Online Course 页码,9/9

Explanation for Incorrect Answer E :


Choice (E) creates a sentence fragment. Since the phrase has no verb (only the
verbal forms "inspiring" and "to repair"), it does not state a complete thought.

Back to Score Report

Copyright © 2006 The College Board. All rights reserved. Privacy Policy Terms of Use Contact Us

更多SAT考试资料,请登录http://sat.tiandaoedu.com获取



ed 用
er

st


i


eg


nR



U

file://E:\新建文件夹\a10.htm 2006-11-12

You might also like